Re: Irreducible randomness in QM

2021-04-24 Thread Bruno Marchal

> On 28 Dec 2020, at 20:22, Alan Grayson  wrote:
> 
> 
> 
> On Monday, December 28, 2020 at 11:47:04 AM UTC-7 Alan Grayson wrote:
> On Monday, December 28, 2020 at 9:17:23 AM UTC-7 Bruno Marchal wrote:
>> On 26 Dec 2020, at 16:41, Alan Grayson > wrote:
>> 
>> Fact is there's no need to bring in pretentious big brains from Australia to 
>> show that Born's rule makes no sense in the context of the MWI. For 
>> simplicity, consider a spin experiment where the observer measures spin UP 
>> in THIS world, and another world (or branch) is created where spin DN is 
>> measured. If we do the experiment again, the MWI is AMBIGUOUS on which world 
>> (or branch) the spin is now measured.
> 
> Why? 
> Not at all. This is already well explained in Everett’s original long paper. 
> In the branch where the observer has measured spin up, he knows that if he 
> redo quickly the measurement, he will find the same result. The measurement 
> only entangle the spin state with his memory. 
> 
> It would be refreshing if you knew what this topic is about. Entanglement has 
> nothing to do with this. We're creating ENSEMBLES of measurements in THIS 
> world; repeatable trials of say spin along some axis.  For each TRIAL, a new 
> world or branch is created if you adopt the MWI. ONE measurement in each of 
> these NEW worlds or branches. No ensemble; hence no probabilities! It's 
> really simple, so please don't bring in your pet theory to obfuscate the 
> scenario. AG
> 
> Or you can look at it this way; when repeated trials of the same measurement 
> are done in this world,

I guess you mean repeated measurement of spin made of spin superposition




> different worlds come into existence for EACH trial if you accept the MWI.


I don’t really believe in “world”, and I prefer to avoid ontological 
commitment. I do not accept the MWI, what happens is that I do not make sense 
of the collapse postulate. 



> Sometimes Alice leaves for a pee, and Bob continues to create the ensemble. 
> No ensembles are created in those other worlds. Hence, no probabilities, and 
> Born's rule doesn't apply. AG 


Of course it does. Ir creates ensemble of Alices and Bobs, well correlated in 
their respective histories. Your statement should be much more elaborate. It is 
too vague here.

Bruno





> 
> 
> 
> 
>> On one of the two worlds just created, or are another pair of worlds 
>> created, one of which measures spin UP and another spin DN?  Of course this 
>> leaves out all the BS on how these worlds are created, along with the 
>> spontaneous creation of all the observers and experimental set-ups to do the 
>> measuring. Better continue to mentally jerkoff in the assumption, for the 
>> sake of argument, that there's some serious interpretation of QM going on 
>> here. AG
> 
> It is simpler to reason in term of histories. That is mandatory with 
> mechanism, but the work of Graham, Hartle, Griffith, Omnes (and even Feynman 
> paths) suggest this is already true in physics. 
> 
> The notion of “world” is a tricky notion. 
> 
>> 
>> Now as for the problem presented by the title of this thread, if the NO-GO 
>> theorems correctly affirm that FTL signaling is impossible, what EXACTLY is 
>> the link between this conclusion and the claim the QM is IRREDUCIBLY RANDOM? 
>> And what does IRREDUCIBLY RANDOM mean? In trying to answer this question, 
>> let me impose one RULE; no jerking off permitted! AG
> 
> 
> Irreducibly random means that we cannot predict the particular result that we 
> will obtained.
> 
> So, based on HUP, we can't do that for a conjugate pair, like position and 
> momentum. Based solely on HUP are pairs of conjugate measurements irreducibly 
> random? If you want a snotty reply, ask the Guru from Australia.  AG
> 
> Imagine that the randomness was reducible. Imagine that Bob and Alice have a 
> perfectly correlated dice, when at a light year of distance between each 
> other. When Alice measure 5, Bob measure 5, etc. (as seen by some third 
> observer to gate a local simultaneity notion). Their result have to be 
> irreducibly random, because if their well able to predict their outcome, 
> without invoking a super-determinacy, if they could predict the result, they 
> would be able to use their correlation (entanglement) tp transmit a message 
> FTL. 
> Note that in 1927 Einstein saw already that the collapse of the wave packet 
> needs to be FTL, and Bohr, in his paper answering the EPR paper, admit that 
> the collapse cannot be a physical process, but then get very fuzzy when 
> attempting to describe what happen.
> I leave you the details, as I have to go. 
> 
> Bruno
> 
> 
> 
> 
> 
> -- 
> You received this message because you are subscribed to the Google Groups 
> "Everything List" group.
> To unsubscribe from this group and stop receiving emails from it, send an 
> email to everything-list+unsubscr...@googlegroups.com 
> .
> To view this discussion on the web visit 

Re: Irreducible randomness in QM

2021-04-24 Thread Bruno Marchal

> On 28 Dec 2020, at 19:47, Alan Grayson  wrote:
> 
> 
> 
> On Monday, December 28, 2020 at 9:17:23 AM UTC-7 Bruno Marchal wrote:
>> On 26 Dec 2020, at 16:41, Alan Grayson > > wrote:
>> 
>> Fact is there's no need to bring in pretentious big brains from Australia to 
>> show that Born's rule makes no sense in the context of the MWI. For 
>> simplicity, consider a spin experiment where the observer measures spin UP 
>> in THIS world, and another world (or branch) is created where spin DN is 
>> measured. If we do the experiment again, the MWI is AMBIGUOUS on which world 
>> (or branch) the spin is now measured.
> 
> Why? 
> Not at all. This is already well explained in Everett’s original long paper. 
> In the branch where the observer has measured spin up, he knows that if he 
> redo quickly the measurement, he will find the same result. The measurement 
> only entangle the spin state with his memory. 
> 
> It would be refreshing if you knew what this topic is about. Entanglement has 
> nothing to do with this. We're creating ENSEMBLES of measurements in THIS 
> world; repeatable trials of say spin along some axis.  For each TRIAL, a new 
> world or branch is created if you adopt the MWI. ONE measurement in each of 
> these NEW worlds or branches. No ensemble; hence no probabilities! It's 
> really simple, so please don't bring in your pet theory to obfuscate the 
> scenario. AG

In the MWI, measurement is entanglement. You don’t address my answer. Today we 
can isolate one atome, or even one electron, and this does not change the 
statistics. The “MWI” should not be interpreted in a two much naïve way, as the 
notion of world is never defined. A particle with a precise position is the 
same as infinitely many world with the particle having each possible momentum 
in each world. Maybe “relative state” theory is a better, more neutral 
ontologically, expression for this. Everett uses what you call “my pet theory”. 
He just does not push its logic to its limit, where we can understand that the 
wave itself or the Heisenberg matrices arise from all computations, not just 
the quantum one, and that is nice as it explains where the quantum comes from.




> 
> 
> 
> 
>> On one of the two worlds just created, or are another pair of worlds 
>> created, one of which measures spin UP and another spin DN?  Of course this 
>> leaves out all the BS on how these worlds are created, along with the 
>> spontaneous creation of all the observers and experimental set-ups to do the 
>> measuring. Better continue to mentally jerkoff in the assumption, for the 
>> sake of argument, that there's some serious interpretation of QM going on 
>> here. AG
> 
> It is simpler to reason in term of histories. That is mandatory with 
> mechanism, but the work of Graham, Hartle, Griffith, Omnes (and even Feynman 
> paths) suggest this is already true in physics. 
> 
> The notion of “world” is a tricky notion. 
> 
>> 
>> Now as for the problem presented by the title of this thread, if the NO-GO 
>> theorems correctly affirm that FTL signaling is impossible, what EXACTLY is 
>> the link between this conclusion and the claim the QM is IRREDUCIBLY RANDOM? 
>> And what does IRREDUCIBLY RANDOM mean? In trying to answer this question, 
>> let me impose one RULE; no jerking off permitted! AG
> 
> 
> Irreducibly random means that we cannot predict the particular result that we 
> will obtained.
> 
> So, based on HUP,

What is HUP?




> we can't do that for a conjugate pair, like position and momentum. Based 
> solely on HUP are pairs of conjugate measurements irreducibly random?

Yes. It has to be random, or you could use entanglement to sent information in 
a faster than light manner.

Bruno




> If you want a snotty reply, ask the Guru from Australia.  AG
> 
> Imagine that the randomness was reducible. Imagine that Bob and Alice have a 
> perfectly correlated dice, when at a light year of distance between each 
> other. When Alice measure 5, Bob measure 5, etc. (as seen by some third 
> observer to gate a local simultaneity notion). Their result have to be 
> irreducibly random, because if their well able to predict their outcome, 
> without invoking a super-determinacy, if they could predict the result, they 
> would be able to use their correlation (entanglement) tp transmit a message 
> FTL. 
> Note that in 1927 Einstein saw already that the collapse of the wave packet 
> needs to be FTL, and Bohr, in his paper answering the EPR paper, admit that 
> the collapse cannot be a physical process, but then get very fuzzy when 
> attempting to describe what happen.
> I leave you the details, as I have to go. 
> 
> Bruno
> 
> 
> 
> 
> 
> -- 
> You received this message because you are subscribed to the Google Groups 
> "Everything List" group.
> To unsubscribe from this group and stop receiving emails from it, send an 
> email to everything-list+unsubscr...@googlegroups.com 
> .
> To view this 

Re: Irreducible randomness in QM

2021-02-06 Thread Bruno Marchal

> On 29 Dec 2020, at 00:09, 'Brent Meeker' via Everything List 
>  wrote:
> 
> 
> 
> On 12/28/2020 7:18 AM, Bruno Marchal wrote:
>> 
>>> On 25 Dec 2020, at 05:37, 'Brent Meeker' via Everything List 
>>> >> > wrote:
>>> 
>>> That the Born rule doesn't derive from the Schroedinger equation doesn't 
>>> bother me.  Gleason's theorem guarantees it's the only consistent 
>>> probability measure in the eigenstates of an observable.  The question 
>>> seems to be why is there probability at all.  But I see the 
>>> unpredictability of measured values and that intrinsic randomness is 
>>> necessary to protect relativity theory.  So randomness per the Born rule 
>>> seems to be the obvious theoretical choice. 
>> 
>> Yes, both inferred from observation, and derived from Mechanism.
>> 
>> 
>>> But then why would I entertain the multiverse. 
>> 
>> Because the many computations is a fact (cf 1930s).
>> We can’t delete them from the arithmetical reality, like we can’t delete the 
>> prime numbers.
> 
> Fine.  Leave them in arithmetic.

But when assuming Indexical Digital Mechanism, that’s enough to enforce the 
constructive reduction of physics to arithmetic. And as long as this works, 
adding something simply does not work, it has to be retrieved from 
arithmetic/computer science, or it becomes “magic”.



> 
>> 
>> 
>> 
>> 
>> 
>>> If I know when I've made a measurement then I know when the wave funcion 
>>> "collapsed”.
>> 
>> Assuming that you don’t obey to quantum mechanics. But if QM applies to all 
>> physical objects, then it is simpler to use the first person indeterminacy 
>> on the superposition. Of course, that requires also to derive the wave from 
>> the mathematics of self-reference, which is done at the propositional level 
>> already.
> 
> But what does "self-reference" mean if there is no unique self. 

That is exactly what the mathematics of self-reference (Gödel, Löb, Solovay) 
makes possible to clarify. Self-reference is Tarski theory of truth 
self-applied. You can write a program capable of computing the number of “goto” 
in its own code, and if he got it right, it is self-referentially correct. I am 
not sure I understand why a self has to be unique to be self-referentially 
correct. “I have six legs” is correct if asserted by a machine which has six 
legs, like “the snow is white” is true if the snow is white.




> You are fond of pointing to consciousness as indubitable, but it is equally 
> indubitable that it is unitary.

How so?

The indubitability of consciousness is already in Descartes, and usually well 
understood.

Unitarity concerns a third person description related to a theory which we 
infer from (conscious) observation, and is already doubted by many people 
trying alternate theory, like GWR QM, or like the whole set of Copenhagen QM in 
physics.

Again, I cannot make much sense of your comment here.



> 
>> 
>> 
>> 
>>>   If I don't know when I've made a measurement, then I have more serious 
>>> problems than just the interpretation of QM.
>> 
>> That is the problem with the collapse: you need to define what is a 
>> measurement, and how it succeeds in NOT being described by the wave function.
> 
> A measurement is what is perceived. 

Yes. I am OK with this. That is why I said that the screen can hardly be taken 
as an observation, or a measurement that we can try to relate to a first person 
experience. Without taking the first person perception seriously, by a machine 
with no memory for example, we avoid the indeterminacy, but we fail on the 
mind-body relation for exactly that reason.



> The "dualism" is just that some events happen and some don’t.

You are quick here. I suspect the usual dismiss of consciousness… (which is not 
part of physics, unless you assume physicalism, but then, what I did show that 
you need to abandon the mechanist theory of mind). To be short, I show that 
Descartes and Aristotle cannot be taken together.

Bruno



> 
> Brent
> 
>> It requires some ontological dualism no more coherent with any causal theory 
>> of the mind (not just mechanism BTW, although mechanism makes the task of 
>> showing this much easier).
>> 
>> Bruno
>> 
>> 
>>> 
>>> Brent
>>> 
>>> On 12/24/2020 1:04 PM, Stathis Papaioannou wrote:
 
 
 On Thu, 24 Dec 2020 at 11:51, Bruce Kellett >>> > wrote:
 On Thu, Dec 24, 2020 at 5:39 AM Stathis Papaioannou >>> > wrote:
 
 You suspected right, I am asking a more basic question about self-sampling 
 and the validity of probabilities when a version of the observer sees all 
 possible outcomes.
 
 There is a problem here -- or maybe it is just careless phrasing. You say 
 there is a question about probabilities when a version of the observer 
 sees all possible outcomes. The question is whether it is merely a version 
 of the observer, a copy of the observer, or the actual observer 

Re: Irreducible randomness in QM

2020-12-28 Thread Alan Grayson

*Typo correction; That WAS recently on this MB.*
On Monday, December 28, 2020 at 12:13:45 PM UTC-7 Alan Grayson wrote:

>
> *Bruce; remember when Brent claimed that physics doesn't PROVE anything? 
> That wasn't recently on this MB. But when you wrote that irreducible 
> randomness GUARANTEES no FTL signaling, you affirmed the existence of a 
> PROOF. That was at the heart of my question; the guarantee or PROOF. But 
> because you're a snotty and presumptuous individual, you couldn't see the 
> depth underlying my question, and how your claim contradicts Brent's 
> confident assertion about what physics does and doesn't do. That is, If you 
> were a fair minded gentleman -- which is obviously not the case -- you 
> would have simply referred me to the no-signaling theorems, and then I 
> might be able to determine the status of your claim. You once asked me if I 
> regarded you as my teacher. Again, the answer the NO. You don't have the 
> emotional requirements to be anyone's teacher. AG*
> On Friday, December 25, 2020 at 11:03:51 PM UTC-7 Alan Grayson wrote:
>
>>
>> *You could have omitted your snotty second sentence. Why is it OK for you 
>> to use the List to mentally jerk off about the applicability of Born's rule 
>> in the context of an interpretation you know is false, the MWI, than not 
>> for me to ask a question which might have a simple answer, or path to take 
>> to answer MY question? Being well informed about physics doesn't mean 
>> you're not behaving like an arrogant schmuck. AG*
>> On Thursday, December 24, 2020 at 11:49:42 PM UTC-7 Bruce wrote:
>>
>>> On Fri, Dec 25, 2020 at 5:29 PM Alan Grayson  
>>> wrote:
>>>

 *Now I raise a similar question I posed to Bruce, thrice, with no 
 replies. Why does the unpredictability of measured values and the 
 intrinsic 
 randomness protect relativity theory? This is really a huge conceptual 
 leap. How would you argue for that conclusion, as distinguished from 
 asserting it? TIA, AG*
>>>
>>>
>>> You need to do some research on the no-signalling theorems. This list 
>>> does not exist to answer your elementary questions.
>>>
>>> Bruce
>>>
>>

-- 
You received this message because you are subscribed to the Google Groups 
"Everything List" group.
To unsubscribe from this group and stop receiving emails from it, send an email 
to everything-list+unsubscr...@googlegroups.com.
To view this discussion on the web visit 
https://groups.google.com/d/msgid/everything-list/3237f383-a2e7-4ec3-8432-a10313e9771cn%40googlegroups.com.


Re: Irreducible randomness in QM

2020-12-28 Thread 'Brent Meeker' via Everything List



On 12/28/2020 7:18 AM, Bruno Marchal wrote:


On 25 Dec 2020, at 05:37, 'Brent Meeker' via Everything List 
> wrote:


That the Born rule doesn't derive from the Schroedinger equation 
doesn't bother me.  Gleason's theorem guarantees it's the only 
consistent probability measure in the eigenstates of an observable.  
The question seems to be why is there probability at all.  But I see 
the unpredictability of measured values and that intrinsic randomness 
is necessary to protect relativity theory.  So randomness per the 
Born rule seems to be the obvious theoretical choice.


Yes, both inferred from observation, and derived from Mechanism.



But then why would I entertain the multiverse.


Because the many computations is a fact (cf 1930s).
We can’t delete them from the arithmetical reality, like we can’t 
delete the prime numbers.


Fine.  Leave them in arithmetic.







If I know when I've made a measurement then I know when the wave 
funcion "collapsed”.


Assuming that you don’t obey to quantum mechanics. But if QM applies 
to all physical objects, then it is simpler to use the first person 
indeterminacy on the superposition. Of course, that requires also to 
derive the wave from the mathematics of self-reference, which is done 
at the propositional level already.


But what does "self-reference" mean if there is no unique self.  You are 
fond of pointing to consciousness as indubitable, but it is equally 
indubitable that it is unitary.






  If I don't know when I've made a measurement, then I have more 
serious problems than just the interpretation of QM.


That is the problem with the collapse: you need to define what is a 
measurement, and how it succeeds in NOT being described by the wave 
function.


A measurement is what is perceived.  The "dualism" is just that some 
events happen and some don't.


Brent

It requires some ontological dualism no more coherent with any causal 
theory of the mind (not just mechanism BTW, although mechanism makes 
the task of showing this much easier).


Bruno




Brent

On 12/24/2020 1:04 PM, Stathis Papaioannou wrote:



On Thu, 24 Dec 2020 at 11:51, Bruce Kellett > wrote:


On Thu, Dec 24, 2020 at 5:39 AM Stathis Papaioannou
mailto:stath...@gmail.com>> wrote:


You suspected right, I am asking a more basic question about
self-sampling and the validity of probabilities when a
version of the observer sees all possible outcomes.


There is a problem here -- or maybe it is just careless
phrasing. You say there is a question about probabilities when a
version of the observer sees all possible outcomes. The question
is whether it is merely a version of the observer, a copy of the
observer, or the actual observer who sees all possible outcomes?
A "version" is somewhat ambiguous. Different 'versions' of an
operating system, for example, differ in some way. Whereas the
duplicates under consideration here are, by hypothesis, all
identical copies of the original.

As John Clark is fond of pointing out, the trouble with
self-sampling from a set of identical duplicate persons is that
the personal pronoun 'you' loses its unique reference. All
copies have an equal claim to be identified as the original
'you', so there is a real sense in which 'you' see all outcomes,
with probability one. If you attempt to single out a particular
individual by some random sampling procedure, you immediately
make a dualist assumption -- the selected individual is
different from the rest (by virtue of a 'soul', or some such,
conferred by the sampling process itself).

Since there is a sense in which 'you' certainly see all possible
outcomes, there is an immediate conflict with the Born rule,
according to which different outcomes have different
probabilities, and 'you' can't see more than one such outcome.


There is no magical “you” persisting from moment to moment even in 
ordinary life. I now have memories of being someone yesterday, I 
feel that the essence of that person has  been transmitted to me 
now. But it’s a delusion, and if I were copied many times each of 
the copies would of course have the same delusion. They can’t help 
it, it’s the way human psychology works. So thinking about 
probabilities if I am copied amounts to this: how should I reason 
about those future copies who share the delusion that they are 
uniquely me, given that I have the delusion that I will become one 
and only one of those copies? You propose that I drop the delusion 
and then there is no reasoning to be done, no question of 
probability. But to be consistent, I should then drop the delusion 
in a single thread universe as well, and not be concerned about the 
outcomes fir the person tomorrow who thinks that he is me just 
because he has memories of being me.

--
Stathis Papaioannou
--
You received this 

Re: Irreducible randomness in QM

2020-12-28 Thread Alan Grayson


On Monday, December 28, 2020 at 11:47:04 AM UTC-7 Alan Grayson wrote:

> On Monday, December 28, 2020 at 9:17:23 AM UTC-7 Bruno Marchal wrote:
>
>> On 26 Dec 2020, at 16:41, Alan Grayson  wrote:
>>
>> Fact is there's no need to bring in pretentious big brains from Australia 
>> to show that Born's rule makes no sense in the context of the MWI. For 
>> simplicity, consider a spin experiment where the observer measures spin UP 
>> in THIS world, and another world (or branch) is created where spin DN is 
>> measured. If we do the experiment again, the MWI is AMBIGUOUS on which 
>> world (or branch) the spin is now measured. 
>>
>>
>> Why? 
>> Not at all. This is already well explained in Everett’s original long 
>> paper. In the branch where the observer has measured spin up, he knows that 
>> if he redo quickly the measurement, he will find the same result. The 
>> measurement only entangle the spin state with his memory. 
>>
>
> *It would be refreshing if you knew what this topic is about. Entanglement 
> has nothing to do with this. We're creating ENSEMBLES of measurements in 
> THIS world; repeatable trials of say spin along some axis.  For each TRIAL, 
> a new world or branch is created if you adopt the MWI. ONE measurement in 
> each of these NEW worlds or branches. No ensemble; hence no probabilities! 
> It's really simple, so please don't bring in your pet theory to obfuscate 
> the scenario. AG*
>

*Or you can look at it this way; when repeated trials of the same 
measurement are done in this world, different worlds come into existence 
for EACH trial if you accept the MWI. Sometimes Alice leaves for a pee, and 
Bob continues to create the ensemble. No ensembles are created in those 
other worlds. Hence, no probabilities, and Born's rule doesn't apply. AG *

>
>>
>>
>>
>> On one of the two worlds just created, or are another pair of worlds 
>> created, one of which measures spin UP and another spin DN?  Of course this 
>> leaves out all the BS on how these worlds are created, along with the 
>> spontaneous creation of all the observers and experimental set-ups to do 
>> the measuring. Better continue to mentally jerkoff in the assumption, for 
>> the sake of argument, that there's some serious interpretation of QM going 
>> on here. AG
>>
>>
>> It is simpler to reason in term of histories. That is mandatory with 
>> mechanism, but the work of Graham, Hartle, Griffith, Omnes (and even 
>> Feynman paths) suggest this is already true in physics. 
>>
>> The notion of “world” is a tricky notion. 
>>
>>
>> Now as for the problem presented by the title of this thread, if the 
>> NO-GO theorems correctly affirm that FTL signaling is impossible, what 
>> EXACTLY is the link between this conclusion and the claim the QM is 
>> IRREDUCIBLY RANDOM? And what does IRREDUCIBLY RANDOM mean? In trying to 
>> answer this question, let me impose one RULE; no jerking off permitted! AG
>>
>>
>>
>> Irreducibly random means that we cannot predict the particular result 
>> that we will obtained.
>>
>
> *So, based on HUP, we can't do that for a conjugate pair, like position 
> and momentum. Based solely on HUP are pairs of conjugate measurements 
> irreducibly random? If you want a snotty reply, ask the Guru from 
> Australia.  AG*
>
>>
>> Imagine that the randomness was reducible. Imagine that Bob and Alice 
>> have a perfectly correlated dice, when at a light year of distance between 
>> each other. When Alice measure 5, Bob measure 5, etc. (as seen by some 
>> third observer to gate a local simultaneity notion). Their result have to 
>> be irreducibly random, because if their well able to predict their outcome, 
>> without invoking a super-determinacy, if they could predict the result, 
>> they would be able to use their correlation (entanglement) tp transmit a 
>> message FTL. 
>> Note that in 1927 Einstein saw already that the collapse of the wave 
>> packet needs to be FTL, and Bohr, in his paper answering the EPR paper, 
>> admit that the collapse cannot be a physical process, but then get very 
>> fuzzy when attempting to describe what happen.
>> I leave you the details, as I have to go. 
>>
>> Bruno
>>
>>
>>
>>

-- 
You received this message because you are subscribed to the Google Groups 
"Everything List" group.
To unsubscribe from this group and stop receiving emails from it, send an email 
to everything-list+unsubscr...@googlegroups.com.
To view this discussion on the web visit 
https://groups.google.com/d/msgid/everything-list/94461e24-e9d6-4e51-856d-f09ccf79fef8n%40googlegroups.com.


Re: Irreducible randomness in QM

2020-12-28 Thread Alan Grayson

*Bruce; remember when Brent claimed that physics doesn't PROVE anything? 
That wasn't recently on this MB. But when you wrote that irreducible 
randomness GUARANTEES no FTL signaling, you affirmed the existence of a 
PROOF. That was at the heart of my question; the guarantee or PROOF. But 
because you're a snotty and presumptuous individual, you couldn't see the 
depth underlying my question, and how your claim contradicts Brent's 
confident assertion about what physics does and doesn't do. That is, If you 
were a fair minded gentleman -- which is obviously not the case -- you 
would have simply referred me to the no-signaling theorems, and then I 
might be able to determine the status of your claim. You once asked me if I 
regarded you as my teacher. Again, the answer the NO. You don't have the 
emotional requirements to be anyone's teacher. AG*
On Friday, December 25, 2020 at 11:03:51 PM UTC-7 Alan Grayson wrote:

>
> *You could have omitted your snotty second sentence. Why is it OK for you 
> to use the List to mentally jerk off about the applicability of Born's rule 
> in the context of an interpretation you know is false, the MWI, than not 
> for me to ask a question which might have a simple answer, or path to take 
> to answer MY question? Being well informed about physics doesn't mean 
> you're not behaving like an arrogant schmuck. AG*
> On Thursday, December 24, 2020 at 11:49:42 PM UTC-7 Bruce wrote:
>
>> On Fri, Dec 25, 2020 at 5:29 PM Alan Grayson  wrote:
>>
>>>
>>> *Now I raise a similar question I posed to Bruce, thrice, with no 
>>> replies. Why does the unpredictability of measured values and the intrinsic 
>>> randomness protect relativity theory? This is really a huge conceptual 
>>> leap. How would you argue for that conclusion, as distinguished from 
>>> asserting it? TIA, AG*
>>
>>
>> You need to do some research on the no-signalling theorems. This list 
>> does not exist to answer your elementary questions.
>>
>> Bruce
>>
>

-- 
You received this message because you are subscribed to the Google Groups 
"Everything List" group.
To unsubscribe from this group and stop receiving emails from it, send an email 
to everything-list+unsubscr...@googlegroups.com.
To view this discussion on the web visit 
https://groups.google.com/d/msgid/everything-list/af49d43d-0589-43cc-8d30-5f676cf9e806n%40googlegroups.com.


Re: Irreducible randomness in QM

2020-12-28 Thread Alan Grayson


On Monday, December 28, 2020 at 9:17:23 AM UTC-7 Bruno Marchal wrote:

> On 26 Dec 2020, at 16:41, Alan Grayson  wrote:
>
> Fact is there's no need to bring in pretentious big brains from Australia 
> to show that Born's rule makes no sense in the context of the MWI. For 
> simplicity, consider a spin experiment where the observer measures spin UP 
> in THIS world, and another world (or branch) is created where spin DN is 
> measured. If we do the experiment again, the MWI is AMBIGUOUS on which 
> world (or branch) the spin is now measured. 
>
>
> Why? 
> Not at all. This is already well explained in Everett’s original long 
> paper. In the branch where the observer has measured spin up, he knows that 
> if he redo quickly the measurement, he will find the same result. The 
> measurement only entangle the spin state with his memory. 
>

*It would be refreshing if you knew what this topic is about. Entanglement 
has nothing to do with this. We're creating ENSEMBLES of measurements in 
THIS world; repeatable trials of say spin along some axis.  For each TRIAL, 
a new world or branch is created if you adopt the MWI. ONE measurement in 
each of these NEW worlds or branches. No ensemble; hence no probabilities! 
It's really simple, so please don't bring in your pet theory to obfuscate 
the scenario. AG*

>
>
>
>
> On one of the two worlds just created, or are another pair of worlds 
> created, one of which measures spin UP and another spin DN?  Of course this 
> leaves out all the BS on how these worlds are created, along with the 
> spontaneous creation of all the observers and experimental set-ups to do 
> the measuring. Better continue to mentally jerkoff in the assumption, for 
> the sake of argument, that there's some serious interpretation of QM going 
> on here. AG
>
>
> It is simpler to reason in term of histories. That is mandatory with 
> mechanism, but the work of Graham, Hartle, Griffith, Omnes (and even 
> Feynman paths) suggest this is already true in physics. 
>
> The notion of “world” is a tricky notion. 
>
>
> Now as for the problem presented by the title of this thread, if the NO-GO 
> theorems correctly affirm that FTL signaling is impossible, what EXACTLY is 
> the link between this conclusion and the claim the QM is IRREDUCIBLY 
> RANDOM? And what does IRREDUCIBLY RANDOM mean? In trying to answer this 
> question, let me impose one RULE; no jerking off permitted! AG
>
>
>
> Irreducibly random means that we cannot predict the particular result that 
> we will obtained.
>

*So, based on HUP, we can't do that for a conjugate pair, like position and 
momentum. Based solely on HUP are pairs of conjugate measurements 
irreducibly random? If you want a snotty reply, ask the Guru from 
Australia.  AG*

>
> Imagine that the randomness was reducible. Imagine that Bob and Alice have 
> a perfectly correlated dice, when at a light year of distance between each 
> other. When Alice measure 5, Bob measure 5, etc. (as seen by some third 
> observer to gate a local simultaneity notion). Their result have to be 
> irreducibly random, because if their well able to predict their outcome, 
> without invoking a super-determinacy, if they could predict the result, 
> they would be able to use their correlation (entanglement) tp transmit a 
> message FTL. 
> Note that in 1927 Einstein saw already that the collapse of the wave 
> packet needs to be FTL, and Bohr, in his paper answering the EPR paper, 
> admit that the collapse cannot be a physical process, but then get very 
> fuzzy when attempting to describe what happen.
> I leave you the details, as I have to go. 
>
> Bruno
>
>
>
>

-- 
You received this message because you are subscribed to the Google Groups 
"Everything List" group.
To unsubscribe from this group and stop receiving emails from it, send an email 
to everything-list+unsubscr...@googlegroups.com.
To view this discussion on the web visit 
https://groups.google.com/d/msgid/everything-list/53a1651c-b9ee-48e6-8cce-612998ee9f07n%40googlegroups.com.


Re: Irreducible randomness in QM

2020-12-28 Thread Bruno Marchal

> On 26 Dec 2020, at 16:41, Alan Grayson  wrote:
> 
> Fact is there's no need to bring in pretentious big brains from Australia to 
> show that Born's rule makes no sense in the context of the MWI. For 
> simplicity, consider a spin experiment where the observer measures spin UP in 
> THIS world, and another world (or branch) is created where spin DN is 
> measured. If we do the experiment again, the MWI is AMBIGUOUS on which world 
> (or branch) the spin is now measured.

Why? 
Not at all. This is already well explained in Everett’s original long paper. In 
the branch where the observer has measured spin up, he knows that if he redo 
quickly the measurement, he will find the same result. The measurement only 
entangle the spin state with his memory. 




> On one of the two worlds just created, or are another pair of worlds created, 
> one of which measures spin UP and another spin DN?  Of course this leaves out 
> all the BS on how these worlds are created, along with the spontaneous 
> creation of all the observers and experimental set-ups to do the measuring. 
> Better continue to mentally jerkoff in the assumption, for the sake of 
> argument, that there's some serious interpretation of QM going on here. AG

It is simpler to reason in term of histories. That is mandatory with mechanism, 
but the work of Graham, Hartle, Griffith, Omnes (and even Feynman paths) 
suggest this is already true in physics. 

The notion of “world” is a tricky notion. 

> 
> Now as for the problem presented by the title of this thread, if the NO-GO 
> theorems correctly affirm that FTL signaling is impossible, what EXACTLY is 
> the link between this conclusion and the claim the QM is IRREDUCIBLY RANDOM? 
> And what does IRREDUCIBLY RANDOM mean? In trying to answer this question, let 
> me impose one RULE; no jerking off permitted! AG


Irreducibly random means that we cannot predict the particular result that we 
will obtained.

Imagine that the randomness was reducible. Imagine that Bob and Alice have a 
perfectly correlated dice, when at a light year of distance between each other. 
When Alice measure 5, Bob measure 5, etc. (as seen by some third observer to 
gate a local simultaneity notion). Their result have to be irreducibly random, 
because if their well able to predict their outcome, without invoking a 
super-determinacy, if they could predict the result, they would be able to use 
their correlation (entanglement) tp transmit a message FTL. 
Note that in 1927 Einstein saw already that the collapse of the wave packet 
needs to be FTL, and Bohr, in his paper answering the EPR paper, admit that the 
collapse cannot be a physical process, but then get very fuzzy when attempting 
to describe what happen.
I leave you the details, as I have to go. 

Bruno


> 
> On Friday, December 25, 2020 at 11:03:51 PM UTC-7 Alan Grayson wrote:
> You could have omitted your snotty second sentence. Why is it OK for you to 
> use the List to mentally jerk off about the applicability of Born's rule in 
> the context of an interpretation you know is false, the MWI, than not for me 
> to ask a question which might have a simple answer, or path to take to answer 
> MY question? Being well informed about physics doesn't mean you're not 
> behaving like an arrogant schmuck. AG
> 
> On Thursday, December 24, 2020 at 11:49:42 PM UTC-7 Bruce wrote:
> On Fri, Dec 25, 2020 at 5:29 PM Alan Grayson > wrote:
> Now I raise a similar question I posed to Bruce, thrice, with no replies. Why 
> does the unpredictability of measured values and the intrinsic randomness 
> protect relativity theory? This is really a huge conceptual leap. How would 
> you argue for that conclusion, as distinguished from asserting it? TIA, AG
> 
> You need to do some research on the no-signalling theorems. This list does 
> not exist to answer your elementary questions.
> 
> Bruce
> 
> -- 
> You received this message because you are subscribed to the Google Groups 
> "Everything List" group.
> To unsubscribe from this group and stop receiving emails from it, send an 
> email to everything-list+unsubscr...@googlegroups.com 
> .
> To view this discussion on the web visit 
> https://groups.google.com/d/msgid/everything-list/7faa292d-cd04-4c7f-9f8c-6d5f27c53c1dn%40googlegroups.com
>  
> .

-- 
You received this message because you are subscribed to the Google Groups 
"Everything List" group.
To unsubscribe from this group and stop receiving emails from it, send an email 
to everything-list+unsubscr...@googlegroups.com.
To view this discussion on the web visit 
https://groups.google.com/d/msgid/everything-list/3CF797F8-0BEA-4DC1-849B-99D251301EE0%40ulb.ac.be.


Re: Irreducible randomness in QM

2020-12-28 Thread Bruno Marchal

> On 26 Dec 2020, at 16:39, 'scerir' via Everything List 
>  wrote:
> 
> Bruno writes: "It is a theorem. No universal machine can determine which 
> computations run it, .. "
> 
> Do we need a 'constructor theory’?
> 

The universal machine is enough, then you can enrich its cognitive abilities by 
allowing her to use induction principles, like PA, which makes them 
automatically “reflexive” or “Löbian”, I mean obeying to the G/G*/S4Grz/X/Z 
mathematics.



> That is to say a set of 'principles' under which we could show whether or not 
> a 'universal constructor' can exist?
> 
> 

The existence of both the universal machine (and of the constructors) are 
theorem of elementary arithmetic. You need only to read Gödel’s original paper 
where that has been proved the first time, albeit implicitly as Gödel missed 
the Church-Turing thesis, but Church, Turing and others have made that precise 
during the same decade (1930s).

Bruno




> https://www.edge.org/conversation/constructor-theory            
>  
> 
> https://arxiv.org/abs/1210.7439 
> 
> 
> -- 
> You received this message because you are subscribed to the Google Groups 
> "Everything List" group.
> To unsubscribe from this group and stop receiving emails from it, send an 
> email to everything-list+unsubscr...@googlegroups.com 
> .
> To view this discussion on the web visit 
> https://groups.google.com/d/msgid/everything-list/1274978825.510110.1608997167952%40mail1.libero.it
>  
> .

-- 
You received this message because you are subscribed to the Google Groups 
"Everything List" group.
To unsubscribe from this group and stop receiving emails from it, send an email 
to everything-list+unsubscr...@googlegroups.com.
To view this discussion on the web visit 
https://groups.google.com/d/msgid/everything-list/574CE7D8-4A57-44C8-B6E7-570473925464%40ulb.ac.be.


Re: Irreducible randomness in QM

2020-12-28 Thread Bruno Marchal

> On 25 Dec 2020, at 20:35, John Clark  wrote:
> 
> 
> On Thu, Dec 24, 2020 at 10:55 AM  > wrote:
> 
> > There are three possible ways out this problem of course. One is to say 
> > these string must be finite and thus the set of them enumerable. The other 
> > is to abandon the axiom of choice which permits the ordering I mention 
> > above. The third would be to abandon MWI. This looks to be an informal 
> > demonstration that MWI + QM(with Born rule) is not consistent with there 
> > being an infinite set of possible outcomes. 
> 
> There may be a fourth way. I think you're assuming that in Many Worlds only 
> one observer sees one precise quantum event, but when you observe an electron 
> do one particular thing there may be an infinite number of other Lawrence 
> Crowells that live in a universe where the electron does something very 
> slightly different, but the difference is so Infinitesimally tiny that no 
> conscious observer could possibly tell the difference, so there is no 
> difference between those infinite number of conscious beings that call 
> themselves Lawrence Crowell either.
> 
> Also, it seems that when the axiom of choice is allowed you can 
> mathematically prove all sorts of things that seem very unphysical, like the 
> Banach-Tarski paradox and many others. It makes life easier for 
> mathematicians who like to prove things but can you think of an example of a 
> physicist, and not a pure mathematician, that found the axiom of choice to be 
> a useful tool in predicting the outcome of an experiment?

You need it to prove that all linear space, or the Hilbert space, have a base, 
which is used in theoretical physics. Of course you can argue that this is due 
to the use of real number, and Cauchy sequence of vectors, which might just be 
“simplifying tools”.




> Mathematics is the language of physics but like any language it can be used 
> to write fiction as well as nonfiction, physics is only concerned with the 
> nonfiction part.


If mathematics was just a language, we would not have the non-algorithmic 
distribution of the halting and non halting machines in arithmetic. 
Then wth Mechanism, physics becomes more like the language that the universal 
machine invent to be able to share their histories.

You do seem to believe in a physical universe which would be ontologically 
primary. That’s OK, but fails when you assume Mechanism. No universal machine 
can feel the difference between being emulated in arithmetic or in physics, 
despite being able to test the difference in the long run. The test is the 
quantum. As the quantum is a consequence of mechanism, you need some experience 
violating quantum mechanics, (at least the version derived from the statistic 
on all computations).


Bruno



> 
> John K Clark
> 
> -- 
> You received this message because you are subscribed to the Google Groups 
> "Everything List" group.
> To unsubscribe from this group and stop receiving emails from it, send an 
> email to everything-list+unsubscr...@googlegroups.com 
> .
> To view this discussion on the web visit 
> https://groups.google.com/d/msgid/everything-list/CAJPayv1OSLSy1kDy4enym5oLDLn9U6UT5mPXkYdJLosHX93pcw%40mail.gmail.com
>  
> .

-- 
You received this message because you are subscribed to the Google Groups 
"Everything List" group.
To unsubscribe from this group and stop receiving emails from it, send an email 
to everything-list+unsubscr...@googlegroups.com.
To view this discussion on the web visit 
https://groups.google.com/d/msgid/everything-list/32DD04B4-CB0F-4460-B3EA-1D4D8BA9E1B0%40ulb.ac.be.


Re: Irreducible randomness in QM

2020-12-28 Thread Bruno Marchal

> On 25 Dec 2020, at 14:23, John Clark  wrote:
> 
> On Thu, Dec 24, 2020 at 11:37 PM 'Brent Meeker' via Everything List 
> mailto:everything-list@googlegroups.com>> 
> wrote:
> 
> >That the Born rule doesn't derive from the Schroedinger equation doesn't 
> >bother me. 
> 
> We don't need to derive the Born Rule from the Schroedinger equation or from 
> anything else to know it's true because we already know from experimentation 
> that it's true. But Copenhagen, Many Worlds, Pilot Wave and every other 
> quantum interpretation needs to derive the Born Rule to prove that it has the 
> right interpretation. If Many Worlds hasn't managed to achieve that high goal 
> it has at least come closer to doing so than any other interpretation, at 
> least so far.
> 
> > Gleason's theorem guarantees it's the only consistent probability measure 
> > in the eigenstates of an observable.  The question seems to be why is there 
> > probability at all. 
> 
> Yes!
>  
> > But I see the unpredictability of measured values and that intrinsic 
> > randomness is necessary to protect relativity theory. 
>  
> I see it is simply a result of the fact that you can't be sure where you are 
> until you open your eyes and look, aka make an observation. If I use Bruno's 
> patented You Duplicating Machine then even after Brent Meeker has been 
> duplicated "You" will not know if "you" are in Moscow or Washington until 
> "you" open the door of the duplicating chamber and look out; until that point 
> "you" could be said to exist in both cities, or in neither city, or in 
> whatever place you happen to be thinking about because contrary to what 
> generations have been taught in grammar school "you" Is not a pronoun, "you"  
> is an adjective; you are the way matter behaves when it is organized in a 
> Brentmeekerian way. And until the instant the doors are open and the 2 see 2 
> different things those 2 chunks of matter have the same memories and behave 
> exactly the same way.

If you are the way matter is organised, and if that organisation is Turing 
emulable, you are in all infinitely many number relations which emulates you at 
that level of description in the arithmetical reality (the so-called standard 
model of the natural numbers).

That is nice, because this promise (at least) to explain where the laws of 
physics come from, and, as a matter of fact, it will explain also why it takes 
the shape of qualia, and why some qualia are sharable (the quanta).

Bruno



> 
> John K Clark
> 
> -- 
> You received this message because you are subscribed to the Google Groups 
> "Everything List" group.
> To unsubscribe from this group and stop receiving emails from it, send an 
> email to everything-list+unsubscr...@googlegroups.com 
> .
> To view this discussion on the web visit 
> https://groups.google.com/d/msgid/everything-list/CAJPayv1RjUYHu5qh0sNNfMKDu48koL5mGf9rCkfWCqf8txu%3Daw%40mail.gmail.com
>  
> .

-- 
You received this message because you are subscribed to the Google Groups 
"Everything List" group.
To unsubscribe from this group and stop receiving emails from it, send an email 
to everything-list+unsubscr...@googlegroups.com.
To view this discussion on the web visit 
https://groups.google.com/d/msgid/everything-list/DC5059F4-8847-48EA-A7C4-DDDE30E47B27%40ulb.ac.be.


Re: Irreducible randomness in QM

2020-12-28 Thread Bruno Marchal

> On 25 Dec 2020, at 12:17, Bruce Kellett  wrote:
> 
> On Fri, Dec 25, 2020 at 9:41 PM Stathis Papaioannou  > wrote:
> On Fri, 25 Dec 2020 at 21:32, Bruce Kellett  > wrote:
> On Fri, Dec 25, 2020 at 6:27 PM Stathis Papaioannou  > wrote:
> On Fri, 25 Dec 2020 at 12:29, Bruce Kellett  > wrote:
> On Fri, Dec 25, 2020 at 12:13 PM Stathis Papaioannou  > wrote:
> 
> One is the probability that a certain branch exists, the other is the 
> subjective probability that a being with the feeling that he is a unique 
> individual persisting through time will experience a particular branch.
> 
> 
> According to the individual on the branch, the Born probability is the 
> probability that that branch will exist -- it is an objective property of the 
> branch. It is a subjective probability only to the extent to which the 
> individual believes in Lewis's Principal Principle!
> 
> The probability that the branch exists under MWI, as you have rightly pointed 
> out, is 1. The probability that an entity that can randomly land in any 
> branch lands in one particular branch is given by the Born rule.
> 
> In other words, you have a dualist interpretation of personhood. There is an 
> exact copy of 'you' on every branch. 'You' do not randomly land on any branch 
> unless there is a unique 'you' specified in some dualst manner.
> 
> Like everyone, I feel that I am a unique individual persisting through time, 
> which does not cause conceptual problems if there is only one extant version 
> of me at a time, but does if there are multiple versions. I know that this 
> feeling I have is just a contingent fact about human psychology. If I were a 
> dualist, I would believe that it was some sort of metaphysical truth.
> 
> It can be a metaphysical truth without there being any dualist underpinnings.

Stathis seems to use the Mechanist Assumption, which is a metaphysical, or 
psychological, or theological hypothesis. 



> The problem, as you point out, is when there are multiple copies of you 
> extant at a single time. If you consider yourself to be a random selection 
> from this reference class, then you have made the dualist assumption that 
> there is something that picks you out -- something that distinguishes you 
> from all the other copies.

Yes, there is something which select you, but it is just the combination of 
your memories, and the result of your self-localisation or other measurement, 
like you remember that the cat was alive, and now you see it either alive or 
dead, even if you gpot a doppelgänger who see the alternative.



> Whereas, in reality, all the copies are the same and must think the same: 
> they can deduce that they are not special, and that the probability for the 
> existence of each copy is exactly one


Trivially in the 0th person description, but the probabilities concerns the 
subjective experience, and once they differ, you can only see one of them.



> -- the Born probabilities have no bearing on their existence because they 
> inevitably exist regardless of the magnitude of the mod-squared quantum 
> amplitude. There is no process that selects just one individual copy at 
> random from a distribution, whether it be the uniform distribution over 
> branches, or the probability distribution obtained by Born weighting each 
> branch.

But then there is no collapse, and the apperance of the collapse is well 
explained by Mechanism, up to the remaining task of deriving the wave from the 
mathematics of self-reference. There we get an unexpected gift: we get two 
mathematics: one for the quanta (which indeed looks already like quantum 
mechanics) and one for the qualia (which is more general and explains the 
private incommunicable aspect of the subject).

Bruno




> 
> Bruce
> 
> -- 
> You received this message because you are subscribed to the Google Groups 
> "Everything List" group.
> To unsubscribe from this group and stop receiving emails from it, send an 
> email to everything-list+unsubscr...@googlegroups.com 
> .
> To view this discussion on the web visit 
> https://groups.google.com/d/msgid/everything-list/CAFxXSLTjyhp0RnK-44jVvwKEeqpwhbSqg3mqBLCiS4pHUCGUQw%40mail.gmail.com
>  
> .

-- 
You received this message because you are subscribed to the Google Groups 
"Everything List" group.
To unsubscribe from this group and stop receiving emails from it, send an email 
to everything-list+unsubscr...@googlegroups.com.
To view this discussion on the web visit 
https://groups.google.com/d/msgid/everything-list/A93B0DDE-0FF6-45EA-BD2B-3CABDFCBC096%40ulb.ac.be.


Re: Irreducible randomness in QM

2020-12-28 Thread Bruno Marchal

> On 25 Dec 2020, at 11:32, Bruce Kellett  wrote:
> 
> On Fri, Dec 25, 2020 at 6:27 PM Stathis Papaioannou  > wrote:
> On Fri, 25 Dec 2020 at 12:29, Bruce Kellett  > wrote:
> On Fri, Dec 25, 2020 at 12:13 PM Stathis Papaioannou  > wrote:
> 
> One is the probability that a certain branch exists, the other is the 
> subjective probability that a being with the feeling that he is a unique 
> individual persisting through time will experience a particular branch.
> 
> 
> According to the individual on the branch, the Born probability is the 
> probability that that branch will exist -- it is an objective property of the 
> branch. It is a subjective probability only to the extent to which the 
> individual believes in Lewis's Principal Principle!
> 
> The probability that the branch exists under MWI, as you have rightly pointed 
> out, is 1. The probability that an entity that can randomly land in any 
> branch lands in one particular branch is given by the Born rule.
> 
> In other words, you have a dualist interpretation of personhood.

That is correct, but with the MWI (or with Mechanism) the duality is 
phenomenological, and not ontological, and is no more than the difference 
between a brain and a conscious sensation. One is describable in arithmetic 
(indeed by the corresponding predicate of Gödel  []p (which obeys to G* and G), 
and the other is described by ([]p & p), which is not definable in arithmetic, 
nor in any effective theories which extends arithmetic. It is metadefinable by 
richer machine when talking on simpler machine, but is not definable by any 
machine about itself. That plays a key role to associate consciousness to a 
computation, and derive the existence of the universal wave from there.




> There is an exact copy of 'you' on every branch. 'You' do not randomly land 
> on any branch unless there is a unique 'you' specified in some dualst manner.

The “you” is not unique here, but all “your” feel unique, and feel the first 
person indeterminacy, be it with quantum superposition or physical 
duplications, or arithmetical differentiation computations when multiplied on 
different inputs possible.

Bruno



> 
> Bruce
> 
> -- 
> You received this message because you are subscribed to the Google Groups 
> "Everything List" group.
> To unsubscribe from this group and stop receiving emails from it, send an 
> email to everything-list+unsubscr...@googlegroups.com 
> .
> To view this discussion on the web visit 
> https://groups.google.com/d/msgid/everything-list/CAFxXSLT%2BG7h%2BsyKPc_Kg9EwWRd7f4E7jNcube4NmJC7JLuZMyw%40mail.gmail.com
>  
> .

-- 
You received this message because you are subscribed to the Google Groups 
"Everything List" group.
To unsubscribe from this group and stop receiving emails from it, send an email 
to everything-list+unsubscr...@googlegroups.com.
To view this discussion on the web visit 
https://groups.google.com/d/msgid/everything-list/C9878C03-48E8-449C-90C4-F03478D1F371%40ulb.ac.be.


Re: Irreducible randomness in QM

2020-12-28 Thread Bruno Marchal

> On 25 Dec 2020, at 06:44, Bruce Kellett  wrote:
> 
> On Fri, Dec 25, 2020 at 3:37 PM 'Brent Meeker' via Everything List 
> mailto:everything-list@googlegroups.com>> 
> wrote:
> That the Born rule doesn't derive from the Schroedinger equation doesn't 
> bother me.
> 
> It doesn't bother me, either. If one needs probabilities, one can simply 
> impose the Born rule. The problem is that you can't do this consistently when 
> every outcome occurs on every trial -- there is no sensible definition of 
> probability in that case.
> 
> Gleason's theorem guarantees it's the only consistent probability measure in 
> the eigenstates of an observable.  The question seems to be why is there 
> probability at all.
> 
> That lies at the heart of the problem. You can't get probabilities out of a 
> deterministic theory.

Assuming Mechanism, the contrary happens. The objective determinacy enforces 
the personal relative subjective indeterminacy. 

Bruno







> 
> Bruce
> 
> But I see the unpredictability of measured values and that intrinsic 
> randomness is necessary to protect relativity theory.  So randomness per the 
> Born rule seems to be the obvious theoretical choice.  But then why would I 
> entertain the multiverse.  If I know when I've made a measurement then I know 
> when the wave funcion "collapsed".  If I don't know when I've made a 
> measurement, then I have more serious problems than just the interpretation 
> of QM.
> 
> Brent
> 
> -- 
> You received this message because you are subscribed to the Google Groups 
> "Everything List" group.
> To unsubscribe from this group and stop receiving emails from it, send an 
> email to everything-list+unsubscr...@googlegroups.com 
> .
> To view this discussion on the web visit 
> https://groups.google.com/d/msgid/everything-list/CAFxXSLS_Pi%2Bn2rNKErLtZ7wWTdFmeuyDgJD7ekvXBMDkYDWpog%40mail.gmail.com
>  
> .

-- 
You received this message because you are subscribed to the Google Groups 
"Everything List" group.
To unsubscribe from this group and stop receiving emails from it, send an email 
to everything-list+unsubscr...@googlegroups.com.
To view this discussion on the web visit 
https://groups.google.com/d/msgid/everything-list/7E2B033E-050D-4990-94CE-545DCBD44E8E%40ulb.ac.be.


Re: Irreducible randomness in QM

2020-12-28 Thread Bruno Marchal

> On 25 Dec 2020, at 05:37, 'Brent Meeker' via Everything List 
>  wrote:
> 
> That the Born rule doesn't derive from the Schroedinger equation doesn't 
> bother me.  Gleason's theorem guarantees it's the only consistent probability 
> measure in the eigenstates of an observable.  The question seems to be why is 
> there probability at all.  But I see the unpredictability of measured values 
> and that intrinsic randomness is necessary to protect relativity theory.  So 
> randomness per the Born rule seems to be the obvious theoretical choice. 

Yes, both inferred from observation, and derived from Mechanism.


> But then why would I entertain the multiverse. 

Because the many computations is a fact (cf 1930s).
We can’t delete them from the arithmetical reality, like we can’t delete the 
prime numbers.





> If I know when I've made a measurement then I know when the wave funcion 
> "collapsed”.

Assuming that you don’t obey to quantum mechanics. But if QM applies to all 
physical objects, then it is simpler to use the first person indeterminacy on 
the superposition. Of course, that requires also to derive the wave from the 
mathematics of self-reference, which is done at the propositional level already.



>   If I don't know when I've made a measurement, then I have more serious 
> problems than just the interpretation of QM.

That is the problem with the collapse: you need to define what is a 
measurement, and how it succeeds in NOT being described by the wave function. 
It requires some ontological dualism no more coherent with any causal theory of 
the mind (not just mechanism BTW, although mechanism makes the task of showing 
this much easier).

Bruno


> 
> Brent
> 
> On 12/24/2020 1:04 PM, Stathis Papaioannou wrote:
>> 
>> 
>> On Thu, 24 Dec 2020 at 11:51, Bruce Kellett > > wrote:
>> On Thu, Dec 24, 2020 at 5:39 AM Stathis Papaioannou > > wrote:
>> 
>> You suspected right, I am asking a more basic question about self-sampling 
>> and the validity of probabilities when a version of the observer sees all 
>> possible outcomes.
>> 
>> There is a problem here -- or maybe it is just careless phrasing. You say 
>> there is a question about probabilities when a version of the observer sees 
>> all possible outcomes. The question is whether it is merely a version of the 
>> observer, a copy of the observer, or the actual observer who sees all 
>> possible outcomes? A "version" is somewhat ambiguous. Different 'versions' 
>> of an operating system, for example, differ in some way. Whereas the 
>> duplicates under consideration here are, by hypothesis, all identical copies 
>> of the original.
>> 
>> As John Clark is fond of pointing out, the trouble with self-sampling from a 
>> set of identical duplicate persons is that the personal pronoun 'you' loses 
>> its unique reference. All copies have an equal claim to be identified as the 
>> original 'you', so there is a real sense in which 'you' see all outcomes, 
>> with probability one. If you attempt to single out a particular individual 
>> by some random sampling procedure, you immediately make a dualist assumption 
>> -- the selected individual is different from the rest (by virtue of a 
>> 'soul', or some such, conferred by the sampling process itself).
>> 
>> Since there is a sense in which 'you' certainly see all possible outcomes, 
>> there is an immediate conflict with the Born rule, according to which 
>> different outcomes have different probabilities, and 'you' can't see more 
>> than one such outcome.
>> 
>> There is no magical “you” persisting from moment to moment even in ordinary 
>> life. I now have memories of being someone yesterday, I feel that the 
>> essence of that person has  been transmitted to me now. But it’s a delusion, 
>> and if I were copied many times each of the copies would of course have the 
>> same delusion. They can’t help it, it’s the way human psychology works. So 
>> thinking about probabilities if I am copied amounts to this: how should I 
>> reason about those future copies who share the delusion that they are 
>> uniquely me, given that I have the delusion that I will become one and only 
>> one of those copies? You propose that I drop the delusion and then there is 
>> no reasoning to be done, no question of probability. But to be consistent, I 
>> should then drop the delusion in a single thread universe as well, and not 
>> be concerned about the outcomes fir the person tomorrow who thinks that he 
>> is me just because he has memories of being me.
>> -- 
>> Stathis Papaioannou
>> -- 
>> You received this message because you are subscribed to the Google Groups 
>> "Everything List" group.
>> To unsubscribe from this group and stop receiving emails from it, send an 
>> email to everything-list+unsubscr...@googlegroups.com 
>> .
>> To view this discussion on the web visit 
>> 

Re: Irreducible randomness in QM

2020-12-28 Thread Bruno Marchal

> On 25 Dec 2020, at 03:06, spudboy100 via Everything List 
>  wrote:
> 
> For "physical continuity" information as a stored coherent process, may be 
> provided by the study of cosmology in the interaction of photons and gravity. 
> It's a long shot, especially from the vantage point of our gloomy age, but it 
> may be baked-in to the cosmos. Some have advocated that quantum foam may 
> "scrub" the data away, and this may be true, but I'd like to have a physics 
> model for this action. 
> https://turingchurch.net/the-infrared-memory-of-the-universe-hints-at-future-akashic-physics-3f9a072f0ca6
> So, in this fashion storage provides continuity. 

If they use the original version of the Church-Turing thesis, they can’t 
invoke, even implicitly, a physical reality without making it into a magical 
sort of God, capable of selecting a computation among many, where mechanism 
explains the section by the usual recursive and indexical manner.

Bruno


> 
> 
> -Original Message-
> From: Bruce Kellett 
> To: Everything List 
> Sent: Thu, Dec 24, 2020 8:04 pm
> Subject: Re: Irreducible randomness in QM
> 
> On Fri, Dec 25, 2020 at 10:47 AM Stathis Papaioannou  <mailto:stath...@gmail.com>> wrote:
> On Fri, 25 Dec 2020 at 08:55, Bruce Kellett  <mailto:bhkellet...@gmail.com>> wrote:
> 
> It is not merely a psychological artefact -- it is a matter of physical 
> continuity. Your theory of personal identity is letting you down here.
> 
> Physical continuity is neither necessary nor sufficient for continuity of 
> personal identity.
> 
>  
> But personal identity cannot be sensibly defined without taking account of 
> physical continuity.
> 
>  
> Apply this psychological artefact where there is copying, and you get 
> probabilities.
> 
> I think you need a little more than this hand-waving in order to get 
> probabilities. They have a physical origin and are objective, after all.
> 
> There is an objective way to calculate probabilities, but when questions like 
> “what will I see tomorrow” are asked a theory of personal identity is 
> introduced, which is a contingent fact about our psychology.
> 
> 
> I think you are being seriously misled by your deficient understanding of 
> personal identity.
> 
> In the MWI of QM, the linearity of the Schrodinger equation means that the 
> whole person, physical and psychological, is duplicated into every branch of 
> the wave function -- with probability one. Even though the Born probability 
> for each branch is strictly less than one.
> 
> Bruce
> -- 
> You received this message because you are subscribed to the Google Groups 
> "Everything List" group.
> To unsubscribe from this group and stop receiving emails from it, send an 
> email to everything-list+unsubscr...@googlegroups.com 
> <mailto:everything-list+unsubscr...@googlegroups.com>.
> To view this discussion on the web visit
> https://groups.google.com/d/msgid/everything-list/CAFxXSLT%2BxOK1zwHNWXjbvj%3D4sqcbjFeecPPpc27i1K%2BsT9mVRA%40mail.gmail.com
>  
> <https://groups.google.com/d/msgid/everything-list/CAFxXSLT%2BxOK1zwHNWXjbvj%3D4sqcbjFeecPPpc27i1K%2BsT9mVRA%40mail.gmail.com?utm_medium=email_source=footer>
> .
> 
> -- 
> You received this message because you are subscribed to the Google Groups 
> "Everything List" group.
> To unsubscribe from this group and stop receiving emails from it, send an 
> email to everything-list+unsubscr...@googlegroups.com 
> <mailto:everything-list+unsubscr...@googlegroups.com>.
> To view this discussion on the web visit 
> https://groups.google.com/d/msgid/everything-list/190409769.3276460.1608861979423%40mail.yahoo.com
>  
> <https://groups.google.com/d/msgid/everything-list/190409769.3276460.1608861979423%40mail.yahoo.com?utm_medium=email_source=footer>.

-- 
You received this message because you are subscribed to the Google Groups 
"Everything List" group.
To unsubscribe from this group and stop receiving emails from it, send an email 
to everything-list+unsubscr...@googlegroups.com.
To view this discussion on the web visit 
https://groups.google.com/d/msgid/everything-list/E13E4F18-221C-4CF4-8718-F96AED21EA31%40ulb.ac.be.


Re: Irreducible randomness in QM

2020-12-28 Thread Bruno Marchal

> On 24 Dec 2020, at 22:55, Bruce Kellett  wrote:
> 
> On Fri, Dec 25, 2020 at 8:44 AM Stathis Papaioannou  > wrote:
> On Fri, 25 Dec 2020 at 08:33, Bruce Kellett  > wrote:
> On Fri, Dec 25, 2020 at 8:04 AM Stathis Papaioannou  > wrote:
> On Thu, 24 Dec 2020 at 11:51, Bruce Kellett  > wrote:
> On Thu, Dec 24, 2020 at 5:39 AM Stathis Papaioannou  > wrote:
> 
> You suspected right, I am asking a more basic question about self-sampling 
> and the validity of probabilities when a version of the observer sees all 
> possible outcomes.
> 
> There is a problem here -- or maybe it is just careless phrasing. You say 
> there is a question about probabilities when a version of the observer sees 
> all possible outcomes. The question is whether it is merely a version of the 
> observer, a copy of the observer, or the actual observer who sees all 
> possible outcomes? A "version" is somewhat ambiguous. Different 'versions' of 
> an operating system, for example, differ in some way. Whereas the duplicates 
> under consideration here are, by hypothesis, all identical copies of the 
> original.
> 
> As John Clark is fond of pointing out, the trouble with self-sampling from a 
> set of identical duplicate persons is that the personal pronoun 'you' loses 
> its unique reference. All copies have an equal claim to be identified as the 
> original 'you', so there is a real sense in which 'you' see all outcomes, 
> with probability one. If you attempt to single out a particular individual by 
> some random sampling procedure, you immediately make a dualist assumption -- 
> the selected individual is different from the rest (by virtue of a 'soul', or 
> some such, conferred by the sampling process itself).
> 
> Since there is a sense in which 'you' certainly see all possible outcomes, 
> there is an immediate conflict with the Born rule, according to which 
> different outcomes have different probabilities, and 'you' can't see more 
> than one such outcome.
> 
> There is no magical “you” persisting from moment to moment even in ordinary 
> life.
> 
> 
> It's not magical -- that is what it means to be a person-- you have physical 
> and psychological continuity from moment to moment. If you throw this away, 
> it is hard to know what you are talking about.
> 
> I’m not throwing it away, but I am acknowledging that it is a psychological 
> artefact, not a dualistic soul.
> 
> 
> It is not merely a psychological artefact -- it is a matter of physical 
> continuity. Your theory of personal identity is letting you down here.
> 
> 
> Apply this psychological artefact where there is copying, and you get 
> probabilities.
> 
> I think you need a little more than this hand-waving in order to get 
> probabilities. They have a physical origin and are objective, after all.

I can prove that this make no sense, or you need to abandon Mechanism (even the 
little amount required to attribute an explanative power to Darwin’s theory of 
evolution).

As a scientist working the fundamental science, I just cannot take the physical 
universe as being something primary. It is an objective illusion, common to all 
universal numbers.

It is up to the believer in physical universe which would be primary to give 
the evidences for that “primariness". It does not work and that is why 
materialist have to abandon Mechanism, or to deny consciousness and (first) 
person. 

Bruno


> 
> Bruce
> 
> 
> The copies each say “I know logically that all the copies are identical, but 
> I feel that I am the only real me, continuing from the original”.
> 
> I now have memories of being someone yesterday, I feel that the essence of 
> that person has  been transmitted to me now. But it’s a delusion, and if I 
> were copied many times each of the copies would of course have the same 
> delusion. They can’t help it, it’s the way human psychology works. So 
> thinking about probabilities if I am copied amounts to this: how should I 
> reason about those future copies who share the delusion that they are 
> uniquely me, given that I have the delusion that I will become one and only 
> one of those copies? You propose that I drop the delusion and then there is 
> no reasoning to be done, no question of probability. But to be consistent, I 
> should then drop the delusion in a single thread universe as well, and not be 
> concerned about the outcomes fir the person tomorrow who thinks that he is me 
> just because he has memories of being me.
> 
> -- 
> You received this message because you are subscribed to the Google Groups 
> "Everything List" group.
> To unsubscribe from this group and stop receiving emails from it, send an 
> email to everything-list+unsubscr...@googlegroups.com 
> .
> To view this discussion on the web visit 
> 

Re: Irreducible randomness in QM

2020-12-26 Thread Alan Grayson
Correction2 (for typos) on spin scenario: Suppose in THIS world an observer 
measures spin UP. Then, according to the MWI, in another NEWLY CREATED 
WORLD, the NEWLY CREATED OBSERVER measures spin DN. On the next measurement 
in THIS world, some spin in measured, say DN, and in ANOTHER CREATED WORLD 
or branch,  ANOTHER NEWLY CREATED observer measures spin UP,  and so. As a 
consequence, we get an ENSEMBLE of measurements in THIS world or branch, 
which is the NECESSARY CONDITION for a probability to exist, and a SET of 
other worlds or branches in which there is ONE measurement in each , only 
one, and thus no ensemble exists in those other worlds or branches. That 
is, since subsequent measurements in THIS world correspond to a NEW wfs, 
additional OTHER worlds or branches are created according to the MWI each 
time a NEW measurement is performed in THIS world. IOW, no ensembles in 
those other worlds; hence no probabilities, and Born's rule does NOT apply. 
This argument is all that's required to show that Born's rule does NOT 
exist in the MWI. No need for continuing pretentious arguments on the 
subject. QED! AG 
On Saturday, December 26, 2020 at 10:52:10 AM UTC-7 Alan Grayson wrote:

> Correction on spin scenario: Suppose in THIS world an observer measures 
> spin UP. According to the MWI. Then in another world or branch the NEWLY 
> CREATED measures spin DN. On the next measurement in THIS world, some spin 
> in measured, say DN, and in ANOTHER world or branch ANOTHER NEWLY CREATED 
> observer measures spin UP,  and so. As a consequence, we get an ENSEMBLE of 
> measurements in THIS world or branch, which is the NECESSARY CONDITION for 
> a probability to exist, and a set of other worlds or branches in which 
> there is ONE measurement, only one, and thus no ensemble exists in those 
> other worlds or branches. Since subsequent measurements in THIS world 
> correspond to a NEW wfs, additional OTHER worlds are created according to 
> the MWI each time a NEW measurement is performed in THIS world. IOW, no 
> ensembles in those other worlds, hence no probabilities and Born's rule 
> does NOT apply. That's basically all that's required to show that Born's 
> rule does NOT exist in the MWI. No need for continuing pretentious 
> arguments on the subject. QED! AG 
>
> On Saturday, December 26, 2020 at 8:41:17 AM UTC-7 Alan Grayson wrote:
>
>> Fact is there's no need to bring in pretentious big brains from Australia 
>> to show that Born's rule makes no sense in the context of the MWI. For 
>> simplicity, consider a spin experiment where the observer measures spin UP 
>> in THIS world, and another world (or branch) is created where spin DN is 
>> measured. If we do the experiment again, the MWI is AMBIGUOUS on which 
>> world (or branch) the spin is now measured. On one of the two worlds just 
>> created, or are another pair of worlds created, one of which measures spin 
>> UP and another spin DN?  Of course this leaves out all the BS on how these 
>> worlds are created, along with the spontaneous creation of all the 
>> observers and experimental set-ups to do the measuring. Better continue to 
>> mentally jerkoff in the assumption, for the sake of argument, that there's 
>> some serious interpretation of QM going on here. AG
>>
>> Now as for the problem presented by the title of this thread, if the 
>> NO-GO theorems correctly affirm that FTL signaling is impossible, what 
>> EXACTLY is the link between this conclusion and the claim the QM is 
>> IRREDUCIBLY RANDOM? And what does IRREDUCIBLY RANDOM mean? In trying to 
>> answer this question, let me impose one RULE; no jerking off permitted! AG
>>
>> On Friday, December 25, 2020 at 11:03:51 PM UTC-7 Alan Grayson wrote:
>>
>>>
>>> *You could have omitted your snotty second sentence. Why is it OK for 
>>> you to use the List to mentally jerk off about the applicability of Born's 
>>> rule in the context of an interpretation you know is false, the MWI, than 
>>> not for me to ask a question which might have a simple answer, or path to 
>>> take to answer MY question? Being well informed about physics doesn't mean 
>>> you're not behaving like an arrogant schmuck. AG*
>>> On Thursday, December 24, 2020 at 11:49:42 PM UTC-7 Bruce wrote:
>>>
 On Fri, Dec 25, 2020 at 5:29 PM Alan Grayson  
 wrote:

>
> *Now I raise a similar question I posed to Bruce, thrice, with no 
> replies. Why does the unpredictability of measured values and the 
> intrinsic 
> randomness protect relativity theory? This is really a huge conceptual 
> leap. How would you argue for that conclusion, as distinguished from 
> asserting it? TIA, AG*


 You need to do some research on the no-signalling theorems. This list 
 does not exist to answer your elementary questions.

 Bruce

>>>

-- 
You received this message because you are subscribed to the Google Groups 
"Everything List" group.
To unsubscribe from this group and 

Re: Irreducible randomness in QM

2020-12-26 Thread Alan Grayson
Correction on spin scenario: Suppose in THIS world an observer measures 
spin UP. According to the MWI. Then in another world or branch the NEWLY 
CREATED measures spin DN. On the next measurement in THIS world, some spin 
in measured, say DN, and in ANOTHER world or branch ANOTHER NEWLY CREATED 
observer measures spin UP,  and so. As a consequence, we get an ENSEMBLE of 
measurements in THIS world or branch, which is the NECESSARY CONDITION for 
a probability to exist, and a set of other worlds or branches in which 
there is ONE measurement, only one, and thus no ensemble exists in those 
other worlds or branches. Since subsequent measurements in THIS world 
correspond to a NEW wfs, additional OTHER worlds are created according to 
the MWI each time a NEW measurement is performed in THIS world. IOW, no 
ensembles in those other worlds, hence no probabilities and Born's rule 
does NOT apply. That's basically all that's required to show that Born's 
rule does NOT exist in the MWI. No need for continuing pretentious 
arguments on the subject. QED! AG 

On Saturday, December 26, 2020 at 8:41:17 AM UTC-7 Alan Grayson wrote:

> Fact is there's no need to bring in pretentious big brains from Australia 
> to show that Born's rule makes no sense in the context of the MWI. For 
> simplicity, consider a spin experiment where the observer measures spin UP 
> in THIS world, and another world (or branch) is created where spin DN is 
> measured. If we do the experiment again, the MWI is AMBIGUOUS on which 
> world (or branch) the spin is now measured. On one of the two worlds just 
> created, or are another pair of worlds created, one of which measures spin 
> UP and another spin DN?  Of course this leaves out all the BS on how these 
> worlds are created, along with the spontaneous creation of all the 
> observers and experimental set-ups to do the measuring. Better continue to 
> mentally jerkoff in the assumption, for the sake of argument, that there's 
> some serious interpretation of QM going on here. AG
>
> Now as for the problem presented by the title of this thread, if the NO-GO 
> theorems correctly affirm that FTL signaling is impossible, what EXACTLY is 
> the link between this conclusion and the claim the QM is IRREDUCIBLY 
> RANDOM? And what does IRREDUCIBLY RANDOM mean? In trying to answer this 
> question, let me impose one RULE; no jerking off permitted! AG
>
> On Friday, December 25, 2020 at 11:03:51 PM UTC-7 Alan Grayson wrote:
>
>>
>> *You could have omitted your snotty second sentence. Why is it OK for you 
>> to use the List to mentally jerk off about the applicability of Born's rule 
>> in the context of an interpretation you know is false, the MWI, than not 
>> for me to ask a question which might have a simple answer, or path to take 
>> to answer MY question? Being well informed about physics doesn't mean 
>> you're not behaving like an arrogant schmuck. AG*
>> On Thursday, December 24, 2020 at 11:49:42 PM UTC-7 Bruce wrote:
>>
>>> On Fri, Dec 25, 2020 at 5:29 PM Alan Grayson  
>>> wrote:
>>>

 *Now I raise a similar question I posed to Bruce, thrice, with no 
 replies. Why does the unpredictability of measured values and the 
 intrinsic 
 randomness protect relativity theory? This is really a huge conceptual 
 leap. How would you argue for that conclusion, as distinguished from 
 asserting it? TIA, AG*
>>>
>>>
>>> You need to do some research on the no-signalling theorems. This list 
>>> does not exist to answer your elementary questions.
>>>
>>> Bruce
>>>
>>

-- 
You received this message because you are subscribed to the Google Groups 
"Everything List" group.
To unsubscribe from this group and stop receiving emails from it, send an email 
to everything-list+unsubscr...@googlegroups.com.
To view this discussion on the web visit 
https://groups.google.com/d/msgid/everything-list/97efae2d-9115-4eb5-8b76-1a79787ceec7n%40googlegroups.com.


Re: Irreducible randomness in QM

2020-12-26 Thread Alan Grayson
Fact is there's no need to bring in pretentious big brains from Australia 
to show that Born's rule makes no sense in the context of the MWI. For 
simplicity, consider a spin experiment where the observer measures spin UP 
in THIS world, and another world (or branch) is created where spin DN is 
measured. If we do the experiment again, the MWI is AMBIGUOUS on which 
world (or branch) the spin is now measured. On one of the two worlds just 
created, or are another pair of worlds created, one of which measures spin 
UP and another spin DN?  Of course this leaves out all the BS on how these 
worlds are created, along with the spontaneous creation of all the 
observers and experimental set-ups to do the measuring. Better continue to 
mentally jerkoff in the assumption, for the sake of argument, that there's 
some serious interpretation of QM going on here. AG

Now as for the problem presented by the title of this thread, if the NO-GO 
theorems correctly affirm that FTL signaling is impossible, what EXACTLY is 
the link between this conclusion and the claim the QM is IRREDUCIBLY 
RANDOM? And what does IRREDUCIBLY RANDOM mean? In trying to answer this 
question, let me impose one RULE; no jerking off permitted! AG

On Friday, December 25, 2020 at 11:03:51 PM UTC-7 Alan Grayson wrote:

>
> *You could have omitted your snotty second sentence. Why is it OK for you 
> to use the List to mentally jerk off about the applicability of Born's rule 
> in the context of an interpretation you know is false, the MWI, than not 
> for me to ask a question which might have a simple answer, or path to take 
> to answer MY question? Being well informed about physics doesn't mean 
> you're not behaving like an arrogant schmuck. AG*
> On Thursday, December 24, 2020 at 11:49:42 PM UTC-7 Bruce wrote:
>
>> On Fri, Dec 25, 2020 at 5:29 PM Alan Grayson  wrote:
>>
>>>
>>> *Now I raise a similar question I posed to Bruce, thrice, with no 
>>> replies. Why does the unpredictability of measured values and the intrinsic 
>>> randomness protect relativity theory? This is really a huge conceptual 
>>> leap. How would you argue for that conclusion, as distinguished from 
>>> asserting it? TIA, AG*
>>
>>
>> You need to do some research on the no-signalling theorems. This list 
>> does not exist to answer your elementary questions.
>>
>> Bruce
>>
>

-- 
You received this message because you are subscribed to the Google Groups 
"Everything List" group.
To unsubscribe from this group and stop receiving emails from it, send an email 
to everything-list+unsubscr...@googlegroups.com.
To view this discussion on the web visit 
https://groups.google.com/d/msgid/everything-list/7faa292d-cd04-4c7f-9f8c-6d5f27c53c1dn%40googlegroups.com.


Re: Irreducible randomness in QM

2020-12-26 Thread 'scerir' via Everything List
Bruno writes: "It is a theorem. No universal machine can determine which 
computations run it, .. "

Do we need a 'constructor theory'? That is to say a set of 'principles' under 
which we could show whether or not a 'universal constructor' can exist?

https://www.edge.org/conversation/constructor-theory
https://www.edge.org/conversation/constructor-theory 
https://arxiv.org/abs/1210.7439

https://arxiv.org/abs/1210.7439

-- 
You received this message because you are subscribed to the Google Groups 
"Everything List" group.
To unsubscribe from this group and stop receiving emails from it, send an email 
to everything-list+unsubscr...@googlegroups.com.
To view this discussion on the web visit 
https://groups.google.com/d/msgid/everything-list/1274978825.510110.1608997167952%40mail1.libero.it.


Re: Irreducible randomness in QM

2020-12-26 Thread Bruno Marchal

> On 23 Dec 2020, at 03:29, Bruce Kellett  wrote:
> 
> On Wed, Dec 23, 2020 at 12:45 PM Stathis Papaioannou  > wrote:
> On Wed, 23 Dec 2020 at 10:58, Bruce Kellett  > wrote:
> On Wed, Dec 23, 2020 at 10:45 AM Stathis Papaioannou  > wrote:
> On Wed, 23 Dec 2020 at 09:15, Bruce Kellett  > wrote:
> On Wed, Dec 23, 2020 at 9:07 AM Stathis Papaioannou  > wrote:
> On Wed, 23 Dec 2020 at 09:02, Bruce Kellett  > wrote:
> On Wed, Dec 23, 2020 at 8:32 AM Stathis Papaioannou  > wrote:
> On Tue, 22 Dec 2020 at 21:31, Bruce Kellett  > wrote:
> On Tue, Dec 22, 2020 at 9:19 PM Stathis Papaioannou  > wrote:
> 
> All the copies could be conscious or all could be zombies; none are 
> privileged.
> 
> What difference does that make? One has to be privileged in some way if there 
> is to be a probability different from zero.
> 
> Why did you say it was dualist if it doesn't make a difference that it isn't 
> dualist?
> 
> It makes no difference if all copies are conscious, or if all are zombies -- 
> you are still making a dualist assumption.
> 
> The probability calculated where there are multiple copies is the probability 
> that one randomly sampled copy will see a particular outcome. I am one 
> randomly sampled copy.
> 
> 
> And that is precisely the dualist assumption that  is intrinsic in all 
> self-location (indexical) arguments. I think Brent has understood this when 
> he says "That seems to imply dualism.  All the bodies exist, but your soul 
> only goes with one."
> 
> I could say that my soul is duplicated and I want to know the probability 
> that I am one randomly sampled soul. I could say that the carrots are 
> duplicated and I want to know the probability that I get a particular 
> randomly sampled carrot. I don't have a problem with it; you do, and there 
> seems to be no way around it.
> 
> 
> Think of it like this: take a randomly shuffled deck of cards and hand one 
> card from the deck to each of 52 people. The probability that one of the 
> people will get the 3-of-Spades is one. The probability that 'You' will get 
> the 3-of-Spades in a fair shuffle is 1/52. The difference is that you have 
> identified yourself in advance. The dualist assumption is equivalent.
> 
> Let's say you are copied 10^100 times. One copy will end up in a place where 
> they use euros and the rest will end up in a place where they use dollars. Do 
> you put euros or dollars in your wallet before duplication?
> 
> 
> Let's say I wait and see! and go to the money exchange if necessary. You are 
> posing a different problem, one in which the number of copies on a particular 
> branch is increased. That is incompatible with MWI and Everett with 
> non-degenerate eigenvalues.
> 
> You don't avoid the dualist implications of self-selection by increasing the 
> number of copies: the example with 52 cards says everything that is necessary.
> 
> From what I understand of your position, you would claim that the 1 in 10^100 
> copy will screw up the very concept of probability. If that extra copy did 
> not exist, you would take dollars, because you will certainly need dollars; 
> but with the extra copy you would just throw up your hands and say you don't 
> know what to do, because it is certain you will need dollars and euros.
> 
> 
> My complaint about your example is that you are changing the problem -- you 
> are changing the probabilities in a way that is incompatible with both the 
> Schrodinger equation and the Born rule. But there could be more moderate 
> examples of branch duplication that would be more in line with what is 
> proposed by some people. For example, both Sean Carroll and Zurek propose a 
> procedure whereby they expand the number of branches so that all branches 
> have equal amplitudes (weights, or Born probabilities). This is incompatible 
> with the Schrodinger equation, but if we leave that aside for the moment, it 
> gives a branch-counting solution to the probability question. The idea then 
> is that you self-select from a uniform random distribution over this expanded 
> set of branches.  However, the expansion of the number of branches in this 
> approach is, in fact, unnecessary, since random self-selection from a 
> distribution would give the same result if the distribution were determined 
> directly by the Born rule.
> 
> But this is still inconsistent with the Schrodinger equation because there is 
> nothing in the SE that tells you that you have a probability distribution 
> given by the Born weights. You can impose the Born rule by fiat, but that is 
> then incompatible with the fact that every outcome in the Schrodinger 
> equation occurs with probability equal to one. (Which is where we started).
> 
> The self-selection idea, whether from 

Re: Irreducible randomness in QM

2020-12-26 Thread Bruno Marchal

> On 23 Dec 2020, at 00:44, Bruce Kellett  wrote:
> 
> On Wed, Dec 23, 2020 at 10:35 AM Stathis Papaioannou  > wrote:
> On Wed, 23 Dec 2020 at 09:15, Bruce Kellett  > wrote:
> On Wed, Dec 23, 2020 at 9:07 AM Stathis Papaioannou  > wrote:
> On Wed, 23 Dec 2020 at 09:02, Bruce Kellett  > wrote:
> On Wed, Dec 23, 2020 at 8:32 AM Stathis Papaioannou  > wrote:
> On Tue, 22 Dec 2020 at 21:31, Bruce Kellett  > wrote:
> On Tue, Dec 22, 2020 at 9:19 PM Stathis Papaioannou  > wrote:
> 
> All the copies could be conscious or all could be zombies; none are 
> privileged.
> 
> What difference does that make? One has to be privileged in some way if there 
> is to be a probability different from zero.
> 
> Why did you say it was dualist if it doesn't make a difference that it isn't 
> dualist?
> 
> It makes no difference if all copies are conscious, or if all are zombies -- 
> you are still making a dualist assumption.
> 
> The probability calculated where there are multiple copies is the probability 
> that one randomly sampled copy will see a particular outcome. I am one 
> randomly sampled copy.
> 
> 
> And that is precisely the dualist assumption that  is intrinsic in all 
> self-location (indexical) arguments. I think Brent has understood this when 
> he says "That seems to imply dualism.  All the bodies exist, but your soul 
> only goes with one."
> 
> I could say that my soul is duplicated and I want to know the probability 
> that I am one randomly sampled soul. I could say that the carrots are 
> duplicated and I want to know the probability that I get a particular 
> randomly sampled carrot. I don't have a problem with it; you do, and there 
> seems to be no way around it.
> 
> 
> Think of it like this: take a randomly shuffled deck of cards and hand one 
> card from the deck to each of 52 people. The probability that one of the 
> people will get the 3-of-Spades is one. The probability that 'You' will get 
> the 3-of-Spades in a fair shuffle is 1/52. The difference is that you have 
> identified yourself in advance. The dualist assumption is equivalent.
> 
> The probability that one particular randomly sampled person will get the 3 of 
> spades is 1/52.
> 
> 
> As I said. But who or what does the random sampling so that you are the 
> selected person? You cannot escape the dualist implications that easily.

That is the dualism between 3p (or []p) and 1p (or []p & p). That dualism is 
phenomenological, and unavoidable for any sufficiently reflexive machine.

That phenomenological account is unavoidable, unless you identify truth and 
proof, but then you have to abandon mechanism (and thus Darwin & Co. but also 
QM-without-collapse). So you will need to eliminate consciousness and first 
person, and you will need a sort of ad hoc God to make the selection.

Bruno


> 
> Bruce
> 
> -- 
> You received this message because you are subscribed to the Google Groups 
> "Everything List" group.
> To unsubscribe from this group and stop receiving emails from it, send an 
> email to everything-list+unsubscr...@googlegroups.com 
> .
> To view this discussion on the web visit 
> https://groups.google.com/d/msgid/everything-list/CAFxXSLSriUUBW59tZzT3vb%3D39DRKd%3DR%3DOq8ZhcShnJ-FN%2BY1ZQ%40mail.gmail.com
>  
> .

-- 
You received this message because you are subscribed to the Google Groups 
"Everything List" group.
To unsubscribe from this group and stop receiving emails from it, send an email 
to everything-list+unsubscr...@googlegroups.com.
To view this discussion on the web visit 
https://groups.google.com/d/msgid/everything-list/08610A89-AACE-4141-9248-E51D72B5AC7E%40ulb.ac.be.


Re: Irreducible randomness in QM

2020-12-26 Thread John Clark
On Sat, Dec 26, 2020 at 8:11 AM Bruno Marchal  wrote:

*> John Clark’s attempt to refute the Digital Mechanist First Person
> Indeterminacy.*


No. I am not arrogant enough to dispute it. I am "the first person" (I
guess) and I don't know everything, very often "I" don't know what's going
to happen next, not even approximately.

  John K Clark



>

-- 
You received this message because you are subscribed to the Google Groups 
"Everything List" group.
To unsubscribe from this group and stop receiving emails from it, send an email 
to everything-list+unsubscr...@googlegroups.com.
To view this discussion on the web visit 
https://groups.google.com/d/msgid/everything-list/CAJPayv2%3DeH54CXC5-ZJwe64ZDp%3Dx5Aa%3DfF1oy%2Bs052MGPBjWMg%40mail.gmail.com.


Re: Irreducible randomness in QM

2020-12-26 Thread Bruno Marchal

> On 22 Dec 2020, at 22:52, Bruce Kellett  wrote:
> 
> On Wed, Dec 23, 2020 at 2:37 AM Bruno Marchal  > wrote:
>> Am Mo, 21. Dez 2020, um 21:35, schrieb Bruce Kellett:
>>> On Mon, Dec 21, 2020 at 10:56 PM John Clark >> > wrote:
>>> 
>>> On Sun, Dec 20, 2020 at 1:38 AM Bruce Kellett >> > wrote:
>>> 
>>> > MWI is incompatible with the Born Rule
>>> 
>>> How do you figure that?  
>>> 
>>> 
>>> It's easy enough. MWI from the Schrodinger equation says that every outcome 
>>> happens, with probability one.
> 
> That is an exemple of confusion between the third person pictured where 
> indeed MWI keep all superposition “intacte”, and that all possible quantum 
> outcome are realised, and the first person account where that does not 
> happen, as each brain is correlated to the terms of the superposition.
> 
> The 1p/3p distinction does not help you here. There are many 1p views,

OK. We are not solipsist. No zombies.



> and for each the probability of that particular observation is one,

Not before the observation is done. The prediction “cat dead” will be surely 
wrong for the guy who saw the cat alive. Only after the observation is done can 
each one claim the probability of (remeasuring) the cat state is one, but that 
is just the usual outcome when two similar measurement are done. 



> contradicting the Born rule calculation of the probability in every case.

The prediction have to be done before the experience.




> This follows from the linearity of the Schrodinger equation. The Born rule 
> cannot be deduced from the Schrodinger equation -- they are incompatible.

That is why Everett use not just the SE, but the SE + the mechanist theory of 
mind. (But then he missed the fact that he has to justify the SE from the logic 
of machine self-reference, …at least to solve the Mind-Body problem).

Bruno



> 
> Bruce
> 
> -- 
> You received this message because you are subscribed to the Google Groups 
> "Everything List" group.
> To unsubscribe from this group and stop receiving emails from it, send an 
> email to everything-list+unsubscr...@googlegroups.com 
> .
> To view this discussion on the web visit 
> https://groups.google.com/d/msgid/everything-list/CAFxXSLQkBEnR3Q32hn9xXyeT3D%2BBB26_nkv7QbCfH-WSRnfvxQ%40mail.gmail.com
>  
> .

-- 
You received this message because you are subscribed to the Google Groups 
"Everything List" group.
To unsubscribe from this group and stop receiving emails from it, send an email 
to everything-list+unsubscr...@googlegroups.com.
To view this discussion on the web visit 
https://groups.google.com/d/msgid/everything-list/072F4180-9503-4C0A-AAD4-46A2AB726D56%40ulb.ac.be.


Re: Irreducible randomness in QM

2020-12-26 Thread Bruno Marchal

> On 22 Dec 2020, at 21:14, 'scerir' via Everything List 
>  wrote:
> 
> Stathis Papaioannou wrote:
> All the copies could be conscious or all could be zombies; none are 
> privileged.
> 
> "In truth there is only one mind. Oneness it is the doctrine of the 
> Upanishads." As far as I remember Schroedinger wrote something like that. 
> Does that "Oneness" could resolve our problem? :-)

It works indeed, if you accept that such a One can differentiate when having 
different experience. One mind is global. Many minds is local, indexical, and 
can have locally contingent aspect.

The only problem is that it makes all universal machine conscious, even when 
not reflexive (aka Löbian, knowing their own universality).

Bruno



> 
> 
> 
> -- 
> You received this message because you are subscribed to the Google Groups 
> "Everything List" group.
> To unsubscribe from this group and stop receiving emails from it, send an 
> email to everything-list+unsubscr...@googlegroups.com 
> .
> To view this discussion on the web visit 
> https://groups.google.com/d/msgid/everything-list/1862682578.460029.1608668067995%40mail1.libero.it
>  
> .

-- 
You received this message because you are subscribed to the Google Groups 
"Everything List" group.
To unsubscribe from this group and stop receiving emails from it, send an email 
to everything-list+unsubscr...@googlegroups.com.
To view this discussion on the web visit 
https://groups.google.com/d/msgid/everything-list/33FA9303-E6D5-479B-98F9-78FCCBE4DA0D%40ulb.ac.be.


Re: Irreducible randomness in QM

2020-12-26 Thread Bruno Marchal

> On 22 Dec 2020, at 19:12, 'Brent Meeker' via Everything List 
>  wrote:
> 
> 
> 
> On 12/22/2020 12:35 AM, Stathis Papaioannou wrote:
>> 
>> 
>> On Tue, 22 Dec 2020 at 18:21, Bruce Kellett > > wrote:
>> On Tue, Dec 22, 2020 at 5:58 PM Stathis Papaioannou > > wrote:
>> On Tue, 22 Dec 2020 at 08:51, Bruce Kellett > > wrote:
>> On Tue, Dec 22, 2020 at 8:47 AM Stathis Papaioannou > > wrote:
>> On Tue, 22 Dec 2020 at 08:35, Bruce Kellett > > wrote:
>> On Mon, Dec 21, 2020 at 10:56 PM John Clark > > wrote:
>> 
>> On Sun, Dec 20, 2020 at 1:38 AM Bruce Kellett > > wrote:
>> 
>> > MWI is incompatible with the Born Rule
>> 
>> How do you figure that?  
>> 
>> 
>> It's easy enough. MWI from the Schrodinger equation says that every outcome 
>> happens, with probability one. The Born rule says that different outcomes 
>> have different probabilities. So MWI + Born gives two incompatible results 
>> for outcome probabilities. Hence Everett is incoherent -- incompatible with 
>> the Born rule.
>> 
>> The probability that an outcome happens and the probability that an observer 
>> will see a particular outcome are two different things.
>> 
>> Only if you say so -- Nature doesn't care what you say! There is an observer 
>> for every outcome. Or do you really believe in a dualist model?
>> 
>> Nature agrees that different observers will see different outcomes, since 
>> they are not in telepathic communication.
>> 
>> Which observer are you? What outcome do you see?
>> 
>> I am potentially any of them, but only one, which is why it is probabilistic.
> 
> That seems to imply dualism.  All the bodies exist, but your soul only goes 
> with one.

It is the dualism between p (truth) and Bp (believable), or between Bp and Bp & 
p, at another level.

In Arithmetic (I use a big A for the semantic notion, and little “a", for 
theories (like RA, PA and ZF) all computations are emulated, but in each of 
them you cannot be aware of the others (for obvious reason, as telepathy is 
inconsistent with Mechanism).

Bruno




> 
> Brent
> 
>> This is independent of consciousness and quantum mechanics. A rational 
>> character in a computer game that branches would reason the same way.
>> -- 
>> Stathis Papaioannou
>> -- 
>> You received this message because you are subscribed to the Google Groups 
>> "Everything List" group.
>> To unsubscribe from this group and stop receiving emails from it, send an 
>> email to everything-list+unsubscr...@googlegroups.com 
>> .
>> To view this discussion on the web visit 
>> https://groups.google.com/d/msgid/everything-list/CAH%3D2ypUD9EWZ0M%3DQXBgRZc91FRd1TndtZHrHC%2B0YQUGm1yPsrg%40mail.gmail.com
>>  
>> .
> 
> 
> -- 
> You received this message because you are subscribed to the Google Groups 
> "Everything List" group.
> To unsubscribe from this group and stop receiving emails from it, send an 
> email to everything-list+unsubscr...@googlegroups.com 
> .
> To view this discussion on the web visit 
> https://groups.google.com/d/msgid/everything-list/0a6d9d69-cdc1-94b5-2330-eab16801b474%40verizon.net
>  
> .

-- 
You received this message because you are subscribed to the Google Groups 
"Everything List" group.
To unsubscribe from this group and stop receiving emails from it, send an email 
to everything-list+unsubscr...@googlegroups.com.
To view this discussion on the web visit 
https://groups.google.com/d/msgid/everything-list/64C8C136-EF6E-4A7C-925D-9109640D616C%40ulb.ac.be.


Re: Irreducible randomness in QM

2020-12-26 Thread Bruno Marchal

> On 22 Dec 2020, at 09:47, Bruce Kellett  wrote:
> 
> On Tue, Dec 22, 2020 at 7:35 PM Stathis Papaioannou  > wrote:
> On Tue, 22 Dec 2020 at 18:21, Bruce Kellett  > wrote:
> On Tue, Dec 22, 2020 at 5:58 PM Stathis Papaioannou  > wrote:
> On Tue, 22 Dec 2020 at 08:51, Bruce Kellett  > wrote:
> On Tue, Dec 22, 2020 at 8:47 AM Stathis Papaioannou  > wrote:
> On Tue, 22 Dec 2020 at 08:35, Bruce Kellett  > wrote:
> On Mon, Dec 21, 2020 at 10:56 PM John Clark  > wrote:
> 
> On Sun, Dec 20, 2020 at 1:38 AM Bruce Kellett  > wrote:
> 
> > MWI is incompatible with the Born Rule
> 
> How do you figure that?  
> 
> 
> It's easy enough. MWI from the Schrodinger equation says that every outcome 
> happens, with probability one. The Born rule says that different outcomes 
> have different probabilities. So MWI + Born gives two incompatible results 
> for outcome probabilities. Hence Everett is incoherent -- incompatible with 
> the Born rule.
> 
> The probability that an outcome happens and the probability that an observer 
> will see a particular outcome are two different things.
> 
> Only if you say so -- Nature doesn't care what you say! There is an observer 
> for every outcome. Or do you really believe in a dualist model?
> 
> Nature agrees that different observers will see different outcomes, since 
> they are not in telepathic communication.
> 
> Which observer are you? What outcome do you see?
> 
> I am potentially any of them, but only one, which is why it is probabilistic. 
> This is independent of consciousness and quantum mechanics. A rational 
> character in a computer game that branches would reason the same way.
> 
> 
> Yes, it is a problem inherent in all indexical reasoning. Either you are all 
> of the copies, and hence the probability for you to see any particular 
> outcome is unity, or you are only one of the copies, and the rest are zombies.

Why zombies? There are just indexically first person different. 



> That is the dualist position, and it is necessary if you want to get 
> probabilities other than unity for outcomes.

Only by reifying Nature. But in serious metaphysics, we cannot invoke the 
ontology as such in any reasoning. That is equivalent with “God made it”, 
which, if true, has to be explained from less (than God!).



> 
> The reasoning is like that in the many minds model of QM of Albert and 
> Loewer, and they now explicitly acknowledge that the reasoning underlying 
> that model is manifestly dualist.

They keep, like many, a naïve idea of what could constitute a “world”. I think. 
The universal machine corrects this, all by herself.

Bruno



> 
> Bruce
> 
> -- 
> You received this message because you are subscribed to the Google Groups 
> "Everything List" group.
> To unsubscribe from this group and stop receiving emails from it, send an 
> email to everything-list+unsubscr...@googlegroups.com 
> .
> To view this discussion on the web visit 
> https://groups.google.com/d/msgid/everything-list/CAFxXSLRtSe1iOj5hwQhQ3QhHSm5ha2VbjAAofxDKZqX1rCPsYw%40mail.gmail.com
>  
> .

-- 
You received this message because you are subscribed to the Google Groups 
"Everything List" group.
To unsubscribe from this group and stop receiving emails from it, send an email 
to everything-list+unsubscr...@googlegroups.com.
To view this discussion on the web visit 
https://groups.google.com/d/msgid/everything-list/B17902B2-E42D-4D13-A7EE-65820260D603%40ulb.ac.be.


Re: Irreducible randomness in QM

2020-12-26 Thread Bruno Marchal

> On 22 Dec 2020, at 07:58, Stathis Papaioannou  wrote:
> 
> 
> 
> On Tue, 22 Dec 2020 at 08:51, Bruce Kellett  > wrote:
> On Tue, Dec 22, 2020 at 8:47 AM Stathis Papaioannou  > wrote:
> On Tue, 22 Dec 2020 at 08:35, Bruce Kellett  > wrote:
> On Mon, Dec 21, 2020 at 10:56 PM John Clark  > wrote:
> 
> On Sun, Dec 20, 2020 at 1:38 AM Bruce Kellett  > wrote:
> 
> > MWI is incompatible with the Born Rule
> 
> How do you figure that?  
> 
> 
> It's easy enough. MWI from the Schrodinger equation says that every outcome 
> happens, with probability one. The Born rule says that different outcomes 
> have different probabilities. So MWI + Born gives two incompatible results 
> for outcome probabilities. Hence Everett is incoherent -- incompatible with 
> the Born rule.
> 
> The probability that an outcome happens and the probability that an observer 
> will see a particular outcome are two different things.
> 
> Only if you say so -- Nature doesn't care what you say! There is an observer 
> for every outcome. Or do you really believe in a dualist model?
> 
> Nature agrees that different observers will see different outcomes, since 
> they are not in telepathic communication.


… which is my usual answer to John Clark’s attempt to refute the Digital 
Mechanist First Person Indeterminacy.

At least Bruce Kellet is coherent, as he uses it for both MWI and Mechanism. 

Bruno



> -- 
> Stathis Papaioannou
> 
> -- 
> You received this message because you are subscribed to the Google Groups 
> "Everything List" group.
> To unsubscribe from this group and stop receiving emails from it, send an 
> email to everything-list+unsubscr...@googlegroups.com 
> .
> To view this discussion on the web visit 
> https://groups.google.com/d/msgid/everything-list/CAH%3D2ypVt4aSCbBPGhG_T5DnaO2pY0NDHNPKLyUojSz3O0xaTbg%40mail.gmail.com
>  
> .

-- 
You received this message because you are subscribed to the Google Groups 
"Everything List" group.
To unsubscribe from this group and stop receiving emails from it, send an email 
to everything-list+unsubscr...@googlegroups.com.
To view this discussion on the web visit 
https://groups.google.com/d/msgid/everything-list/58FCF780-E313-42A6-953F-7BAEEE50513F%40ulb.ac.be.


Re: Irreducible randomness in QM

2020-12-26 Thread Bruno Marchal

> On 21 Dec 2020, at 12:55, John Clark  wrote:
> 
> 
> On Sun, Dec 20, 2020 at 1:38 AM Bruce Kellett  > wrote:
> 
> > MWI is incompatible with the Born Rule
> 
> How do you figure that?  

Good question.

For me it is an open problem to decide if the MWI does or not imply logically 
the Born Rule. I do think it does (by either Gleason theorem, or by the 
Destouches-Février, Graham, Hartle, “usual” account.

But Bruce seems to believe that the MWI refutes the Born Rule, but I have not 
yet understood his argument. 
Anyway, the notion of “world” is never clearly defined, and my own (mechanist) 
interpretation of QM is more a statistics on computations/histories (which 
exists in arithmetic) than on worlds or models (in the logician sense).

Bruno



> 
> John K Clark  To see my new list go to  extropolis 
> 
> 
> -- 
> You received this message because you are subscribed to the Google Groups 
> "Everything List" group.
> To unsubscribe from this group and stop receiving emails from it, send an 
> email to everything-list+unsubscr...@googlegroups.com 
> .
> To view this discussion on the web visit 
> https://groups.google.com/d/msgid/everything-list/CAJPayv0SaD_aw%3Dbfk3nKQKqcaSs5i%3Dp5Lcqgwqzj_Kj%3D89k-uQ%40mail.gmail.com
>  
> .

-- 
You received this message because you are subscribed to the Google Groups 
"Everything List" group.
To unsubscribe from this group and stop receiving emails from it, send an email 
to everything-list+unsubscr...@googlegroups.com.
To view this discussion on the web visit 
https://groups.google.com/d/msgid/everything-list/E7A8A14D-07F4-478E-8191-BE5DD166D25C%40ulb.ac.be.


Re: Irreducible randomness in QM

2020-12-26 Thread Bruno Marchal

> On 19 Dec 2020, at 11:18, Alan Grayson  wrote:
> 
> 
> 
> On Saturday, December 19, 2020 at 1:30:17 AM UTC-7 sce...@libero.it wrote:
> https://arxiv.org/abs/1908.07068 
> Randomness? What randomness?
> 
> Klaas Landsman 
>  
> This is a review of the issue of randomness in quantum mechanics, with 
> special emphasis on its ambiguity; for example, randomness has different 
> antipodal relationships to determinism, computability, and compressibility. 
> Following a (Wittgensteinian) philosophical discussion of randomness in 
> general, I argue that deterministic interpretations of quantum mechanics 
> (like Bohmian mechanics or 't Hooft's Cellular Automaton interpretation) are 
> strictly speaking incompatible with the Born rule. I also stress the role of 
> outliers, i.e. measurement outcomes that are not 1-random. Although these 
> occur with low (or even zero) probability, their very existence implies that 
> the no-signaling principle used in proofs of randomness of outcomes of 
> quantum-mechanical measurements (and of the safety of quantum cryptography) 
> should be reinterpreted statistically, like the second law of thermodynamics. 
> In appendices I discuss the Born rule and its status in both single and 
> repeated experiments, and review the notion of 1-randomness introduced by 
> Kolmogorov, Chaitin, Martin-Lo"f, Schnorr, and others.
> 
> This might be helpful, but more likely over my head. What I want to know is 
> WHO originally came up with the interpretation of QM that it is irreducibly 
> random

Born


> -- meaning that in principle there is no way to predetermine outcomes of 
> experiments -- and WHAT was the justification.


That the wave described some intensities, together with the fact that the wave 
described also unique system. Like Feynman and Deutsch say: you need already 
this to explain the (thought) experiment with two slits, and with the particles 
sent one by one. The wave amplitude seems to be a wave of probability, but 
eventually it is the square of the wave which has to be interpreted as a 
probability. With the MWI it is a probability on relatively accessible 
computational state, confirming the Mechanist consequence in physics: physics 
is the science of the probability on our continuations in arithmetic (or in any 
Turing universal machinery).

Bruno



> I think it was Bohr, and what was his reasoning for this interpretation, 
> which is the "end of the road" for any theory better than QM. AG 
> 
> -- 
> You received this message because you are subscribed to the Google Groups 
> "Everything List" group.
> To unsubscribe from this group and stop receiving emails from it, send an 
> email to everything-list+unsubscr...@googlegroups.com 
> .
> To view this discussion on the web visit 
> https://groups.google.com/d/msgid/everything-list/8829438e-9fe9-42fe-93fe-4f028fbdd420n%40googlegroups.com
>  
> .

-- 
You received this message because you are subscribed to the Google Groups 
"Everything List" group.
To unsubscribe from this group and stop receiving emails from it, send an email 
to everything-list+unsubscr...@googlegroups.com.
To view this discussion on the web visit 
https://groups.google.com/d/msgid/everything-list/B4C362EE-BD24-440C-B3E7-A63E80136694%40ulb.ac.be.


Re: Irreducible randomness in QM

2020-12-26 Thread Stathis Papaioannou
On Sat, 26 Dec 2020 at 17:04, Bruce Kellett  wrote:

> On Sat, Dec 26, 2020 at 4:02 PM Stathis Papaioannou 
> wrote:
>
>> On Sat, 26 Dec 2020 at 14:14, Bruce Kellett 
>> wrote:
>>
>>> On Sat, Dec 26, 2020 at 2:08 PM Stathis Papaioannou 
>>> wrote:
>>>
 On Sat, 26 Dec 2020 at 13:01, Bruce Kellett 
 wrote:

> On Sat, Dec 26, 2020 at 12:32 PM Stathis Papaioannou <
> stath...@gmail.com> wrote:
>
>> On Sat, 26 Dec 2020 at 09:02, Bruce Kellett 
>> wrote:
>>
>>> On Fri, Dec 25, 2020 at 11:29 PM Stathis Papaioannou <
>>> stath...@gmail.com> wrote:
>>>
 On Fri, 25 Dec 2020 at 22:17, Bruce Kellett 
 wrote:

>
> It can be a metaphysical truth without there being any dualist
> underpinnings. The problem, as you point out, is when there are 
> multiple
> copies of you extant at a single time. If you consider yourself to be 
> a
> random selection from this reference class, then you have made the 
> dualist
> assumption that there is something that picks you out -- something 
> that
> distinguishes you from all the other copies. Whereas, in reality, all 
> the
> copies are the same and must think the same: they can deduce that 
> they are
> not special, and that the probability for the existence of each copy 
> is
> exactly one -- the Born probabilities have no bearing on their 
> existence
> because they inevitably exist regardless of the magnitude of the
> mod-squared quantum amplitude. There is no process that selects just 
> one
> individual copy at random from a distribution, whether it be the 
> uniform
> distribution over branches, or the probability distribution obtained 
> by
> Born weighting each branch.
>

 I know that all the other copies feel as I do, that they are the
 unique continuation of the original. For an entity that feels this 
 way, the
 Born probabilities apply.

>>>
>>> The Born probabilities apply because a single-world model has been
>>> adopted by default. You cannot prove that there are any intrinsic
>>> probabilities in a deterministic branching world.
>>>
>>
>> People in a deterministic branching world will say “I tossed a coin
>> many times and about half the time it came up heads”. They will then 
>> assign
>> a probability of 1/2 to heads coming up on the next toss. I don’t know if
>> you would call this an “intrinsic probability”, but that is what would
>> happen.
>>
>
> That is probability coming from ignorance of initial conditions. Sure,
> that applies in a deterministic world too. But the intrinsic probability 
> of
> interest is specifically quantum: the probability that a radioactive
> nucleus will decay within the next specific time period; or the 
> probability
> that you will get spin-up on a S-G measurement of a left-spin atom. This 
> is
> the probability of the Born rule, and that cannot be derived in a
> deterministic theory.
>

 Well, what you call the “probability coming from ignorance of initial
 conditions” is the probability that everyone is interested in when it comes
 to making decisions about their everyday lives.

>>>
>>>
>>> Is that relevant to a discussion of probability in quantum mechanics?
>>>
>>
>> Yes, because it is the only probability relevant to beings like us when
>> they make observations while embedded in the many worlds.
>>
>
> That is a surprising admission. So you are claiming that there are no
> intrinsic probabilities other than those due to ignorance of initial
> conditions and the like? In other words, the Born probabilities based on
> the mod-square of quantum amplitudes are irrelevant to beings in many
> worlds. That is what I have been arguing all along -- the Born
> probabilities are inconsistent with the unit probabilities and
> deterministic outcomes of Everettian QM. The only chances in your view of
> the world are those due to ignorance -- and there is no ignorance of either
> the initial state, or of the possible outcomes, in a standard quantum
> experiment. Such a view is, of course, at variance with the results of
> quantum mechanics.
>

The Born probabilities are the subjective probabilities seen by beings
embedded in the Many Worlds. Objectively, all the branches occur with
probability 1 given Many Worlds, but this is trivial. You seem to devalue
subjective probability, but subjectivity is very important to people.

> --
Stathis Papaioannou

-- 
You received this message because you are subscribed to the Google Groups 
"Everything List" group.
To unsubscribe from this group and stop receiving emails from it, send an email 
to everything-list+unsubscr...@googlegroups.com.
To view this 

Re: Irreducible randomness in QM

2020-12-25 Thread Bruce Kellett
On Sat, Dec 26, 2020 at 4:02 PM Stathis Papaioannou 
wrote:

> On Sat, 26 Dec 2020 at 14:14, Bruce Kellett  wrote:
>
>> On Sat, Dec 26, 2020 at 2:08 PM Stathis Papaioannou 
>> wrote:
>>
>>> On Sat, 26 Dec 2020 at 13:01, Bruce Kellett 
>>> wrote:
>>>
 On Sat, Dec 26, 2020 at 12:32 PM Stathis Papaioannou <
 stath...@gmail.com> wrote:

> On Sat, 26 Dec 2020 at 09:02, Bruce Kellett 
> wrote:
>
>> On Fri, Dec 25, 2020 at 11:29 PM Stathis Papaioannou <
>> stath...@gmail.com> wrote:
>>
>>> On Fri, 25 Dec 2020 at 22:17, Bruce Kellett 
>>> wrote:
>>>

 It can be a metaphysical truth without there being any dualist
 underpinnings. The problem, as you point out, is when there are 
 multiple
 copies of you extant at a single time. If you consider yourself to be a
 random selection from this reference class, then you have made the 
 dualist
 assumption that there is something that picks you out -- something that
 distinguishes you from all the other copies. Whereas, in reality, all 
 the
 copies are the same and must think the same: they can deduce that they 
 are
 not special, and that the probability for the existence of each copy is
 exactly one -- the Born probabilities have no bearing on their 
 existence
 because they inevitably exist regardless of the magnitude of the
 mod-squared quantum amplitude. There is no process that selects just 
 one
 individual copy at random from a distribution, whether it be the 
 uniform
 distribution over branches, or the probability distribution obtained by
 Born weighting each branch.

>>>
>>> I know that all the other copies feel as I do, that they are the
>>> unique continuation of the original. For an entity that feels this way, 
>>> the
>>> Born probabilities apply.
>>>
>>
>> The Born probabilities apply because a single-world model has been
>> adopted by default. You cannot prove that there are any intrinsic
>> probabilities in a deterministic branching world.
>>
>
> People in a deterministic branching world will say “I tossed a coin
> many times and about half the time it came up heads”. They will then 
> assign
> a probability of 1/2 to heads coming up on the next toss. I don’t know if
> you would call this an “intrinsic probability”, but that is what would
> happen.
>

 That is probability coming from ignorance of initial conditions. Sure,
 that applies in a deterministic world too. But the intrinsic probability of
 interest is specifically quantum: the probability that a radioactive
 nucleus will decay within the next specific time period; or the probability
 that you will get spin-up on a S-G measurement of a left-spin atom. This is
 the probability of the Born rule, and that cannot be derived in a
 deterministic theory.

>>>
>>> Well, what you call the “probability coming from ignorance of initial
>>> conditions” is the probability that everyone is interested in when it comes
>>> to making decisions about their everyday lives.
>>>
>>
>>
>> Is that relevant to a discussion of probability in quantum mechanics?
>>
>
> Yes, because it is the only probability relevant to beings like us when
> they make observations while embedded in the many worlds.
>

That is a surprising admission. So you are claiming that there are no
intrinsic probabilities other than those due to ignorance of initial
conditions and the like? In other words, the Born probabilities based on
the mod-square of quantum amplitudes are irrelevant to beings in many
worlds. That is what I have been arguing all along -- the Born
probabilities are inconsistent with the unit probabilities and
deterministic outcomes of Everettian QM. The only chances in your view of
the world are those due to ignorance -- and there is no ignorance of either
the initial state, or of the possible outcomes, in a standard quantum
experiment. Such a view is, of course, at variance with the results of
quantum mechanics.

Bruce

-- 
You received this message because you are subscribed to the Google Groups 
"Everything List" group.
To unsubscribe from this group and stop receiving emails from it, send an email 
to everything-list+unsubscr...@googlegroups.com.
To view this discussion on the web visit 
https://groups.google.com/d/msgid/everything-list/CAFxXSLTQ6KehQf81hpQSxstKcdobFCi6XcC_pOM0NTTnz%2BWh3Q%40mail.gmail.com.


Re: Irreducible randomness in QM

2020-12-25 Thread Alan Grayson

*You could have omitted your snotty second sentence. Why is it OK for you 
to use the List to mentally jerk off about the applicability of Born's rule 
in the context of an interpretation you know is false, the MWI, than not 
for me to ask a question which might have a simple answer, or path to take 
to answer MY question? Being well informed about physics doesn't mean 
you're not behaving like an arrogant schmuck. AG*
On Thursday, December 24, 2020 at 11:49:42 PM UTC-7 Bruce wrote:

> On Fri, Dec 25, 2020 at 5:29 PM Alan Grayson  wrote:
>
>>
>> *Now I raise a similar question I posed to Bruce, thrice, with no 
>> replies. Why does the unpredictability of measured values and the intrinsic 
>> randomness protect relativity theory? This is really a huge conceptual 
>> leap. How would you argue for that conclusion, as distinguished from 
>> asserting it? TIA, AG*
>
>
> You need to do some research on the no-signalling theorems. This list does 
> not exist to answer your elementary questions.
>
> Bruce
>

-- 
You received this message because you are subscribed to the Google Groups 
"Everything List" group.
To unsubscribe from this group and stop receiving emails from it, send an email 
to everything-list+unsubscr...@googlegroups.com.
To view this discussion on the web visit 
https://groups.google.com/d/msgid/everything-list/56a4b677-4335-4030-bd23-95666aeb204fn%40googlegroups.com.


Re: Irreducible randomness in QM

2020-12-25 Thread Stathis Papaioannou
On Sat, 26 Dec 2020 at 14:14, Bruce Kellett  wrote:

> On Sat, Dec 26, 2020 at 2:08 PM Stathis Papaioannou 
> wrote:
>
>> On Sat, 26 Dec 2020 at 13:01, Bruce Kellett 
>> wrote:
>>
>>> On Sat, Dec 26, 2020 at 12:32 PM Stathis Papaioannou 
>>> wrote:
>>>
 On Sat, 26 Dec 2020 at 09:02, Bruce Kellett 
 wrote:

> On Fri, Dec 25, 2020 at 11:29 PM Stathis Papaioannou <
> stath...@gmail.com> wrote:
>
>> On Fri, 25 Dec 2020 at 22:17, Bruce Kellett 
>> wrote:
>>
>>>
>>> It can be a metaphysical truth without there being any dualist
>>> underpinnings. The problem, as you point out, is when there are multiple
>>> copies of you extant at a single time. If you consider yourself to be a
>>> random selection from this reference class, then you have made the 
>>> dualist
>>> assumption that there is something that picks you out -- something that
>>> distinguishes you from all the other copies. Whereas, in reality, all 
>>> the
>>> copies are the same and must think the same: they can deduce that they 
>>> are
>>> not special, and that the probability for the existence of each copy is
>>> exactly one -- the Born probabilities have no bearing on their existence
>>> because they inevitably exist regardless of the magnitude of the
>>> mod-squared quantum amplitude. There is no process that selects just one
>>> individual copy at random from a distribution, whether it be the uniform
>>> distribution over branches, or the probability distribution obtained by
>>> Born weighting each branch.
>>>
>>
>> I know that all the other copies feel as I do, that they are the
>> unique continuation of the original. For an entity that feels this way, 
>> the
>> Born probabilities apply.
>>
>
> The Born probabilities apply because a single-world model has been
> adopted by default. You cannot prove that there are any intrinsic
> probabilities in a deterministic branching world.
>

 People in a deterministic branching world will say “I tossed a coin
 many times and about half the time it came up heads”. They will then assign
 a probability of 1/2 to heads coming up on the next toss. I don’t know if
 you would call this an “intrinsic probability”, but that is what would
 happen.

>>>
>>> That is probability coming from ignorance of initial conditions. Sure,
>>> that applies in a deterministic world too. But the intrinsic probability of
>>> interest is specifically quantum: the probability that a radioactive
>>> nucleus will decay within the next specific time period; or the probability
>>> that you will get spin-up on a S-G measurement of a left-spin atom. This is
>>> the probability of the Born rule, and that cannot be derived in a
>>> deterministic theory.
>>>
>>
>> Well, what you call the “probability coming from ignorance of initial
>> conditions” is the probability that everyone is interested in when it comes
>> to making decisions about their everyday lives.
>>
>
>
> Is that relevant to a discussion of probability in quantum mechanics?
>

Yes, because it is the only probability relevant to beings like us when
they make observations while embedded in the many worlds.


-- 
Stathis Papaioannou

-- 
You received this message because you are subscribed to the Google Groups 
"Everything List" group.
To unsubscribe from this group and stop receiving emails from it, send an email 
to everything-list+unsubscr...@googlegroups.com.
To view this discussion on the web visit 
https://groups.google.com/d/msgid/everything-list/CAH%3D2ypVVo%3DE8Q1ypJ-JQdohXBkOpe24SmKH7gDEpVB6q6U%3D05A%40mail.gmail.com.


Re: Irreducible randomness in QM

2020-12-25 Thread Bruce Kellett
On Sat, Dec 26, 2020 at 2:08 PM Stathis Papaioannou 
wrote:

> On Sat, 26 Dec 2020 at 13:01, Bruce Kellett  wrote:
>
>> On Sat, Dec 26, 2020 at 12:32 PM Stathis Papaioannou 
>> wrote:
>>
>>> On Sat, 26 Dec 2020 at 09:02, Bruce Kellett 
>>> wrote:
>>>
 On Fri, Dec 25, 2020 at 11:29 PM Stathis Papaioannou <
 stath...@gmail.com> wrote:

> On Fri, 25 Dec 2020 at 22:17, Bruce Kellett 
> wrote:
>
>>
>> It can be a metaphysical truth without there being any dualist
>> underpinnings. The problem, as you point out, is when there are multiple
>> copies of you extant at a single time. If you consider yourself to be a
>> random selection from this reference class, then you have made the 
>> dualist
>> assumption that there is something that picks you out -- something that
>> distinguishes you from all the other copies. Whereas, in reality, all the
>> copies are the same and must think the same: they can deduce that they 
>> are
>> not special, and that the probability for the existence of each copy is
>> exactly one -- the Born probabilities have no bearing on their existence
>> because they inevitably exist regardless of the magnitude of the
>> mod-squared quantum amplitude. There is no process that selects just one
>> individual copy at random from a distribution, whether it be the uniform
>> distribution over branches, or the probability distribution obtained by
>> Born weighting each branch.
>>
>
> I know that all the other copies feel as I do, that they are the
> unique continuation of the original. For an entity that feels this way, 
> the
> Born probabilities apply.
>

 The Born probabilities apply because a single-world model has been
 adopted by default. You cannot prove that there are any intrinsic
 probabilities in a deterministic branching world.

>>>
>>> People in a deterministic branching world will say “I tossed a coin many
>>> times and about half the time it came up heads”. They will then assign a
>>> probability of 1/2 to heads coming up on the next toss. I don’t know if you
>>> would call this an “intrinsic probability”, but that is what would happen.
>>>
>>
>> That is probability coming from ignorance of initial conditions. Sure,
>> that applies in a deterministic world too. But the intrinsic probability of
>> interest is specifically quantum: the probability that a radioactive
>> nucleus will decay within the next specific time period; or the probability
>> that you will get spin-up on a S-G measurement of a left-spin atom. This is
>> the probability of the Born rule, and that cannot be derived in a
>> deterministic theory.
>>
>
> Well, what you call the “probability coming from ignorance of initial
> conditions” is the probability that everyone is interested in when it comes
> to making decisions about their everyday lives.
>


Is that relevant to a discussion of probability in quantum mechanics?

Bruce

-- 
You received this message because you are subscribed to the Google Groups 
"Everything List" group.
To unsubscribe from this group and stop receiving emails from it, send an email 
to everything-list+unsubscr...@googlegroups.com.
To view this discussion on the web visit 
https://groups.google.com/d/msgid/everything-list/CAFxXSLS5t-gaxhV%3Dovq6FmHuck-HN_-2_yM%3DvQjGBfOsHte0iQ%40mail.gmail.com.


Re: Irreducible randomness in QM

2020-12-25 Thread Stathis Papaioannou
On Sat, 26 Dec 2020 at 13:01, Bruce Kellett  wrote:

> On Sat, Dec 26, 2020 at 12:32 PM Stathis Papaioannou 
> wrote:
>
>> On Sat, 26 Dec 2020 at 09:02, Bruce Kellett 
>> wrote:
>>
>>> On Fri, Dec 25, 2020 at 11:29 PM Stathis Papaioannou 
>>> wrote:
>>>
 On Fri, 25 Dec 2020 at 22:17, Bruce Kellett 
 wrote:

>
> It can be a metaphysical truth without there being any dualist
> underpinnings. The problem, as you point out, is when there are multiple
> copies of you extant at a single time. If you consider yourself to be a
> random selection from this reference class, then you have made the dualist
> assumption that there is something that picks you out -- something that
> distinguishes you from all the other copies. Whereas, in reality, all the
> copies are the same and must think the same: they can deduce that they are
> not special, and that the probability for the existence of each copy is
> exactly one -- the Born probabilities have no bearing on their existence
> because they inevitably exist regardless of the magnitude of the
> mod-squared quantum amplitude. There is no process that selects just one
> individual copy at random from a distribution, whether it be the uniform
> distribution over branches, or the probability distribution obtained by
> Born weighting each branch.
>

 I know that all the other copies feel as I do, that they are the unique
 continuation of the original. For an entity that feels this way, the Born
 probabilities apply.

>>>
>>> The Born probabilities apply because a single-world model has been
>>> adopted by default. You cannot prove that there are any intrinsic
>>> probabilities in a deterministic branching world.
>>>
>>
>> People in a deterministic branching world will say “I tossed a coin many
>> times and about half the time it came up heads”. They will then assign a
>> probability of 1/2 to heads coming up on the next toss. I don’t know if you
>> would call this an “intrinsic probability”, but that is what would happen.
>>
>
> That is probability coming from ignorance of initial conditions. Sure,
> that applies in a deterministic world too. But the intrinsic probability of
> interest is specifically quantum: the probability that a radioactive
> nucleus will decay within the next specific time period; or the probability
> that you will get spin-up on a S-G measurement of a left-spin atom. This is
> the probability of the Born rule, and that cannot be derived in a
> deterministic theory.
>

Well, what you call the “probability coming from ignorance of initial
conditions” is the probability that everyone is interested in when it comes
to making decisions about their everyday lives.

> --
Stathis Papaioannou

-- 
You received this message because you are subscribed to the Google Groups 
"Everything List" group.
To unsubscribe from this group and stop receiving emails from it, send an email 
to everything-list+unsubscr...@googlegroups.com.
To view this discussion on the web visit 
https://groups.google.com/d/msgid/everything-list/CAH%3D2ypULAvLHy1agF0yO7PPSGN9y39Brw580QfKQJ8UnqRcG4g%40mail.gmail.com.


Re: Irreducible randomness in QM

2020-12-25 Thread Bruce Kellett
On Sat, Dec 26, 2020 at 12:32 PM Stathis Papaioannou 
wrote:

> On Sat, 26 Dec 2020 at 09:02, Bruce Kellett  wrote:
>
>> On Fri, Dec 25, 2020 at 11:29 PM Stathis Papaioannou 
>> wrote:
>>
>>> On Fri, 25 Dec 2020 at 22:17, Bruce Kellett 
>>> wrote:
>>>

 It can be a metaphysical truth without there being any dualist
 underpinnings. The problem, as you point out, is when there are multiple
 copies of you extant at a single time. If you consider yourself to be a
 random selection from this reference class, then you have made the dualist
 assumption that there is something that picks you out -- something that
 distinguishes you from all the other copies. Whereas, in reality, all the
 copies are the same and must think the same: they can deduce that they are
 not special, and that the probability for the existence of each copy is
 exactly one -- the Born probabilities have no bearing on their existence
 because they inevitably exist regardless of the magnitude of the
 mod-squared quantum amplitude. There is no process that selects just one
 individual copy at random from a distribution, whether it be the uniform
 distribution over branches, or the probability distribution obtained by
 Born weighting each branch.

>>>
>>> I know that all the other copies feel as I do, that they are the unique
>>> continuation of the original. For an entity that feels this way, the Born
>>> probabilities apply.
>>>
>>
>> The Born probabilities apply because a single-world model has been
>> adopted by default. You cannot prove that there are any intrinsic
>> probabilities in a deterministic branching world.
>>
>
> People in a deterministic branching world will say “I tossed a coin many
> times and about half the time it came up heads”. They will then assign a
> probability of 1/2 to heads coming up on the next toss. I don’t know if you
> would call this an “intrinsic probability”, but that is what would happen.
>

That is probability coming from ignorance of initial conditions. Sure, that
applies in a deterministic world too. But the intrinsic probability of
interest is specifically quantum: the probability that a radioactive
nucleus will decay within the next specific time period; or the probability
that you will get spin-up on a S-G measurement of a left-spin atom. This is
the probability of the Born rule, and that cannot be derived in a
deterministic theory.

Bruce

-- 
You received this message because you are subscribed to the Google Groups 
"Everything List" group.
To unsubscribe from this group and stop receiving emails from it, send an email 
to everything-list+unsubscr...@googlegroups.com.
To view this discussion on the web visit 
https://groups.google.com/d/msgid/everything-list/CAFxXSLTT%3De0nwtGtQT4BeOVemRzaqmBLhoOk736ssVGvwj%3DNsQ%40mail.gmail.com.


Re: Irreducible randomness in QM

2020-12-25 Thread Stathis Papaioannou
On Sat, 26 Dec 2020 at 09:02, Bruce Kellett  wrote:

> On Fri, Dec 25, 2020 at 11:29 PM Stathis Papaioannou 
> wrote:
>
>> On Fri, 25 Dec 2020 at 22:17, Bruce Kellett 
>> wrote:
>>
>>>
>>> It can be a metaphysical truth without there being any dualist
>>> underpinnings. The problem, as you point out, is when there are multiple
>>> copies of you extant at a single time. If you consider yourself to be a
>>> random selection from this reference class, then you have made the dualist
>>> assumption that there is something that picks you out -- something that
>>> distinguishes you from all the other copies. Whereas, in reality, all the
>>> copies are the same and must think the same: they can deduce that they are
>>> not special, and that the probability for the existence of each copy is
>>> exactly one -- the Born probabilities have no bearing on their existence
>>> because they inevitably exist regardless of the magnitude of the
>>> mod-squared quantum amplitude. There is no process that selects just one
>>> individual copy at random from a distribution, whether it be the uniform
>>> distribution over branches, or the probability distribution obtained by
>>> Born weighting each branch.
>>>
>>
>> I know that all the other copies feel as I do, that they are the unique
>> continuation of the original. For an entity that feels this way, the Born
>> probabilities apply.
>>
>
> The Born probabilities apply because a single-world model has been adopted
> by default. You cannot prove that there are any intrinsic probabilities in
> a deterministic branching world.
>

People in a deterministic branching world will say “I tossed a coin many
times and about half the time it came up heads”. They will then assign a
probability of 1/2 to heads coming up on the next toss. I don’t know if you
would call this an “intrinsic probability”, but that is what would happen.

> --
Stathis Papaioannou

-- 
You received this message because you are subscribed to the Google Groups 
"Everything List" group.
To unsubscribe from this group and stop receiving emails from it, send an email 
to everything-list+unsubscr...@googlegroups.com.
To view this discussion on the web visit 
https://groups.google.com/d/msgid/everything-list/CAH%3D2ypXzYj6tDsjExGhL9r3VXWf1j8Ap8ZsqiJJ%2B2WLU0W%2Bajw%40mail.gmail.com.


Re: Irreducible randomness in QM

2020-12-25 Thread Bruce Kellett
On Fri, Dec 25, 2020 at 11:29 PM Stathis Papaioannou 
wrote:

> On Fri, 25 Dec 2020 at 22:17, Bruce Kellett  wrote:
>
>>
>> It can be a metaphysical truth without there being any dualist
>> underpinnings. The problem, as you point out, is when there are multiple
>> copies of you extant at a single time. If you consider yourself to be a
>> random selection from this reference class, then you have made the dualist
>> assumption that there is something that picks you out -- something that
>> distinguishes you from all the other copies. Whereas, in reality, all the
>> copies are the same and must think the same: they can deduce that they are
>> not special, and that the probability for the existence of each copy is
>> exactly one -- the Born probabilities have no bearing on their existence
>> because they inevitably exist regardless of the magnitude of the
>> mod-squared quantum amplitude. There is no process that selects just one
>> individual copy at random from a distribution, whether it be the uniform
>> distribution over branches, or the probability distribution obtained by
>> Born weighting each branch.
>>
>
> I know that all the other copies feel as I do, that they are the unique
> continuation of the original. For an entity that feels this way, the Born
> probabilities apply.
>

The Born probabilities apply because a single-world model has been adopted
by default. You cannot prove that there are any intrinsic probabilities in
a deterministic branching world.

Bruce


Call it delusional, call it dualist, but it’s the way everyone’s mind works.
>
>> --
> Stathis Papaioannou
>

-- 
You received this message because you are subscribed to the Google Groups 
"Everything List" group.
To unsubscribe from this group and stop receiving emails from it, send an email 
to everything-list+unsubscr...@googlegroups.com.
To view this discussion on the web visit 
https://groups.google.com/d/msgid/everything-list/CAFxXSLQOe%3DLEq5updB5FBrtkUV%2B4ASG9B%2BOUV5%2B7d7PUKGmyog%40mail.gmail.com.


Re: Irreducible randomness in QM

2020-12-25 Thread 'Brent Meeker' via Everything List



On 12/25/2020 3:17 AM, Bruce Kellett wrote:
On Fri, Dec 25, 2020 at 9:41 PM Stathis Papaioannou 
mailto:stath...@gmail.com>> wrote:


On Fri, 25 Dec 2020 at 21:32, Bruce Kellett mailto:bhkellet...@gmail.com>> wrote:

On Fri, Dec 25, 2020 at 6:27 PM Stathis Papaioannou
mailto:stath...@gmail.com>> wrote:

On Fri, 25 Dec 2020 at 12:29, Bruce Kellett
mailto:bhkellet...@gmail.com>> wrote:

On Fri, Dec 25, 2020 at 12:13 PM Stathis Papaioannou
mailto:stath...@gmail.com>> wrote:


One is the probability that a certain branch
exists, the other is the subjective probability
that a being with the feeling that he is a unique
individual persisting through time will experience
a particular branch.



According to the individual on the branch, the Born
probability is the probability that that branch will
exist -- it is an objective property of the branch. It
is a subjective probability only to the extent to
which the individual believes in Lewis's Principal
Principle!


The probability that the branch exists under MWI, as you
have rightly pointed out, is 1. The probability that an
entity that can randomly land in any branch lands in one
particular branch is given by the Born rule.


In other words, you have a dualist interpretation of
personhood. There is an exact copy of 'you' on every branch.
'You' do not randomly land on any branch unless there is a
unique 'you' specified in some dualst manner.


Like everyone, I feel that I am a unique individual persisting
through time, which does not cause conceptual problems if there is
only one extant version of me at a time, but does if there are
multiple versions. I know that this feeling I have is just a
contingent fact about human psychology. If I were a dualist, I
would believe that it was some sort of metaphysical truth.


It can be a metaphysical truth without there being any dualist 
underpinnings. The problem, as you point out, is when there are 
multiple copies of you extant at a single time. If you consider 
yourself to be a random selection from this reference class, then you 
have made the dualist assumption that there is something that picks 
you out -- something that distinguishes you from all the other copies. 
Whereas, in reality, all the copies are the same and must think the same:


If they were really the same they wouldn't be distinct, by Leibniz's 
principle of the identity of indiscernibles.  The MWI hypothesis is that 
the copies are different if they have perceived a different result.  
This would deny the possibility of branch counting.  Two copies who saw 
the same result wouldn't really be two; they'd only be one.  Decoherence 
might be a way out of this since what appears as "the same result" to 
physically distinct persons can be different in random ways in it's 
interactions at the micro level.  But then this brings back randomness, 
which MWI seeks to avoid.


Brent

they can deduce that they are not special, and that the probability 
for the existence of each copy is exactly one -- the Born 
probabilities have no bearing on their existence because they 
inevitably exist regardless of the magnitude of the mod-squared 
quantum amplitude. There is no process that selects just one 
individual copy at random from a distribution, whether it be the 
uniform distribution over branches, or the probability distribution 
obtained by Born weighting each branch.


Bruce
--
You received this message because you are subscribed to the Google 
Groups "Everything List" group.
To unsubscribe from this group and stop receiving emails from it, send 
an email to everything-list+unsubscr...@googlegroups.com 
.
To view this discussion on the web visit 
https://groups.google.com/d/msgid/everything-list/CAFxXSLTjyhp0RnK-44jVvwKEeqpwhbSqg3mqBLCiS4pHUCGUQw%40mail.gmail.com 
.


--
You received this message because you are subscribed to the Google Groups 
"Everything List" group.
To unsubscribe from this group and stop receiving emails from it, send an email 
to everything-list+unsubscr...@googlegroups.com.
To view this discussion on the web visit 
https://groups.google.com/d/msgid/everything-list/c75ee5ce-108b-bfc2-69b1-cc3a7cedf4dc%40verizon.net.


Re: Irreducible randomness in QM

2020-12-25 Thread John Clark
On Thu, Dec 24, 2020 at 10:55 AM  wrote:

*> There are three possible ways out this problem of course. One is to say
> these string must be finite and thus the set of them enumerable. The other
> is to abandon the axiom of choice which permits the ordering I mention
> above. The third would be to abandon MWI. This looks to be an informal
> demonstration that MWI + QM(with Born rule) is not consistent with there
> being an infinite set of possible outcomes. *


There may be a fourth way. I think you're assuming that in Many Worlds only
one observer sees one precise quantum event, but when you observe an
electron do one particular thing there may be an infinite number of other
Lawrence Crowells that live in a universe where the electron does something
very slightly different, but the difference is so Infinitesimally tiny that
no conscious observer could possibly tell the difference, so there is no
difference between those infinite number of conscious beings that call
themselves Lawrence Crowell either.

Also, it seems that when the axiom of choice is allowed you can
mathematically prove all sorts of things that seem very unphysical, like
the Banach-Tarski paradox and many others. It makes life easier for
mathematicians who like to prove things but can you think of an example of
a physicist, and not a pure mathematician, that found the axiom of choice
to be a useful tool in predicting the outcome of an experiment? Mathematics
is the language of physics but like any language it can be used to write
fiction as well as nonfiction, physics is only concerned with the
nonfiction part.

John K Clark

-- 
You received this message because you are subscribed to the Google Groups 
"Everything List" group.
To unsubscribe from this group and stop receiving emails from it, send an email 
to everything-list+unsubscr...@googlegroups.com.
To view this discussion on the web visit 
https://groups.google.com/d/msgid/everything-list/CAJPayv1OSLSy1kDy4enym5oLDLn9U6UT5mPXkYdJLosHX93pcw%40mail.gmail.com.


Re: Irreducible randomness in QM

2020-12-25 Thread John Clark
On Thu, Dec 24, 2020 at 11:37 PM 'Brent Meeker' via Everything List <
everything-list@googlegroups.com> wrote:

* >That the Born rule doesn't derive from the Schroedinger equation doesn't
> bother me. *
>

We don't need to derive the Born Rule from the Schroedinger equation or
from anything else to know it's true because we already know from
experimentation that it's true. But Copenhagen, Many Worlds, Pilot Wave and
every other quantum interpretation needs to derive the Born Rule to prove
that it has the right interpretation. If Many Worlds hasn't managed to
achieve that high goal it has at least come closer to doing so than any
other interpretation, at least so far.

* > Gleason's theorem guarantees it's the only consistent probability
> measure in the eigenstates of an observable.  The question seems to be why
> is there probability at all. *
>

Yes!


> * > But I see the unpredictability of measured values and that intrinsic
> randomness is necessary to protect relativity theory.*
>

I see it is simply a result of the fact that you can't be sure where you
are until you open your eyes and look, aka make an observation. If I use
Bruno's patented You Duplicating Machine then even after Brent Meeker has
been duplicated "You" will not know if "you" are in Moscow or Washington
until "you" open the door of the duplicating chamber and look out; until
that point "you" could be said to exist in both cities, or in neither city,
or in whatever place you happen to be thinking about because contrary to
what generations have been taught in grammar school "you" Is not a pronoun,
"you"  is an adjective; you are the way matter behaves when it is organized
in a Brentmeekerian way. And until the instant the doors are open and the 2
see 2 different things those 2 chunks of matter have the same memories and
behave exactly the same way.

John K Clark

-- 
You received this message because you are subscribed to the Google Groups 
"Everything List" group.
To unsubscribe from this group and stop receiving emails from it, send an email 
to everything-list+unsubscr...@googlegroups.com.
To view this discussion on the web visit 
https://groups.google.com/d/msgid/everything-list/CAJPayv1RjUYHu5qh0sNNfMKDu48koL5mGf9rCkfWCqf8txu%3Daw%40mail.gmail.com.


Re: Irreducible randomness in QM

2020-12-25 Thread Stathis Papaioannou
On Fri, 25 Dec 2020 at 22:17, Bruce Kellett  wrote:

> On Fri, Dec 25, 2020 at 9:41 PM Stathis Papaioannou 
> wrote:
>
>> On Fri, 25 Dec 2020 at 21:32, Bruce Kellett 
>> wrote:
>>
>>> On Fri, Dec 25, 2020 at 6:27 PM Stathis Papaioannou 
>>> wrote:
>>>
 On Fri, 25 Dec 2020 at 12:29, Bruce Kellett 
 wrote:

> On Fri, Dec 25, 2020 at 12:13 PM Stathis Papaioannou <
> stath...@gmail.com> wrote:
>
>>
>> One is the probability that a certain branch exists, the other is the
>> subjective probability that a being with the feeling that he is a unique
>> individual persisting through time will experience a particular branch.
>>
>
>
> According to the individual on the branch, the Born probability is the
> probability that that branch will exist -- it is an objective property of
> the branch. It is a subjective probability only to the extent to which the
> individual believes in Lewis's Principal Principle!
>

 The probability that the branch exists under MWI, as you have rightly
 pointed out, is 1. The probability that an entity that can randomly land in
 any branch lands in one particular branch is given by the Born rule.

>>>
>>> In other words, you have a dualist interpretation of personhood. There
>>> is an exact copy of 'you' on every branch. 'You' do not randomly land on
>>> any branch unless there is a unique 'you' specified in some dualst manner.
>>>
>>
>> Like everyone, I feel that I am a unique individual persisting through
>> time, which does not cause conceptual problems if there is only one extant
>> version of me at a time, but does if there are multiple versions. I know
>> that this feeling I have is just a contingent fact about human psychology.
>> If I were a dualist, I would believe that it was some sort of metaphysical
>> truth.
>>
>
> It can be a metaphysical truth without there being any dualist
> underpinnings. The problem, as you point out, is when there are multiple
> copies of you extant at a single time. If you consider yourself to be a
> random selection from this reference class, then you have made the dualist
> assumption that there is something that picks you out -- something that
> distinguishes you from all the other copies. Whereas, in reality, all the
> copies are the same and must think the same: they can deduce that they are
> not special, and that the probability for the existence of each copy is
> exactly one -- the Born probabilities have no bearing on their existence
> because they inevitably exist regardless of the magnitude of the
> mod-squared quantum amplitude. There is no process that selects just one
> individual copy at random from a distribution, whether it be the uniform
> distribution over branches, or the probability distribution obtained by
> Born weighting each branch.
>

I know that all the other copies feel as I do, that they are the unique
continuation of the original. For an entity that feels this way, the Born
probabilities apply. Call it delusional, call it dualist, but it’s the way
everyone’s mind works.

> --
Stathis Papaioannou

-- 
You received this message because you are subscribed to the Google Groups 
"Everything List" group.
To unsubscribe from this group and stop receiving emails from it, send an email 
to everything-list+unsubscr...@googlegroups.com.
To view this discussion on the web visit 
https://groups.google.com/d/msgid/everything-list/CAH%3D2ypWmX-d4N6vBKi6J5amtyb2o8F3sZYwfKJokosDZZ9keiA%40mail.gmail.com.


Re: Irreducible randomness in QM

2020-12-25 Thread Bruce Kellett
On Fri, Dec 25, 2020 at 9:41 PM Stathis Papaioannou 
wrote:

> On Fri, 25 Dec 2020 at 21:32, Bruce Kellett  wrote:
>
>> On Fri, Dec 25, 2020 at 6:27 PM Stathis Papaioannou 
>> wrote:
>>
>>> On Fri, 25 Dec 2020 at 12:29, Bruce Kellett 
>>> wrote:
>>>
 On Fri, Dec 25, 2020 at 12:13 PM Stathis Papaioannou <
 stath...@gmail.com> wrote:

>
> One is the probability that a certain branch exists, the other is the
> subjective probability that a being with the feeling that he is a unique
> individual persisting through time will experience a particular branch.
>


 According to the individual on the branch, the Born probability is the
 probability that that branch will exist -- it is an objective property of
 the branch. It is a subjective probability only to the extent to which the
 individual believes in Lewis's Principal Principle!

>>>
>>> The probability that the branch exists under MWI, as you have rightly
>>> pointed out, is 1. The probability that an entity that can randomly land in
>>> any branch lands in one particular branch is given by the Born rule.
>>>
>>
>> In other words, you have a dualist interpretation of personhood. There is
>> an exact copy of 'you' on every branch. 'You' do not randomly land on any
>> branch unless there is a unique 'you' specified in some dualst manner.
>>
>
> Like everyone, I feel that I am a unique individual persisting through
> time, which does not cause conceptual problems if there is only one extant
> version of me at a time, but does if there are multiple versions. I know
> that this feeling I have is just a contingent fact about human psychology.
> If I were a dualist, I would believe that it was some sort of metaphysical
> truth.
>

It can be a metaphysical truth without there being any dualist
underpinnings. The problem, as you point out, is when there are multiple
copies of you extant at a single time. If you consider yourself to be a
random selection from this reference class, then you have made the dualist
assumption that there is something that picks you out -- something that
distinguishes you from all the other copies. Whereas, in reality, all the
copies are the same and must think the same: they can deduce that they are
not special, and that the probability for the existence of each copy is
exactly one -- the Born probabilities have no bearing on their existence
because they inevitably exist regardless of the magnitude of the
mod-squared quantum amplitude. There is no process that selects just one
individual copy at random from a distribution, whether it be the uniform
distribution over branches, or the probability distribution obtained by
Born weighting each branch.

Bruce

-- 
You received this message because you are subscribed to the Google Groups 
"Everything List" group.
To unsubscribe from this group and stop receiving emails from it, send an email 
to everything-list+unsubscr...@googlegroups.com.
To view this discussion on the web visit 
https://groups.google.com/d/msgid/everything-list/CAFxXSLTjyhp0RnK-44jVvwKEeqpwhbSqg3mqBLCiS4pHUCGUQw%40mail.gmail.com.


Re: Irreducible randomness in QM

2020-12-25 Thread Stathis Papaioannou
On Fri, 25 Dec 2020 at 21:32, Bruce Kellett  wrote:

> On Fri, Dec 25, 2020 at 6:27 PM Stathis Papaioannou 
> wrote:
>
>> On Fri, 25 Dec 2020 at 12:29, Bruce Kellett 
>> wrote:
>>
>>> On Fri, Dec 25, 2020 at 12:13 PM Stathis Papaioannou 
>>> wrote:
>>>

 One is the probability that a certain branch exists, the other is the
 subjective probability that a being with the feeling that he is a unique
 individual persisting through time will experience a particular branch.

>>>
>>>
>>> According to the individual on the branch, the Born probability is the
>>> probability that that branch will exist -- it is an objective property of
>>> the branch. It is a subjective probability only to the extent to which the
>>> individual believes in Lewis's Principal Principle!
>>>
>>
>> The probability that the branch exists under MWI, as you have rightly
>> pointed out, is 1. The probability that an entity that can randomly land in
>> any branch lands in one particular branch is given by the Born rule.
>>
>
> In other words, you have a dualist interpretation of personhood. There is
> an exact copy of 'you' on every branch. 'You' do not randomly land on any
> branch unless there is a unique 'you' specified in some dualst manner.
>

Like everyone, I feel that I am a unique individual persisting through
time, which does not cause conceptual problems if there is only one extant
version of me at a time, but does if there are multiple versions. I know
that this feeling I have is just a contingent fact about human psychology.
If I were a dualist, I would believe that it was some sort of metaphysical
truth.

> --
Stathis Papaioannou

-- 
You received this message because you are subscribed to the Google Groups 
"Everything List" group.
To unsubscribe from this group and stop receiving emails from it, send an email 
to everything-list+unsubscr...@googlegroups.com.
To view this discussion on the web visit 
https://groups.google.com/d/msgid/everything-list/CAH%3D2ypWos%2BkTe4%2Ba5VA8hH57b9zBSL6JtWsq0b7yUw8mJiRkQg%40mail.gmail.com.


Re: Irreducible randomness in QM

2020-12-25 Thread Bruce Kellett
On Fri, Dec 25, 2020 at 6:27 PM Stathis Papaioannou 
wrote:

> On Fri, 25 Dec 2020 at 12:29, Bruce Kellett  wrote:
>
>> On Fri, Dec 25, 2020 at 12:13 PM Stathis Papaioannou 
>> wrote:
>>
>>>
>>> One is the probability that a certain branch exists, the other is the
>>> subjective probability that a being with the feeling that he is a unique
>>> individual persisting through time will experience a particular branch.
>>>
>>
>>
>> According to the individual on the branch, the Born probability is the
>> probability that that branch will exist -- it is an objective property of
>> the branch. It is a subjective probability only to the extent to which the
>> individual believes in Lewis's Principal Principle!
>>
>
> The probability that the branch exists under MWI, as you have rightly
> pointed out, is 1. The probability that an entity that can randomly land in
> any branch lands in one particular branch is given by the Born rule.
>

In other words, you have a dualist interpretation of personhood. There is
an exact copy of 'you' on every branch. 'You' do not randomly land on any
branch unless there is a unique 'you' specified in some dualst manner.

Bruce

-- 
You received this message because you are subscribed to the Google Groups 
"Everything List" group.
To unsubscribe from this group and stop receiving emails from it, send an email 
to everything-list+unsubscr...@googlegroups.com.
To view this discussion on the web visit 
https://groups.google.com/d/msgid/everything-list/CAFxXSLT%2BG7h%2BsyKPc_Kg9EwWRd7f4E7jNcube4NmJC7JLuZMyw%40mail.gmail.com.


Re: Irreducible randomness in QM

2020-12-25 Thread Alan Grayson

*FY interest and information, the two prior links you supplied are within 
my pay grade and have high priority on my reading to-do list. The same 
hopefully applies to your links below. Also, of course, I will be studying 
the no-go theorems suggested by Bruce. TY, AG*
On Friday, December 25, 2020 at 12:54:58 AM UTC-7 sce...@libero.it wrote:

>
> Il 25/12/2020 07:29 Alan Grayson  ha scritto: 
>
>
> *Now I raise a similar question I posed to Bruce, thrice, with no replies. 
> Why does the unpredictability of measured values and the intrinsic 
> randomness protect relativity theory? This is really a huge conceptual 
> leap. How would you argue for that conclusion, as distinguished from 
> asserting it? TIA, AG* 
>
>
> there is some literature, under the name of 'uncontrollable signaling' 
> (Abner Shimony)
>
> https://arxiv.org/abs/0912.0177
>
> https://arxiv.org/abs/0808.2178
>
> https://arxiv.org/abs/1010.3714
>
> https://arxiv.org/abs/1402.0351
>
> https://aapt.scitation.org/doi/abs/10.1119/1.15059
>

-- 
You received this message because you are subscribed to the Google Groups 
"Everything List" group.
To unsubscribe from this group and stop receiving emails from it, send an email 
to everything-list+unsubscr...@googlegroups.com.
To view this discussion on the web visit 
https://groups.google.com/d/msgid/everything-list/4607c275-d363-4961-a477-b253c67e0fb9n%40googlegroups.com.


Re: Irreducible randomness in QM

2020-12-25 Thread Alan Grayson
*In fact, if you could set your arrogance and ego aside for a moment, you'd 
see that I am raising a serious issue, albeit INDIRECTLY. Even if no FTL 
signaling is possible in QM according to the no-signaling theorems (and 
thereby saving relativity), this is "guaranteed" by assuming "intrinsic 
randomness". That is, the "guarantee" flows from that which is inherently 
unintelligible!  Why unintelligible? Why does the King have no clothes? 
Because for human beings "intelligible" anything requires some definable 
PROCESS to account for the observations. In effect, I claim that knowledge 
requires determinism, in the absence of which we find ourselves with a 
non-explanation of observations, and no way to find one.  Assuming the 
non-existence of LOCAL hidden variables (that is, affirming Bell's 
theorem), there must be NON-LOCAL hidden variables whose existence restores 
explanatory value to physical theories. So I think Bohm's project was on 
the right track, but nonetheless remains a work in progress. AG*


*As for why the Born's rule discussion is, from my pov, silly, is because 
the MWI is easily demonstrated as having no merit. So what's to be gained 
from establishing its incompatibility from an interpretation of QM that has 
zero merit? AG*
On Friday, December 25, 2020 at 12:19:56 AM UTC-7 Alan Grayson wrote:

> *Well, if what you're sure of, that is "guarantees" -- that intrinsic 
> randomness guarantees no FTL signaling -- the issue would have already been 
> settled. But apparently it isn't. Time to get off your high horse. I'd say 
> that time arrived long ago. Instead you prefer indulging silly issues like 
> Born's rule in the context of MWI. No accounting for taste. AG*
>
> On Thursday, December 24, 2020 at 11:49:42 PM UTC-7 Bruce wrote:
>
>> On Fri, Dec 25, 2020 at 5:29 PM Alan Grayson  wrote:
>>
>>>
>>> *Now I raise a similar question I posed to Bruce, thrice, with no 
>>> replies. Why does the unpredictability of measured values and the intrinsic 
>>> randomness protect relativity theory? This is really a huge conceptual 
>>> leap. How would you argue for that conclusion, as distinguished from 
>>> asserting it? TIA, AG*
>>
>>
>> You need to do some research on the no-signalling theorems. This list 
>> does not exist to answer your elementary questions.
>>
>> Bruce
>>
>

-- 
You received this message because you are subscribed to the Google Groups 
"Everything List" group.
To unsubscribe from this group and stop receiving emails from it, send an email 
to everything-list+unsubscr...@googlegroups.com.
To view this discussion on the web visit 
https://groups.google.com/d/msgid/everything-list/06101a69-5692-46ae-8081-da22c0e6d1a0n%40googlegroups.com.


Re: Irreducible randomness in QM

2020-12-24 Thread 'scerir' via Everything List

> Il 25/12/2020 07:29 Alan Grayson  ha scritto:
> 
> 
> Now I raise a similar question I posed to Bruce, thrice, with no replies. 
> Why does the unpredictability of measured values and the intrinsic randomness 
> protect relativity theory? This is really a huge conceptual leap. How would 
> you argue for that conclusion, as distinguished from asserting it? TIA, AG
> 


there is some literature, under the name of 'uncontrollable signaling' (Abner 
Shimony)

https://arxiv.org/abs/0912.0177

https://arxiv.org/abs/0808.2178

https://arxiv.org/abs/1010.3714

https://arxiv.org/abs/1402.0351

https://aapt.scitation.org/doi/abs/10.1119/1.15059

-- 
You received this message because you are subscribed to the Google Groups 
"Everything List" group.
To unsubscribe from this group and stop receiving emails from it, send an email 
to everything-list+unsubscr...@googlegroups.com.
To view this discussion on the web visit 
https://groups.google.com/d/msgid/everything-list/346816733.588828.1608882895197%40mail1.libero.it.


Re: Irreducible randomness in QM

2020-12-24 Thread Stathis Papaioannou
On Fri, 25 Dec 2020 at 12:29, Bruce Kellett  wrote:

> On Fri, Dec 25, 2020 at 12:13 PM Stathis Papaioannou 
> wrote:
>
>> On Fri, 25 Dec 2020 at 11:58, Bruce Kellett 
>> wrote:
>>
>>> On Fri, Dec 25, 2020 at 2:37 AM smitra  wrote:
>>>
 On 23-12-2020 05:17, Bruce Kellett wrote:
 > On Wed, Dec 23, 2020 at 2:53 PM smitra  wrote:
 >
 >> imposing the Born rule by fiat is not incompatible with each outcome
 >>
 >> being realized.
 >
 > It is, actually. The Born rule gives probabilities that are
 > incompatible with every outcoming occurring on every trial. If every
 > outcome always occurs you are led to statements such as "This low
 > probability outcome is certain to occur", which is nonsense. If
 > something is certain to occur, it has probability one, but the Born
 > rule never gives unit probability to a single outcome from the set.

 The probability of 1 here is what you get when you consider the entire
 multiverse, but this is not the probability that one measures in
 experiments to which the Born rule refers to. This probability is
 contained in the statistics of the outcome of many experiments, and
 this
 exists in a physical form accessible to the observer. So, if 100 spin
 measurements experiments have been done then the outcomes of these
 exist
 in each sector in the form of a physical record.

>>>
>>> Confusing how you measure the probability with how it is calculated as
>>> the mod-square of the amplitude is something of a mistake.
>>>
>>> However, the inherent contradiction cannot be pushed off into the
>>> multiverse as you seem to think. The branch we are on is part of the
>>> multiverse, and it exists with probability one. But the
>>> mod-squared amplitude of the branch is less than one, so there are two
>>> distinct calculations of the probability for the existence of our branch,
>>> and they don't agree. The same holds for every branch, so the
>>> contradiction is present in every branch of the multiverse -- not easily
>>> dispatched!
>>>
>>
>> One is the probability that a certain branch exists, the other is the
>> subjective probability that a being with the feeling that he is a unique
>> individual persisting through time will experience a particular branch.
>>
>
>
> According to the individual on the branch, the Born probability is the
> probability that that branch will exist -- it is an objective property of
> the branch. It is a subjective probability only to the extent to which the
> individual believes in Lewis's Principal Principle!
>

The probability that the branch exists under MWI, as you have rightly
pointed out, is 1. The probability that an entity that can randomly land in
any branch lands in one particular branch is given by the Born rule. Due to
the nature of their psychology, humans feel themselves to be such entities.

You may say that this feeling is based on a delusion but we still have it.
>>
>
> I have not said that the feeling of continuous existence in time is a
> delusion. It seems that you are trying to claim that our
> continued existence as persons is a delusion, but that is patent nonsense.
>

The feeling of continuous existence in time as a unique entity would
persist despite copying, where it is objectively false. Maintaining this
feeling, which I claim is impossible to shake off, results in the
subjective probabilities we have been discussing.

> --
Stathis Papaioannou

-- 
You received this message because you are subscribed to the Google Groups 
"Everything List" group.
To unsubscribe from this group and stop receiving emails from it, send an email 
to everything-list+unsubscr...@googlegroups.com.
To view this discussion on the web visit 
https://groups.google.com/d/msgid/everything-list/CAH%3D2ypXgYLm-4YswfoQHU462RbEmznRg%2BnmzgJD8XjzeCMhA1w%40mail.gmail.com.


Re: Irreducible randomness in QM

2020-12-24 Thread Alan Grayson
*Well, if what you're sure of, that is "guarantees" -- that intrinsic 
randomness guarantees no FTL signaling -- the issue would have already been 
settled. But apparently it isn't. Time to get off your high horse. I'd say 
that time arrived long ago. Instead you prefer indulging silly issues like 
Born's rule in the context of MWI. No accounting for taste. AG*

On Thursday, December 24, 2020 at 11:49:42 PM UTC-7 Bruce wrote:

> On Fri, Dec 25, 2020 at 5:29 PM Alan Grayson  wrote:
>
>>
>> *Now I raise a similar question I posed to Bruce, thrice, with no 
>> replies. Why does the unpredictability of measured values and the intrinsic 
>> randomness protect relativity theory? This is really a huge conceptual 
>> leap. How would you argue for that conclusion, as distinguished from 
>> asserting it? TIA, AG*
>
>
> You need to do some research on the no-signalling theorems. This list does 
> not exist to answer your elementary questions.
>
> Bruce
>

-- 
You received this message because you are subscribed to the Google Groups 
"Everything List" group.
To unsubscribe from this group and stop receiving emails from it, send an email 
to everything-list+unsubscr...@googlegroups.com.
To view this discussion on the web visit 
https://groups.google.com/d/msgid/everything-list/a9298a8c-c144-4f76-a193-20e8bcff4238n%40googlegroups.com.


Re: Irreducible randomness in QM

2020-12-24 Thread Bruce Kellett
On Fri, Dec 25, 2020 at 5:29 PM Alan Grayson  wrote:

>
> *Now I raise a similar question I posed to Bruce, thrice, with no replies.
> Why does the unpredictability of measured values and the intrinsic
> randomness protect relativity theory? This is really a huge conceptual
> leap. How would you argue for that conclusion, as distinguished from
> asserting it? TIA, AG*


You need to do some research on the no-signalling theorems. This list does
not exist to answer your elementary questions.

Bruce

-- 
You received this message because you are subscribed to the Google Groups 
"Everything List" group.
To unsubscribe from this group and stop receiving emails from it, send an email 
to everything-list+unsubscr...@googlegroups.com.
To view this discussion on the web visit 
https://groups.google.com/d/msgid/everything-list/CAFxXSLS2whGYSpJRZ8PRsNsJxF%3DpA57LX8o_W5yh55ZNaTEicA%40mail.gmail.com.


Re: Irreducible randomness in QM

2020-12-24 Thread Alan Grayson

*Now I raise a similar question I posed to Bruce, thrice, with no replies. 
Why does the unpredictability of measured values and the intrinsic 
randomness protect relativity theory? This is really a huge conceptual 
leap. How would you argue for that conclusion, as distinguished from 
asserting it? TIA, AG*
On Thursday, December 24, 2020 at 9:37:50 PM UTC-7 Brent wrote:

> That the Born rule doesn't derive from the Schroedinger equation doesn't 
> bother me.  Gleason's theorem guarantees it's the only consistent 
> probability measure in the eigenstates of an observable.  The question 
> seems to be why is there probability at all.  But I see the 
> unpredictability of measured values and that intrinsic randomness is 
> necessary to protect relativity theory.  So randomness per the Born rule 
> seems to be the obvious theoretical choice.  But then why would I entertain 
> the multiverse.  If I know when I've made a measurement then I know when 
> the wave funcion "collapsed".  If I don't know when I've made a 
> measurement, then I have more serious problems than just the interpretation 
> of QM.
>
> Brent
>
>
> On 12/24/2020 1:04 PM, Stathis Papaioannou wrote:
>
>
>
> On Thu, 24 Dec 2020 at 11:51, Bruce Kellett  wrote:
>
>> On Thu, Dec 24, 2020 at 5:39 AM Stathis Papaioannou  
>> wrote:
>>
>>>
>>> You suspected right, I am asking a more basic question about 
>>> self-sampling and the validity of probabilities when a version of the 
>>> observer sees all possible outcomes.
>>>
>>
>> There is a problem here -- or maybe it is just careless phrasing. You say 
>> there is a question about probabilities when a version of the observer sees 
>> all possible outcomes. The question is whether it is merely a version of 
>> the observer, a copy of the observer, or the actual observer who sees all 
>> possible outcomes? A "version" is somewhat ambiguous. Different 'versions' 
>> of an operating system, for example, differ in some way. Whereas the 
>> duplicates under consideration here are, by hypothesis, all identical 
>> copies of the original.
>>
>> As John Clark is fond of pointing out, the trouble with self-sampling 
>> from a set of identical duplicate persons is that the personal pronoun 
>> 'you' loses its unique reference. All copies have an equal claim to be 
>> identified as the original 'you', so there is a real sense in which 'you' 
>> see all outcomes, with probability one. If you attempt to single out a 
>> particular individual by some random sampling procedure, you immediately 
>> make a dualist assumption -- the selected individual is different from the 
>> rest (by virtue of a 'soul', or some such, conferred by the sampling 
>> process itself).
>>
>> Since there is a sense in which 'you' certainly see all possible 
>> outcomes, there is an immediate conflict with the Born rule, according to 
>> which different outcomes have different probabilities, and 'you' can't see 
>> more than one such outcome.
>>
>
> There is no magical “you” persisting from moment to moment even in 
> ordinary life. I now have memories of being someone yesterday, I feel that 
> the essence of that person has  been transmitted to me now. But it’s a 
> delusion, and if I were copied many times each of the copies would of 
> course have the same delusion. They can’t help it, it’s the way human 
> psychology works. So thinking about probabilities if I am copied amounts to 
> this: how should I reason about those future copies who share the delusion 
> that they are uniquely me, given that I have the delusion that I will 
> become one and only one of those copies? You propose that I drop the 
> delusion and then there is no reasoning to be done, no question of 
> probability. But to be consistent, I should then drop the delusion in a 
> single thread universe as well, and not be concerned about the outcomes fir 
> the person tomorrow who thinks that he is me just because he has memories 
> of being me.
> -- 
> Stathis Papaioannou
>
> -- 
> You received this message because you are subscribed to the Google Groups 
> "Everything List" group.
> To unsubscribe from this group and stop receiving emails from it, send an 
> email to everything-li...@googlegroups.com.
>
> To view this discussion on the web visit 
> https://groups.google.com/d/msgid/everything-list/CAH%3D2ypUQdPMnvYy8d6paJOkBt7yB2F1N8UZw0cQEOJyUy%2BeUbA%40mail.gmail.com
>  
> 
> .
>
>
>

-- 
You received this message because you are subscribed to the Google Groups 
"Everything List" group.
To unsubscribe from this group and stop receiving emails from it, send an email 
to everything-list+unsubscr...@googlegroups.com.
To view this discussion on the web visit 
https://groups.google.com/d/msgid/everything-list/7e822eca-36c2-4c9a-8593-62900ec33a69n%40googlegroups.com.


Re: Irreducible randomness in QM

2020-12-24 Thread Bruce Kellett
On Fri, Dec 25, 2020 at 3:37 PM 'Brent Meeker' via Everything List <
everything-list@googlegroups.com> wrote:

> That the Born rule doesn't derive from the Schroedinger equation doesn't
> bother me.
>

It doesn't bother me, either. If one needs probabilities, one can simply
impose the Born rule. The problem is that you can't do this consistently
when every outcome occurs on every trial -- there is no sensible
definition of probability in that case.

Gleason's theorem guarantees it's the only consistent probability measure
> in the eigenstates of an observable.  The question seems to be why is there
> probability at all.
>

That lies at the heart of the problem. You can't get probabilities out of a
deterministic theory.

Bruce

But I see the unpredictability of measured values and that intrinsic
> randomness is necessary to protect relativity theory.  So randomness per
> the Born rule seems to be the obvious theoretical choice.  But then why
> would I entertain the multiverse.  If I know when I've made a measurement
> then I know when the wave funcion "collapsed".  If I don't know when I've
> made a measurement, then I have more serious problems than just the
> interpretation of QM.
>
> Brent
>

-- 
You received this message because you are subscribed to the Google Groups 
"Everything List" group.
To unsubscribe from this group and stop receiving emails from it, send an email 
to everything-list+unsubscr...@googlegroups.com.
To view this discussion on the web visit 
https://groups.google.com/d/msgid/everything-list/CAFxXSLS_Pi%2Bn2rNKErLtZ7wWTdFmeuyDgJD7ekvXBMDkYDWpog%40mail.gmail.com.


Re: Irreducible randomness in QM

2020-12-24 Thread 'Brent Meeker' via Everything List



On 12/24/2020 1:43 PM, Stathis Papaioannou wrote:



On Fri, 25 Dec 2020 at 08:33, Bruce Kellett > wrote:


On Fri, Dec 25, 2020 at 8:04 AM Stathis Papaioannou
mailto:stath...@gmail.com>> wrote:

On Thu, 24 Dec 2020 at 11:51, Bruce Kellett
mailto:bhkellet...@gmail.com>> wrote:

On Thu, Dec 24, 2020 at 5:39 AM Stathis Papaioannou
mailto:stath...@gmail.com>> wrote:


You suspected right, I am asking a more basic question
about self-sampling and the validity of probabilities
when a version of the observer sees all possible outcomes.


There is a problem here -- or maybe it is just careless
phrasing. You say there is a question about probabilities
when a version of the observer sees all possible outcomes.
The question is whether it is merely a version of the
observer, a copy of the observer, or the actual observer
who sees all possible outcomes? A "version" is somewhat
ambiguous. Different 'versions' of an operating system,
for example, differ in some way. Whereas the duplicates
under consideration here are, by hypothesis, all identical
copies of the original.

As John Clark is fond of pointing out, the trouble with
self-sampling from a set of identical duplicate persons is
that the personal pronoun 'you' loses its unique
reference. All copies have an equal claim to be identified
as the original 'you', so there is a real sense in
which 'you' see all outcomes, with probability one. If you
attempt to single out a particular individual by some
random sampling procedure, you immediately make a dualist
assumption -- the selected individual is different from
the rest (by virtue of a 'soul', or some such, conferred
by the sampling process itself).

Since there is a sense in which 'you' certainly see all
possible outcomes, there is an immediate conflict with the
Born rule, according to which different outcomes have
different probabilities, and 'you' can't see more than one
such outcome.


There is no magical “you” persisting from moment to moment
even in ordinary life.



It's not magical -- that is what it means to be a person-- you
have physical and psychological continuity from moment to moment.
If you throw this away, it is hard to know what you are talking about.


I’m not throwing it away, but I am acknowledging that it is a 
psychological artefact, not a dualistic soul. Apply this psychological 
artefact where there is copying, and you get probabilities. The copies 
each say “I know logically that all the copies are identical, but I 
feel that I am the only real me, continuing from the original”.


Artefact implies that it is constructed.  And I agree that if I were 
creating an artificial mind to be like the human mind, one of the things 
it would include is a module that produces a simulation of itself as it 
might exist in various imagined circumstances.  This is essential to 
have foresight and rational planning.


Brent
"Math is a cybervirus that lives in human minds, evolves therein and 
reproduces itself via language."

    --- Stephen Paul King


I now have memories of being someone yesterday, I feel that
the essence of that person has  been transmitted to me now.
But it’s a delusion, and if I were copied many times each of
the copies would of course have the same delusion. They can’t
help it, it’s the way human psychology works. So thinking
about probabilities if I am copied amounts to this: how should
I reason about those future copies who share the delusion that
they are uniquely me, given that I have the delusion that I
will become one and only one of those copies? You propose that
I drop the delusion and then there is no reasoning to be done,
no question of probability. But to be consistent, I should
then drop the delusion in a single thread universe as well,
and not be concerned about the outcomes fir the person
tomorrow who thinks that he is me just because he has memories
of being me.

-- 
You received this message because you are subscribed to the Google

Groups "Everything List" group.
To unsubscribe from this group and stop receiving emails from it,
send an email to everything-list+unsubscr...@googlegroups.com
.
To view this discussion on the web visit

https://groups.google.com/d/msgid/everything-list/CAFxXSLRRao9jN9-P4cFwivZ6idUEXY8m4Dm-6QKs%3DJOMgm0R-Q%40mail.gmail.com


Re: Irreducible randomness in QM

2020-12-24 Thread 'Brent Meeker' via Everything List
That the Born rule doesn't derive from the Schroedinger equation doesn't 
bother me.  Gleason's theorem guarantees it's the only consistent 
probability measure in the eigenstates of an observable. The question 
seems to be why is there probability at all.  But I see the 
unpredictability of measured values and that intrinsic randomness is 
necessary to protect relativity theory.  So randomness per the Born rule 
seems to be the obvious theoretical choice.  But then why would I 
entertain the multiverse.  If I know when I've made a measurement then I 
know when the wave funcion "collapsed".  If I don't know when I've made 
a measurement, then I have more serious problems than just the 
interpretation of QM.


Brent

On 12/24/2020 1:04 PM, Stathis Papaioannou wrote:



On Thu, 24 Dec 2020 at 11:51, Bruce Kellett > wrote:


On Thu, Dec 24, 2020 at 5:39 AM Stathis Papaioannou
mailto:stath...@gmail.com>> wrote:


You suspected right, I am asking a more basic question about
self-sampling and the validity of probabilities when a version
of the observer sees all possible outcomes.


There is a problem here -- or maybe it is just careless phrasing.
You say there is a question about probabilities when a version of
the observer sees all possible outcomes. The question is whether
it is merely a version of the observer, a copy of the observer, or
the actual observer who sees all possible outcomes? A "version" is
somewhat ambiguous. Different 'versions' of an operating system,
for example, differ in some way. Whereas the duplicates under
consideration here are, by hypothesis, all identical copies of the
original.

As John Clark is fond of pointing out, the trouble with
self-sampling from a set of identical duplicate persons is that
the personal pronoun 'you' loses its unique reference. All copies
have an equal claim to be identified as the original 'you', so
there is a real sense in which 'you' see all outcomes,
with probability one. If you attempt to single out a particular
individual by some random sampling procedure, you immediately make
a dualist assumption -- the selected individual is different from
the rest (by virtue of a 'soul', or some such, conferred by the
sampling process itself).

Since there is a sense in which 'you' certainly see all possible
outcomes, there is an immediate conflict with the Born rule,
according to which different outcomes have different
probabilities, and 'you' can't see more than one such outcome.


There is no magical “you” persisting from moment to moment even in 
ordinary life. I now have memories of being someone yesterday, I feel 
that the essence of that person has  been transmitted to me now. But 
it’s a delusion, and if I were copied many times each of the copies 
would of course have the same delusion. They can’t help it, it’s the 
way human psychology works. So thinking about probabilities if I am 
copied amounts to this: how should I reason about those future copies 
who share the delusion that they are uniquely me, given that I have 
the delusion that I will become one and only one of those copies? You 
propose that I drop the delusion and then there is no reasoning to be 
done, no question of probability. But to be consistent, I should then 
drop the delusion in a single thread universe as well, and not be 
concerned about the outcomes fir the person tomorrow who thinks that 
he is me just because he has memories of being me.

--
Stathis Papaioannou
--
You received this message because you are subscribed to the Google 
Groups "Everything List" group.
To unsubscribe from this group and stop receiving emails from it, send 
an email to everything-list+unsubscr...@googlegroups.com 
.
To view this discussion on the web visit 
https://groups.google.com/d/msgid/everything-list/CAH%3D2ypUQdPMnvYy8d6paJOkBt7yB2F1N8UZw0cQEOJyUy%2BeUbA%40mail.gmail.com 
.


--
You received this message because you are subscribed to the Google Groups 
"Everything List" group.
To unsubscribe from this group and stop receiving emails from it, send an email 
to everything-list+unsubscr...@googlegroups.com.
To view this discussion on the web visit 
https://groups.google.com/d/msgid/everything-list/4ccd2cf4-3539-26b9-c7ad-c9dc32f19dbf%40verizon.net.


Re: Irreducible randomness in QM

2020-12-24 Thread spudboy100 via Everything List
For "physical continuity" information as a stored coherent process, may be 
provided by the study of cosmology in the interaction of photons and gravity. 
It's a long shot, especially from the vantage point of our gloomy age, but it 
may be baked-in to the cosmos. Some have advocated that quantum foam may 
"scrub" the data away, and this may be true, but I'd like to have a physics 
model for this action. 
https://turingchurch.net/the-infrared-memory-of-the-universe-hints-at-future-akashic-physics-3f9a072f0ca6So,
 in this fashion storage provides continuity. 


-Original Message-
From: Bruce Kellett 
To: Everything List 
Sent: Thu, Dec 24, 2020 8:04 pm
Subject: Re: Irreducible randomness in QM

On Fri, Dec 25, 2020 at 10:47 AM Stathis Papaioannou  wrote:

On Fri, 25 Dec 2020 at 08:55, Bruce Kellett  wrote:

It is not merely a psychological artefact -- it is a matter of physical 
continuity. Your theory of personal identity is letting you down here.

Physical continuity is neither necessary nor sufficient for continuity of 
personal identity.

 
But personal identity cannot be sensibly defined without taking account of 
physical continuity.

 


Apply this psychological artefact where there is copying, and you get 
probabilities.

I think you need a little more than this hand-waving in order to get 
probabilities. They have a physical origin and are objective, after all.

There is an objective way to calculate probabilities, but when questions like 
“what will I see tomorrow” are asked a theory of personal identity is 
introduced, which is a contingent fact about our psychology.


I think you are being seriously misled by your deficient understanding of 
personal identity.
In the MWI of QM, the linearity of the Schrodinger equation means that the 
whole person, physical and psychological, is duplicated into every branch of 
the wave function -- with probability one. Even though the Born probability for 
each branch is strictly less than one.
Bruce-- 
You received this message because you are subscribed to the Google Groups 
"Everything List" group.
To unsubscribe from this group and stop receiving emails from it, send an email 
to everything-list+unsubscr...@googlegroups.com.
To view this discussion on the web visit 
https://groups.google.com/d/msgid/everything-list/CAFxXSLT%2BxOK1zwHNWXjbvj%3D4sqcbjFeecPPpc27i1K%2BsT9mVRA%40mail.gmail.com.

-- 
You received this message because you are subscribed to the Google Groups 
"Everything List" group.
To unsubscribe from this group and stop receiving emails from it, send an email 
to everything-list+unsubscr...@googlegroups.com.
To view this discussion on the web visit 
https://groups.google.com/d/msgid/everything-list/190409769.3276460.1608861979423%40mail.yahoo.com.


Re: Irreducible randomness in QM

2020-12-24 Thread Bruce Kellett
On Fri, Dec 25, 2020 at 12:13 PM Stathis Papaioannou 
wrote:

> On Fri, 25 Dec 2020 at 11:58, Bruce Kellett  wrote:
>
>> On Fri, Dec 25, 2020 at 2:37 AM smitra  wrote:
>>
>>> On 23-12-2020 05:17, Bruce Kellett wrote:
>>> > On Wed, Dec 23, 2020 at 2:53 PM smitra  wrote:
>>> >
>>> >> imposing the Born rule by fiat is not incompatible with each outcome
>>> >>
>>> >> being realized.
>>> >
>>> > It is, actually. The Born rule gives probabilities that are
>>> > incompatible with every outcoming occurring on every trial. If every
>>> > outcome always occurs you are led to statements such as "This low
>>> > probability outcome is certain to occur", which is nonsense. If
>>> > something is certain to occur, it has probability one, but the Born
>>> > rule never gives unit probability to a single outcome from the set.
>>>
>>> The probability of 1 here is what you get when you consider the entire
>>> multiverse, but this is not the probability that one measures in
>>> experiments to which the Born rule refers to. This probability is
>>> contained in the statistics of the outcome of many experiments, and this
>>> exists in a physical form accessible to the observer. So, if 100 spin
>>> measurements experiments have been done then the outcomes of these exist
>>> in each sector in the form of a physical record.
>>>
>>
>> Confusing how you measure the probability with how it is calculated as
>> the mod-square of the amplitude is something of a mistake.
>>
>> However, the inherent contradiction cannot be pushed off into the
>> multiverse as you seem to think. The branch we are on is part of the
>> multiverse, and it exists with probability one. But the
>> mod-squared amplitude of the branch is less than one, so there are two
>> distinct calculations of the probability for the existence of our branch,
>> and they don't agree. The same holds for every branch, so the
>> contradiction is present in every branch of the multiverse -- not easily
>> dispatched!
>>
>
> One is the probability that a certain branch exists, the other is the
> subjective probability that a being with the feeling that he is a unique
> individual persisting through time will experience a particular branch.
>


According to the individual on the branch, the Born probability is the
probability that that branch will exist -- it is an objective property of
the branch. It is a subjective probability only to the extent to which the
individual believes in Lewis's Principal Principle!

You may say that this feeling is based on a delusion but we still have it.
>

I have not said that the feeling of continuous existence in time is a
delusion. It seems that you are trying to claim that our
continued existence as persons is a delusion, but that is patent nonsense.

Bruce

-- 
You received this message because you are subscribed to the Google Groups 
"Everything List" group.
To unsubscribe from this group and stop receiving emails from it, send an email 
to everything-list+unsubscr...@googlegroups.com.
To view this discussion on the web visit 
https://groups.google.com/d/msgid/everything-list/CAFxXSLREi_CNn9kTEgpZYZL5PBf%2BcdSA%3DmoZhYdiurvKgJL4kw%40mail.gmail.com.


Re: Irreducible randomness in QM

2020-12-24 Thread Stathis Papaioannou
On Fri, 25 Dec 2020 at 11:58, Bruce Kellett  wrote:

> On Fri, Dec 25, 2020 at 2:37 AM smitra  wrote:
>
>> On 23-12-2020 05:17, Bruce Kellett wrote:
>> > On Wed, Dec 23, 2020 at 2:53 PM smitra  wrote:
>> >
>> >> imposing the Born rule by fiat is not incompatible with each outcome
>> >>
>> >> being realized.
>> >
>> > It is, actually. The Born rule gives probabilities that are
>> > incompatible with every outcoming occurring on every trial. If every
>> > outcome always occurs you are led to statements such as "This low
>> > probability outcome is certain to occur", which is nonsense. If
>> > something is certain to occur, it has probability one, but the Born
>> > rule never gives unit probability to a single outcome from the set.
>>
>> The probability of 1 here is what you get when you consider the entire
>> multiverse, but this is not the probability that one measures in
>> experiments to which the Born rule refers to. This probability is
>> contained in the statistics of the outcome of many experiments, and this
>> exists in a physical form accessible to the observer. So, if 100 spin
>> measurements experiments have been done then the outcomes of these exist
>> in each sector in the form of a physical record.
>>
>
> Confusing how you measure the probability with how it is calculated as the
> mod-square of the amplitude is something of a mistake.
>
> However, the inherent contradiction cannot be pushed off into the
> multiverse as you seem to think. The branch we are on is part of the
> multiverse, and it exists with probability one. But the
> mod-squared amplitude of the branch is less than one, so there are two
> distinct calculations of the probability for the existence of our branch,
> and they don't agree. The same holds for every branch, so the
> contradiction is present in every branch of the multiverse -- not easily
> dispatched!
>

One is the probability that a certain branch exists, the other is the
subjective probability that a being with the feeling that he is a unique
individual persisting through time will experience a particular branch. You
may say that this feeling is based on a delusion but we still have it.

> --
Stathis Papaioannou

-- 
You received this message because you are subscribed to the Google Groups 
"Everything List" group.
To unsubscribe from this group and stop receiving emails from it, send an email 
to everything-list+unsubscr...@googlegroups.com.
To view this discussion on the web visit 
https://groups.google.com/d/msgid/everything-list/CAH%3D2ypVhexnDDMWoZDkXg5HB8MtF1idX6sU%2By__he0vOo%3D3GqA%40mail.gmail.com.


Re: Irreducible randomness in QM

2020-12-24 Thread Bruce Kellett
On Fri, Dec 25, 2020 at 10:47 AM Stathis Papaioannou 
wrote:

> On Fri, 25 Dec 2020 at 08:55, Bruce Kellett  wrote:
>
>>
>> It is not merely a psychological artefact -- it is a matter of physical
>> continuity. Your theory of personal identity is letting you down here.
>>
>
> Physical continuity is neither necessary nor sufficient for continuity of
> personal identity.
>



> But personal identity cannot be sensibly defined without taking account of
> physical continuity.
>



> Apply this psychological artefact where there is copying, and you get
>>> probabilities.
>>>
>>
>> I think you need a little more than this hand-waving in order to get
>> probabilities. They have a physical origin and are objective, after all.
>>
>
> There is an objective way to calculate probabilities, but when questions
> like “what will I see tomorrow” are asked a theory of personal identity is
> introduced, which is a contingent fact about our psychology.
>


I think you are being seriously misled by your deficient understanding of
personal identity.

In the MWI of QM, the linearity of the Schrodinger equation means that the
whole person, physical and psychological, is duplicated into every branch
of the wave function -- with probability one. Even though the Born
probability for each branch is strictly less than one.

Bruce

-- 
You received this message because you are subscribed to the Google Groups 
"Everything List" group.
To unsubscribe from this group and stop receiving emails from it, send an email 
to everything-list+unsubscr...@googlegroups.com.
To view this discussion on the web visit 
https://groups.google.com/d/msgid/everything-list/CAFxXSLT%2BxOK1zwHNWXjbvj%3D4sqcbjFeecPPpc27i1K%2BsT9mVRA%40mail.gmail.com.


Re: Irreducible randomness in QM

2020-12-24 Thread Bruce Kellett
On Fri, Dec 25, 2020 at 2:37 AM smitra  wrote:

> On 23-12-2020 05:17, Bruce Kellett wrote:
> > On Wed, Dec 23, 2020 at 2:53 PM smitra  wrote:
> >
> >> imposing the Born rule by fiat is not incompatible with each outcome
> >>
> >> being realized.
> >
> > It is, actually. The Born rule gives probabilities that are
> > incompatible with every outcoming occurring on every trial. If every
> > outcome always occurs you are led to statements such as "This low
> > probability outcome is certain to occur", which is nonsense. If
> > something is certain to occur, it has probability one, but the Born
> > rule never gives unit probability to a single outcome from the set.
>
> The probability of 1 here is what you get when you consider the entire
> multiverse, but this is not the probability that one measures in
> experiments to which the Born rule refers to. This probability is
> contained in the statistics of the outcome of many experiments, and this
> exists in a physical form accessible to the observer. So, if 100 spin
> measurements experiments have been done then the outcomes of these exist
> in each sector in the form of a physical record.
>

Confusing how you measure the probability with how it is calculated as the
mod-square of the amplitude is something of a mistake.

However, the inherent contradiction cannot be pushed off into the
multiverse as you seem to think. The branch we are on is part of the
multiverse, and it exists with probability one. But the
mod-squared amplitude of the branch is less than one, so there are two
distinct calculations of the probability for the existence of our branch,
and they don't agree. The same holds for every branch, so the
contradiction is present in every branch of the multiverse -- not easily
dispatched!

Bruce

-- 
You received this message because you are subscribed to the Google Groups 
"Everything List" group.
To unsubscribe from this group and stop receiving emails from it, send an email 
to everything-list+unsubscr...@googlegroups.com.
To view this discussion on the web visit 
https://groups.google.com/d/msgid/everything-list/CAFxXSLRpSbr40LHO99i_PJY0zBiunpQfEZ5mYzfpDu8eDTHECg%40mail.gmail.com.


Re: Irreducible randomness in QM

2020-12-24 Thread Stathis Papaioannou
On Fri, 25 Dec 2020 at 08:55, Bruce Kellett  wrote:

> On Fri, Dec 25, 2020 at 8:44 AM Stathis Papaioannou 
> wrote:
>
>> On Fri, 25 Dec 2020 at 08:33, Bruce Kellett 
>> wrote:
>>
>>> On Fri, Dec 25, 2020 at 8:04 AM Stathis Papaioannou 
>>> wrote:
>>>
 On Thu, 24 Dec 2020 at 11:51, Bruce Kellett 
 wrote:

> On Thu, Dec 24, 2020 at 5:39 AM Stathis Papaioannou <
> stath...@gmail.com> wrote:
>
>>
>> You suspected right, I am asking a more basic question about
>> self-sampling and the validity of probabilities when a version of the
>> observer sees all possible outcomes.
>>
>
> There is a problem here -- or maybe it is just careless phrasing. You
> say there is a question about probabilities when a version of the observer
> sees all possible outcomes. The question is whether it is merely a version
> of the observer, a copy of the observer, or the actual observer who sees
> all possible outcomes? A "version" is somewhat ambiguous. Different
> 'versions' of an operating system, for example, differ in some way. 
> Whereas
> the duplicates under consideration here are, by hypothesis, all identical
> copies of the original.
>
> As John Clark is fond of pointing out, the trouble with self-sampling
> from a set of identical duplicate persons is that the personal pronoun
> 'you' loses its unique reference. All copies have an equal claim to be
> identified as the original 'you', so there is a real sense in which 'you'
> see all outcomes, with probability one. If you attempt to single out a
> particular individual by some random sampling procedure, you immediately
> make a dualist assumption -- the selected individual is different from the
> rest (by virtue of a 'soul', or some such, conferred by the sampling
> process itself).
>
> Since there is a sense in which 'you' certainly see all possible
> outcomes, there is an immediate conflict with the Born rule, according to
> which different outcomes have different probabilities, and 'you' can't see
> more than one such outcome.
>

 There is no magical “you” persisting from moment to moment even in
 ordinary life.

>>>
>>>
>>> It's not magical -- that is what it means to be a person-- you have
>>> physical and psychological continuity from moment to moment. If you throw
>>> this away, it is hard to know what you are talking about.
>>>
>>
>> I’m not throwing it away, but I am acknowledging that it is a
>> psychological artefact, not a dualistic soul.
>>
>
>
> It is not merely a psychological artefact -- it is a matter of physical
> continuity. Your theory of personal identity is letting you down here.
>

Physical continuity is neither necessary nor sufficient for continuity of
personal identity.

Apply this psychological artefact where there is copying, and you get
>> probabilities.
>>
>
> I think you need a little more than this hand-waving in order to get
> probabilities. They have a physical origin and are objective, after all.
>

There is an objective way to calculate probabilities, but when questions
like “what will I see tomorrow” are asked a theory of personal identity is
introduced, which is a contingent fact about our psychology.

The copies each say “I know logically that all the copies are identical,
>> but I feel that I am the only real me, continuing from the original”.
>>
>> I now have memories of being someone yesterday, I feel that the essence
 of that person has  been transmitted to me now. But it’s a delusion, and if
 I were copied many times each of the copies would of course have the same
 delusion. They can’t help it, it’s the way human psychology works. So
 thinking about probabilities if I am copied amounts to this: how should I
 reason about those future copies who share the delusion that they are
 uniquely me, given that I have the delusion that I will become one and only
 one of those copies? You propose that I drop the delusion and then there is
 no reasoning to be done, no question of probability. But to be consistent,
 I should then drop the delusion in a single thread universe as well, and
 not be concerned about the outcomes fir the person tomorrow who thinks that
 he is me just because he has memories of being me.

>>> --
>>> You received this message because you are subscribed to the Google
>>> Groups "Everything List" group.
>>> To unsubscribe from this group and stop receiving emails from it, send
>>> an email to everything-list+unsubscr...@googlegroups.com.
>>> To view this discussion on the web visit
>>> https://groups.google.com/d/msgid/everything-list/CAFxXSLRRao9jN9-P4cFwivZ6idUEXY8m4Dm-6QKs%3DJOMgm0R-Q%40mail.gmail.com
>>> 
>>> .
>>>
>> --
>> Stathis Papaioannou
>>
>> --
>> 

Re: Irreducible randomness in QM

2020-12-24 Thread Bruce Kellett
On Fri, Dec 25, 2020 at 8:44 AM Stathis Papaioannou 
wrote:

> On Fri, 25 Dec 2020 at 08:33, Bruce Kellett  wrote:
>
>> On Fri, Dec 25, 2020 at 8:04 AM Stathis Papaioannou 
>> wrote:
>>
>>> On Thu, 24 Dec 2020 at 11:51, Bruce Kellett 
>>> wrote:
>>>
 On Thu, Dec 24, 2020 at 5:39 AM Stathis Papaioannou 
 wrote:

>
> You suspected right, I am asking a more basic question about
> self-sampling and the validity of probabilities when a version of the
> observer sees all possible outcomes.
>

 There is a problem here -- or maybe it is just careless phrasing. You
 say there is a question about probabilities when a version of the observer
 sees all possible outcomes. The question is whether it is merely a version
 of the observer, a copy of the observer, or the actual observer who sees
 all possible outcomes? A "version" is somewhat ambiguous. Different
 'versions' of an operating system, for example, differ in some way. Whereas
 the duplicates under consideration here are, by hypothesis, all identical
 copies of the original.

 As John Clark is fond of pointing out, the trouble with self-sampling
 from a set of identical duplicate persons is that the personal pronoun
 'you' loses its unique reference. All copies have an equal claim to be
 identified as the original 'you', so there is a real sense in which 'you'
 see all outcomes, with probability one. If you attempt to single out a
 particular individual by some random sampling procedure, you immediately
 make a dualist assumption -- the selected individual is different from the
 rest (by virtue of a 'soul', or some such, conferred by the sampling
 process itself).

 Since there is a sense in which 'you' certainly see all possible
 outcomes, there is an immediate conflict with the Born rule, according to
 which different outcomes have different probabilities, and 'you' can't see
 more than one such outcome.

>>>
>>> There is no magical “you” persisting from moment to moment even in
>>> ordinary life.
>>>
>>
>>
>> It's not magical -- that is what it means to be a person-- you have
>> physical and psychological continuity from moment to moment. If you throw
>> this away, it is hard to know what you are talking about.
>>
>
> I’m not throwing it away, but I am acknowledging that it is a
> psychological artefact, not a dualistic soul.
>


It is not merely a psychological artefact -- it is a matter of physical
continuity. Your theory of personal identity is letting you down here.


Apply this psychological artefact where there is copying, and you get
> probabilities.
>

I think you need a little more than this hand-waving in order to get
probabilities. They have a physical origin and are objective, after all.

Bruce


The copies each say “I know logically that all the copies are identical,
> but I feel that I am the only real me, continuing from the original”.
>
> I now have memories of being someone yesterday, I feel that the essence of
>>> that person has  been transmitted to me now. But it’s a delusion, and if I
>>> were copied many times each of the copies would of course have the same
>>> delusion. They can’t help it, it’s the way human psychology works. So
>>> thinking about probabilities if I am copied amounts to this: how should I
>>> reason about those future copies who share the delusion that they are
>>> uniquely me, given that I have the delusion that I will become one and only
>>> one of those copies? You propose that I drop the delusion and then there is
>>> no reasoning to be done, no question of probability. But to be consistent,
>>> I should then drop the delusion in a single thread universe as well, and
>>> not be concerned about the outcomes fir the person tomorrow who thinks that
>>> he is me just because he has memories of being me.
>>>
>> --
>> You received this message because you are subscribed to the Google Groups
>> "Everything List" group.
>> To unsubscribe from this group and stop receiving emails from it, send an
>> email to everything-list+unsubscr...@googlegroups.com.
>> To view this discussion on the web visit
>> https://groups.google.com/d/msgid/everything-list/CAFxXSLRRao9jN9-P4cFwivZ6idUEXY8m4Dm-6QKs%3DJOMgm0R-Q%40mail.gmail.com
>> 
>> .
>>
> --
> Stathis Papaioannou
>
> --
> You received this message because you are subscribed to the Google Groups
> "Everything List" group.
> To unsubscribe from this group and stop receiving emails from it, send an
> email to everything-list+unsubscr...@googlegroups.com.
> To view this discussion on the web visit
> https://groups.google.com/d/msgid/everything-list/CAH%3D2ypWfLPXRd1a-kUT4OOtgyRNVxOz6pG750yTK4wyDeostKw%40mail.gmail.com
> 

Re: Irreducible randomness in QM

2020-12-24 Thread Stathis Papaioannou
On Fri, 25 Dec 2020 at 08:33, Bruce Kellett  wrote:

> On Fri, Dec 25, 2020 at 8:04 AM Stathis Papaioannou 
> wrote:
>
>> On Thu, 24 Dec 2020 at 11:51, Bruce Kellett 
>> wrote:
>>
>>> On Thu, Dec 24, 2020 at 5:39 AM Stathis Papaioannou 
>>> wrote:
>>>

 You suspected right, I am asking a more basic question about
 self-sampling and the validity of probabilities when a version of the
 observer sees all possible outcomes.

>>>
>>> There is a problem here -- or maybe it is just careless phrasing. You
>>> say there is a question about probabilities when a version of the observer
>>> sees all possible outcomes. The question is whether it is merely a version
>>> of the observer, a copy of the observer, or the actual observer who sees
>>> all possible outcomes? A "version" is somewhat ambiguous. Different
>>> 'versions' of an operating system, for example, differ in some way. Whereas
>>> the duplicates under consideration here are, by hypothesis, all identical
>>> copies of the original.
>>>
>>> As John Clark is fond of pointing out, the trouble with self-sampling
>>> from a set of identical duplicate persons is that the personal pronoun
>>> 'you' loses its unique reference. All copies have an equal claim to be
>>> identified as the original 'you', so there is a real sense in which 'you'
>>> see all outcomes, with probability one. If you attempt to single out a
>>> particular individual by some random sampling procedure, you immediately
>>> make a dualist assumption -- the selected individual is different from the
>>> rest (by virtue of a 'soul', or some such, conferred by the sampling
>>> process itself).
>>>
>>> Since there is a sense in which 'you' certainly see all possible
>>> outcomes, there is an immediate conflict with the Born rule, according to
>>> which different outcomes have different probabilities, and 'you' can't see
>>> more than one such outcome.
>>>
>>
>> There is no magical “you” persisting from moment to moment even in
>> ordinary life.
>>
>
>
> It's not magical -- that is what it means to be a person-- you have
> physical and psychological continuity from moment to moment. If you throw
> this away, it is hard to know what you are talking about.
>

I’m not throwing it away, but I am acknowledging that it is a psychological
artefact, not a dualistic soul. Apply this psychological artefact where
there is copying, and you get probabilities. The copies each say “I know
logically that all the copies are identical, but I feel that I am the only
real me, continuing from the original”.

I now have memories of being someone yesterday, I feel that the essence of
>> that person has  been transmitted to me now. But it’s a delusion, and if I
>> were copied many times each of the copies would of course have the same
>> delusion. They can’t help it, it’s the way human psychology works. So
>> thinking about probabilities if I am copied amounts to this: how should I
>> reason about those future copies who share the delusion that they are
>> uniquely me, given that I have the delusion that I will become one and only
>> one of those copies? You propose that I drop the delusion and then there is
>> no reasoning to be done, no question of probability. But to be consistent,
>> I should then drop the delusion in a single thread universe as well, and
>> not be concerned about the outcomes fir the person tomorrow who thinks that
>> he is me just because he has memories of being me.
>>
> --
> You received this message because you are subscribed to the Google Groups
> "Everything List" group.
> To unsubscribe from this group and stop receiving emails from it, send an
> email to everything-list+unsubscr...@googlegroups.com.
> To view this discussion on the web visit
> https://groups.google.com/d/msgid/everything-list/CAFxXSLRRao9jN9-P4cFwivZ6idUEXY8m4Dm-6QKs%3DJOMgm0R-Q%40mail.gmail.com
> 
> .
>
-- 
Stathis Papaioannou

-- 
You received this message because you are subscribed to the Google Groups 
"Everything List" group.
To unsubscribe from this group and stop receiving emails from it, send an email 
to everything-list+unsubscr...@googlegroups.com.
To view this discussion on the web visit 
https://groups.google.com/d/msgid/everything-list/CAH%3D2ypWfLPXRd1a-kUT4OOtgyRNVxOz6pG750yTK4wyDeostKw%40mail.gmail.com.


Re: Irreducible randomness in QM

2020-12-24 Thread 'scerir' via Everything List
BTW at last I've found that quotation.

It seems nteresting that Schroedinger writes: 'Not only has none of us ever 
experienced more than one consciousness, but there is also no trace of 
circumstantial evidence of this ever happening anywhere in the world.' 

Unfortunately I cannot understand the following statement: 'I should say: the 
over-all number of minds is just one.'

"The reason why our sentient, percipient and thinking ego is met nowhere within 
our scientific world picture can easily be indicated in seven words: because it 
is itself that world picture. It is identical with the whole and therefore 
cannot be contained in it as a part of it. But, of course, here we knock 
against the arithmetical paradox; there appears to be a great multitude of 
these conscious egos, the world is however only one.
There is obviously only one alternative, namely the unification of minds or 
consciousnesses, Their multiplicity is only apparent, in truth there is only 
one mind. This is the doctrine of the Upanishads.
The doctrine of identity can claim that it is clinched by the empirical fact 
that consciousness is never experienced in the plural, only in the singular. 
Not only has none of us ever experienced more than one consciousness, but there 
is also no trace of circumstantial evidence of this ever happening anywhere in 
the world.
Mind is by its very nature a singulare tantum. I should say: the over-all 
number of minds is just one. I venture to call it indesctructible since it has 
a peculiar time-table, namely mind is always now. There is really no before and 
after for mind. There is only now that includes memories and expectations. But 
I grant that our language is not adequate to express this, and I also grant, 
should anyone wish to state it, that I am now talking religion, not science – a 
religion, however not opposed to science, but supported by what disinterested 
scientific research has brought to the fore."

Erwin Schrödinger, Mind and Matter

Chapter 4: The Arithmetical Paradox: The Oneness of Mind



> Il 22/12/2020 21:14 'scerir' via Everything List 
>  ha scritto:
> 
> 
> Stathis Papaioannou wrote:
> All the copies could be conscious or all could be zombies; none are 
> privileged.
> 
> "In truth there is only one mind. Oneness it is the doctrine of the 
> Upanishads." As far as I remember Schroedinger wrote something like that. 
> Does that "Oneness" could resolve our problem? :-)
> 
> 
> 
> 
> 
> --
> You received this message because you are subscribed to the Google Groups 
> "Everything List" group.
> To unsubscribe from this group and stop receiving emails from it, send an 
> email to everything-list+unsubscr...@googlegroups.com 
> mailto:everything-list+unsubscr...@googlegroups.com .
> To view this discussion on the web visit 
> https://groups.google.com/d/msgid/everything-list/1862682578.460029.1608668067995%40mail1.libero.it
>  
> https://groups.google.com/d/msgid/everything-list/1862682578.460029.1608668067995%40mail1.libero.it?utm_medium=email_source=footer
>  .
> 

-- 
You received this message because you are subscribed to the Google Groups 
"Everything List" group.
To unsubscribe from this group and stop receiving emails from it, send an email 
to everything-list+unsubscr...@googlegroups.com.
To view this discussion on the web visit 
https://groups.google.com/d/msgid/everything-list/740433897.491833.1608846077491%40mail1.libero.it.


Re: Irreducible randomness in QM

2020-12-24 Thread Bruce Kellett
On Fri, Dec 25, 2020 at 8:04 AM Stathis Papaioannou 
wrote:

> On Thu, 24 Dec 2020 at 11:51, Bruce Kellett  wrote:
>
>> On Thu, Dec 24, 2020 at 5:39 AM Stathis Papaioannou 
>> wrote:
>>
>>>
>>> You suspected right, I am asking a more basic question about
>>> self-sampling and the validity of probabilities when a version of the
>>> observer sees all possible outcomes.
>>>
>>
>> There is a problem here -- or maybe it is just careless phrasing. You say
>> there is a question about probabilities when a version of the observer sees
>> all possible outcomes. The question is whether it is merely a version of
>> the observer, a copy of the observer, or the actual observer who sees all
>> possible outcomes? A "version" is somewhat ambiguous. Different 'versions'
>> of an operating system, for example, differ in some way. Whereas the
>> duplicates under consideration here are, by hypothesis, all identical
>> copies of the original.
>>
>> As John Clark is fond of pointing out, the trouble with self-sampling
>> from a set of identical duplicate persons is that the personal pronoun
>> 'you' loses its unique reference. All copies have an equal claim to be
>> identified as the original 'you', so there is a real sense in which 'you'
>> see all outcomes, with probability one. If you attempt to single out a
>> particular individual by some random sampling procedure, you immediately
>> make a dualist assumption -- the selected individual is different from the
>> rest (by virtue of a 'soul', or some such, conferred by the sampling
>> process itself).
>>
>> Since there is a sense in which 'you' certainly see all possible
>> outcomes, there is an immediate conflict with the Born rule, according to
>> which different outcomes have different probabilities, and 'you' can't see
>> more than one such outcome.
>>
>
> There is no magical “you” persisting from moment to moment even in
> ordinary life.
>


It's not magical -- that is what it means to be a person-- you have
physical and psychological continuity from moment to moment. If you throw
this away, it is hard to know what you are talking about.


I now have memories of being someone yesterday, I feel that the essence of
> that person has  been transmitted to me now. But it’s a delusion, and if I
> were copied many times each of the copies would of course have the same
> delusion. They can’t help it, it’s the way human psychology works. So
> thinking about probabilities if I am copied amounts to this: how should I
> reason about those future copies who share the delusion that they are
> uniquely me, given that I have the delusion that I will become one and only
> one of those copies? You propose that I drop the delusion and then there is
> no reasoning to be done, no question of probability. But to be consistent,
> I should then drop the delusion in a single thread universe as well, and
> not be concerned about the outcomes fir the person tomorrow who thinks that
> he is me just because he has memories of being me.
>

-- 
You received this message because you are subscribed to the Google Groups 
"Everything List" group.
To unsubscribe from this group and stop receiving emails from it, send an email 
to everything-list+unsubscr...@googlegroups.com.
To view this discussion on the web visit 
https://groups.google.com/d/msgid/everything-list/CAFxXSLRRao9jN9-P4cFwivZ6idUEXY8m4Dm-6QKs%3DJOMgm0R-Q%40mail.gmail.com.


Re: Irreducible randomness in QM

2020-12-24 Thread Stathis Papaioannou
On Thu, 24 Dec 2020 at 11:51, Bruce Kellett  wrote:

> On Thu, Dec 24, 2020 at 5:39 AM Stathis Papaioannou 
> wrote:
>
>>
>> You suspected right, I am asking a more basic question about
>> self-sampling and the validity of probabilities when a version of the
>> observer sees all possible outcomes.
>>
>
> There is a problem here -- or maybe it is just careless phrasing. You say
> there is a question about probabilities when a version of the observer sees
> all possible outcomes. The question is whether it is merely a version of
> the observer, a copy of the observer, or the actual observer who sees all
> possible outcomes? A "version" is somewhat ambiguous. Different 'versions'
> of an operating system, for example, differ in some way. Whereas the
> duplicates under consideration here are, by hypothesis, all identical
> copies of the original.
>
> As John Clark is fond of pointing out, the trouble with self-sampling from
> a set of identical duplicate persons is that the personal pronoun 'you'
> loses its unique reference. All copies have an equal claim to be identified
> as the original 'you', so there is a real sense in which 'you' see all
> outcomes, with probability one. If you attempt to single out a particular
> individual by some random sampling procedure, you immediately make a
> dualist assumption -- the selected individual is different from the rest
> (by virtue of a 'soul', or some such, conferred by the sampling process
> itself).
>
> Since there is a sense in which 'you' certainly see all possible outcomes,
> there is an immediate conflict with the Born rule, according to which
> different outcomes have different probabilities, and 'you' can't see more
> than one such outcome.
>

There is no magical “you” persisting from moment to moment even in ordinary
life. I now have memories of being someone yesterday, I feel that the
essence of that person has  been transmitted to me now. But it’s a
delusion, and if I were copied many times each of the copies would of
course have the same delusion. They can’t help it, it’s the way human
psychology works. So thinking about probabilities if I am copied amounts to
this: how should I reason about those future copies who share the delusion
that they are uniquely me, given that I have the delusion that I will
become one and only one of those copies? You propose that I drop the
delusion and then there is no reasoning to be done, no question of
probability. But to be consistent, I should then drop the delusion in a
single thread universe as well, and not be concerned about the outcomes fir
the person tomorrow who thinks that he is me just because he has memories
of being me.

> --
Stathis Papaioannou

-- 
You received this message because you are subscribed to the Google Groups 
"Everything List" group.
To unsubscribe from this group and stop receiving emails from it, send an email 
to everything-list+unsubscr...@googlegroups.com.
To view this discussion on the web visit 
https://groups.google.com/d/msgid/everything-list/CAH%3D2ypUQdPMnvYy8d6paJOkBt7yB2F1N8UZw0cQEOJyUy%2BeUbA%40mail.gmail.com.


Re: Irreducible randomness in QM

2020-12-24 Thread Lawrence Crowell
I have recently thought of something a bit odd, which is related to MWI. If 
we consider the universe, say multiverse and well everything possible, as a 
set of of possible quantum outcomes, say Hadamard gate operations etc, and 
these may be infinite then there are two oddities. This means the universe, 
including all possible states of the universe, is a set of outcomes in some 
order, say (0100101110100110110101011...  , 1010010101001010001110101... , 
etc. then two things are evident. This is a power set and not enumerable. 
The second is there are an infinite number of such strings outside the list 
and third this leads to a paradoxical decomposition of the Bloch sphere. 
These symbol strings are the set of all possible outcomes that can exist in 
something like and MWI setting. I am also assuming, at least for the sake 
of argument, that this may be infinite.

The first two of these are not hard to see. In fact they are related. The 
Cantor diagonalization trick leads to the result there can never be an 
enumeration of this set. The third is not as easy to see. If we list these 
outcomes in some ordering, say listing those with N zeros before those with 
N-1 zeros, it is not hard to those with N-1 0s contain a reduced set 
equivalent to those with N 0s. Because of this by performing the SU(2) 
group operations on the sphere we can separate it into 5 sets, where 3 of 
these complete the sphere and the other two complete it as well. In effect 
this is the Banach-Tarski paradox. This leads to a conundrum, for this 
duplication means there is some redundancy or indefinability with 
probabilities. 

There are three possible ways out this problem of course. One is to say 
these string must be finite and thus the set of them enumerable. The other 
is to abandon the axiom of choice which permits the ordering I mention 
above. The third would be to abandon MWI. This looks to be an informal 
demonstration that MWI + QM(with Born rule) is not consistent with there 
being an infinite set of possible outcomes. This is motivated in part by 
Carroll's demonstration that MWI is consistent with Born rule by his 
throwing out symbol strings with a large number of redundant outcomes, say 
a million 1's in a row. I ponder whether the only hope for MWI with an 
infinite possible outcomes is if there is some regularization or 
renormalization of this sort that "pushes" this decomposition of the Bloch 
sphere into non-physics, much as renormalization of charge and mass does.

LC

On Wednesday, December 23, 2020 at 6:51:00 PM UTC-6 Bruce wrote:

> On Thu, Dec 24, 2020 at 5:39 AM Stathis Papaioannou  
> wrote:
>
>>
>> You suspected right, I am asking a more basic question about 
>> self-sampling and the validity of probabilities when a version of the 
>> observer sees all possible outcomes.
>>
>
> There is a problem here -- or maybe it is just careless phrasing. You say 
> there is a question about probabilities when a version of the observer sees 
> all possible outcomes. The question is whether it is merely a version of 
> the observer, a copy of the observer, or the actual observer who sees all 
> possible outcomes? A "version" is somewhat ambiguous. Different 'versions' 
> of an operating system, for example, differ in some way. Whereas the 
> duplicates under consideration here are, by hypothesis, all identical 
> copies of the original.
>
> As John Clark is fond of pointing out, the trouble with self-sampling from 
> a set of identical duplicate persons is that the personal pronoun 'you' 
> loses its unique reference. All copies have an equal claim to be identified 
> as the original 'you', so there is a real sense in which 'you' see all 
> outcomes, with probability one. If you attempt to single out a particular 
> individual by some random sampling procedure, you immediately make a 
> dualist assumption -- the selected individual is different from the rest 
> (by virtue of a 'soul', or some such, conferred by the sampling process 
> itself).
>
> Since there is a sense in which 'you' certainly see all possible outcomes, 
> there is an immediate conflict with the Born rule, according to which 
> different outcomes have different probabilities, and 'you' can't see more 
> than one such outcome.
>
> Bruce
>

-- 
You received this message because you are subscribed to the Google Groups 
"Everything List" group.
To unsubscribe from this group and stop receiving emails from it, send an email 
to everything-list+unsubscr...@googlegroups.com.
To view this discussion on the web visit 
https://groups.google.com/d/msgid/everything-list/39f6c45a-3d4c-49ef-b203-a9431ce36d0bn%40googlegroups.com.


Re: Irreducible randomness in QM

2020-12-24 Thread smitra

On 23-12-2020 05:17, Bruce Kellett wrote:

On Wed, Dec 23, 2020 at 2:53 PM smitra  wrote:


imposing the Born rule by fiat is not incompatible with each outcome

being realized.


It is, actually. The Born rule gives probabilities that are
incompatible with every outcoming occurring on every trial. If every
outcome always occurs you are led to statements such as "This low
probability outcome is certain to occur", which is nonsense. If
something is certain to occur, it has probability one, but the Born
rule never gives unit probability to a single outcome from the set.


The probability of 1 here is what you get when you consider the entire 
multiverse, but this is not the probability that one measures in 
experiments to which the Born rule refers to. This probability is 
contained in the statistics of the outcome of many experiments, and this 
exists in a physical form accessible to the observer. So, if 100 spin 
measurements experiments have been done then the outcomes of these exist 
in each sector in the form of a physical record.



Saibal

--
You received this message because you are subscribed to the Google Groups 
"Everything List" group.
To unsubscribe from this group and stop receiving emails from it, send an email 
to everything-list+unsubscr...@googlegroups.com.
To view this discussion on the web visit 
https://groups.google.com/d/msgid/everything-list/14e823bc8ed35d0543c510c08e1d33ed%40zonnet.nl.


Re: Irreducible randomness in QM

2020-12-24 Thread John Clark
On Wed, Dec 23, 2020 at 7:51 PM Bruce Kellett  wrote:

> *The question is whether it is merely a version of the observer, a copy
> of the observer, or the actual observer who sees all possible outcomes*


In Many Worlds it's meaningless to ask which one is the original, there is
nothing original about "the original ''. If you've seen one John Clark
you've seen them all.

*> Since there is a sense in which 'you' certainly see all possible
> outcomes, there is an immediate conflict with the Born rule, according to
> which different outcomes have different probabilities,*


Predictions may have different probabilities but outcomes always have
exactly the same probability,  if X is observed to have happened then the
probability that X had occurred is always exactly 1. If Many Worlds is
right and there are an astronomical number of John Clarks then about half
of them will see the coin land heads and about half will see it land tails
and a very very small number of them will see neither, they will defy the
odds and see the coin land on its edge. But before the flip all of them
predicted that the odds would be 50-50.

*> the duplicates under consideration here are, by hypothesis, all
> identical copies of the original.*
>

Yes, just as all copies of an email sent out via a mailing list are
identical with the original that was typed on the author's computer. If we
were arguing about something it would be silly for me to say I failed to
convince you because you only saw a copy of my email not the original.

John K Clark

-- 
You received this message because you are subscribed to the Google Groups 
"Everything List" group.
To unsubscribe from this group and stop receiving emails from it, send an email 
to everything-list+unsubscr...@googlegroups.com.
To view this discussion on the web visit 
https://groups.google.com/d/msgid/everything-list/CAJPayv0Art-Vw59t%3DaF3Wq9k0vmd-puFs%2B8Uyh29_Y4eZ2Z0Ag%40mail.gmail.com.


Re: Irreducible randomness in QM

2020-12-23 Thread Alan Grayson


On Wednesday, December 23, 2020 at 1:45:06 AM UTC-7 Alan Grayson wrote:

>
>
> On Sunday, December 20, 2020 at 6:36:53 AM UTC-7 Alan Grayson wrote:
>
>> On Sunday, December 20, 2020 at 12:52:26 AM UTC-7 Bruce wrote:
>>
>>> On Sun, Dec 20, 2020 at 5:57 PM 'Brent Meeker' via Everything List <
>>> everyth...@googlegroups.com> wrote:
>>>
 On 12/19/2020 10:38 PM, Bruce Kellett wrote:

>>> On Sun, Dec 20, 2020 at 6:48 AM 'Brent Meeker' via Everything List <
 everyth...@googlegroups.com> wrote:

 As far as I know, it was Born who came up with the interpretation of 
> the equations as expressing probabilities.  But there was (and maybe 
> still 
> is) controversy over whether this was irreducibly random or whether there 
> were hidden variables and it was just the randomness of ignorance.  For 
> most physicists this was resolved by the experimental confirmation of the 
> violation of Bell inequalities.  At that point the choice was irreducible 
> randomness or nonlocal effects
>

 That is not quite right. The choice is not between randomnesss and 
 non-locality. Non-local hidden variables (Bohm) do reduce the apparent 
 randomness to ignorance of the detailed quantum state, but at the price of 
 non-locality. Bell's result implies that non-locality is unavoidable, and 
 this has nothing to do with the presence or absence of intrinsic 
 randomness. 


 If there were not intrinsic randomness then the extra correlation of 
 that violates Bell's inequality could be used to signal faster than light.

>>>
>>>
>>> True, but irrelevant to what I said. There is no theory that gives a 
>>> local account of the Bell correlations. Intrinsic randomness guarantees no 
>>> FTL signalling. This seems to rule out local deterministic theories.
>>>
>>
>> *Intrinsic randomness guarantees no FTL signaling. Wow! That's a 
>> breathtaking claim. How is it justified? What is the argument? TIA, AG *
>>
>
> *I'm not disputing your claim. But it's hugely profound, if true. Can you 
> say something, anything about how you've reached this conclusion? TIA, AG *
>

*Maybe your claim is so profound that it's sacrilege to say anything about 
it? Could be. AG *

>
>>> Bruce
>>>
 It is only deterministic theories like MWI and Bohm that eliminate 
 randomness, but MWI does not solve the locality issue either. Besides, MWI 
 is incompatible with the Born Rule; and the Born rule, while consistent 
 with Bohm, cannot be derived from Bohmian mechanics.

 Bruce

 and most physicists saw randomness as the more likely, less disruptive 
> choice.
>
> Brent
>


-- 
You received this message because you are subscribed to the Google Groups 
"Everything List" group.
To unsubscribe from this group and stop receiving emails from it, send an email 
to everything-list+unsubscr...@googlegroups.com.
To view this discussion on the web visit 
https://groups.google.com/d/msgid/everything-list/5b2b3d3b-2f1c-47ac-b398-fe0f74f9a0a3n%40googlegroups.com.


Re: Irreducible randomness in QM

2020-12-23 Thread Bruce Kellett
On Thu, Dec 24, 2020 at 5:39 AM Stathis Papaioannou 
wrote:

>
> You suspected right, I am asking a more basic question about self-sampling
> and the validity of probabilities when a version of the observer sees all
> possible outcomes.
>

There is a problem here -- or maybe it is just careless phrasing. You say
there is a question about probabilities when a version of the observer sees
all possible outcomes. The question is whether it is merely a version of
the observer, a copy of the observer, or the actual observer who sees all
possible outcomes? A "version" is somewhat ambiguous. Different 'versions'
of an operating system, for example, differ in some way. Whereas the
duplicates under consideration here are, by hypothesis, all identical
copies of the original.

As John Clark is fond of pointing out, the trouble with self-sampling from
a set of identical duplicate persons is that the personal pronoun 'you'
loses its unique reference. All copies have an equal claim to be identified
as the original 'you', so there is a real sense in which 'you' see all
outcomes, with probability one. If you attempt to single out a particular
individual by some random sampling procedure, you immediately make a
dualist assumption -- the selected individual is different from the rest
(by virtue of a 'soul', or some such, conferred by the sampling process
itself).

Since there is a sense in which 'you' certainly see all possible outcomes,
there is an immediate conflict with the Born rule, according to which
different outcomes have different probabilities, and 'you' can't see more
than one such outcome.

Bruce

-- 
You received this message because you are subscribed to the Google Groups 
"Everything List" group.
To unsubscribe from this group and stop receiving emails from it, send an email 
to everything-list+unsubscr...@googlegroups.com.
To view this discussion on the web visit 
https://groups.google.com/d/msgid/everything-list/CAFxXSLQ%2B_MBo1by2ttD3-GMFAHysh9zxg3Zfd1c6kOMdGJQRbQ%40mail.gmail.com.


Re: Irreducible randomness in QM

2020-12-23 Thread Lawrence Crowell
The equation I derived is a geodesic deviation equation! 

LC

On Wednesday, December 23, 2020 at 1:19:13 PM UTC-6 johnk...@gmail.com 
wrote:

> On Wed, Dec 23, 2020 at 2:00 PM Lawrence Crowell  
> wrote:
>
> > There is a correspondence between the geodesic deviation equation and 
>> the Schrodinger equation.
>>
>
> Wouldn't geodesic deviation be more relevant when dealing with gravity 
> and General Relativity than with Quantum Mechanics?
>
> John K Clark
>
>

-- 
You received this message because you are subscribed to the Google Groups 
"Everything List" group.
To unsubscribe from this group and stop receiving emails from it, send an email 
to everything-list+unsubscr...@googlegroups.com.
To view this discussion on the web visit 
https://groups.google.com/d/msgid/everything-list/9c0a7042-9162-4add-96f5-2ed3e7ae5203n%40googlegroups.com.


Re: Irreducible randomness in QM

2020-12-23 Thread John Clark
On Wed, Dec 23, 2020 at 2:00 PM Lawrence Crowell <
goldenfieldquaterni...@gmail.com> wrote:

> There is a correspondence between the geodesic deviation equation and the
> Schrodinger equation.
>

Wouldn't geodesic deviation be more relevant when dealing with gravity and
General Relativity than with Quantum Mechanics?

John K Clark

-- 
You received this message because you are subscribed to the Google Groups 
"Everything List" group.
To unsubscribe from this group and stop receiving emails from it, send an email 
to everything-list+unsubscr...@googlegroups.com.
To view this discussion on the web visit 
https://groups.google.com/d/msgid/everything-list/CAJPayv0ZFyMVkWsKfhg6TA75mMY4RDD07FWyiV5_4i_0sp8Z%2BQ%40mail.gmail.com.


Re: Irreducible randomness in QM

2020-12-23 Thread Lawrence Crowell
On Wednesday, December 23, 2020 at 8:43:05 AM UTC-6 johnk...@gmail.com 
wrote:

> On Tue, Dec 22, 2020 at 4:52 PM Bruce Kellett  wrote:
>
> > *There are many 1p views*
>>
>
> Yes.
>
> > *and for each the probability of that particular observation is one*
>>
>  
> Yes, AFTER Bruce Kellett has observed that the electron went left rather 
> than right then the probability the electron had gone left rather than 
> right is exactly precisely 1. And that proves that making accurate 
> predictions is very very easy if one is permitted to make the predictions 
> AFTER the events in question have happened.
>
> > contradicting the Born rule calculation of the probability in every 
>> case.
>>
>
> No, the Born rule Is about making a prediction BEFORE the event has 
> happened not AFTER.
>  
>
>> > *The Born rule cannot be deduced from the Schrodinger equation*
>>
>
> That is true. The Schrodinger equation just says for every particle there 
> is a wave that is associated with it, the equation says nothing about what 
> is waving, it doesn't even say if that wave has any observable 
> consequences; for that you need a quantum interpretation. The Born Rule is 
> a quantum interpretation. Max Born advanced the idea because from 
> experiments it was shown to have worked and for no other reason. Max Born 
> noticed that if Max Born took the square of the absolute value of the wave 
> function at a point it would provide the probability that Max Born would be 
> able to observe that particle at that point. 
>
> For example, Max Born noticed that if Max Born assumed that the square of 
> the absolute value of the wave function at a point was a probability and if 
> it said the electron was 60% likely to go left and 40% likely to go right 
> under the specified experimental conditions and the experiment was repeated 
> many times with the same conditions then Max Born would observe the 
> electron go left about 60% of the time and right about 40% of the time, and 
> the more times the experiment was repeated the closer it would get to that 
> 60/40 ratio. 
>
> > *they are incompatible.*
>>
>
> If there is one thing that we know for certain about Quantum Mechanics 
> it's that the Born Rule WORKS, so if the Schrodinger Equation was 
> incompatible with the Born Rule then the Schrodinger equation would be a 
> useless piece of garbage. It's not.
>
> John K Clark
>

There is a correspondence between the geodesic deviation equation and the 
Schrodinger equation. A unitary operator U(t) acts on a wave function so 
that U(t)ψ(0) = ψ(t) and U obeys the Schrödinger equation 

iU_t = HU.

This Schrodinger equation may be re-expressed as iU_tU^{-1} = H or iU_tU^† 
= H. We can take an overall time derivative to get

iU_{tt} = H_tU + HU_t 

or

iU_{tt} = i∂_t(U_tU^†) + iU_tU^†U_t.

The term ∂_t(U_tU^†) or ∂_tH is equal to the commutator [H, H] = 0 and so 
we have the elementary equation

U_{tt} = U_tU^\dagger U_t.

This equation is analogous to 

d^2x^α/ds^2} = R^α_{μβν}U^μx^βU^ν.

which is like the unitary evolution equation. The curvature terms may be 
absorbed into the U^μ where the geodesic equation this is a real valued 
analogue of unitary evolution.

LC

-- 
You received this message because you are subscribed to the Google Groups 
"Everything List" group.
To unsubscribe from this group and stop receiving emails from it, send an email 
to everything-list+unsubscr...@googlegroups.com.
To view this discussion on the web visit 
https://groups.google.com/d/msgid/everything-list/8dd51478-8fc7-47f1-b71f-2ed7417792b2n%40googlegroups.com.


Re: Irreducible randomness in QM

2020-12-23 Thread Stathis Papaioannou
On Wed, 23 Dec 2020 at 21:51, Bruce Kellett  wrote:

> On Wed, Dec 23, 2020 at 8:49 PM Stathis Papaioannou 
> wrote:
>
>> On Wed, 23 Dec 2020 at 13:29, Bruce Kellett 
>> wrote:
>>
>>> On Wed, Dec 23, 2020 at 12:45 PM Stathis Papaioannou 
>>> wrote:
>>>

 From what I understand of your position, you would claim that the 1 in
 10^100 copy will screw up the very concept of probability. If that extra
 copy did not exist, you would take dollars, because you will certainly need
 dollars; but with the extra copy you would just throw up your hands and say
 you don't know what to do, because it is certain you will need dollars and
 euros.

>>>
>>>
>>> My complaint about your example is that you are changing the problem --
>>> you are changing the probabilities in a way that is incompatible with both
>>> the Schrodinger equation and the Born rule.
>>>
>>
>> I thought you were talking about probabilities in general where the
>> observer is duplicated because you have a problem with self-sampling. Do
>> you think it makes a difference if there is a 1/2 event, a 1/52 event or a
>> 1/10^100 event? Would the 1/10^100 event cause your metaphysical problem,
>> such that ignoring it requires an assumption of an immaterial soul?
>>
>
> As I suspected, you are simply avoiding the basic question, which is "Is
> MWI (Everett) consistent with the Born rule?". In order to approach this
> problem, we have to look at probabilities and duplication as implied by
> Everettian QM. You have lost track of this by making up extreme examples
> and asking the wrong questions.
>
> We can return to the initial problem if you are interested. But I doubt
> that you are really going to engage with the case that I have made. If
> every outcome occurs on every trial, each outcome individually has
> unit probability -- it certainly occurs. This is incompatible with the Born
> rule. (Do low probability outcomes certainly occur?)
>

You suspected right, I am asking a more basic question about self-sampling
and the validity of probabilities when a version of the observer sees all
possible outcomes.

> --
Stathis Papaioannou

-- 
You received this message because you are subscribed to the Google Groups 
"Everything List" group.
To unsubscribe from this group and stop receiving emails from it, send an email 
to everything-list+unsubscr...@googlegroups.com.
To view this discussion on the web visit 
https://groups.google.com/d/msgid/everything-list/CAH%3D2ypVDejYXsB5rFcXEX%2BM6hpKQ2AuRRNjNywDAqhfv4pP3qQ%40mail.gmail.com.


Re: Irreducible randomness in QM

2020-12-23 Thread John Clark
On Tue, Dec 22, 2020 at 9:29 PM Bruce Kellett  wrote:

>* t**his is still inconsistent with the Schrodinger equation because* [blah
> blah]


The Schrodinger Equation is not holy writ, but the Born Rule is because it
has been shown over and over again to work. The Schrodinger Equation Is
useful only because it provides the function that the Born Rule says you
should take the square of the absolute value of.

John K Clark

-- 
You received this message because you are subscribed to the Google Groups 
"Everything List" group.
To unsubscribe from this group and stop receiving emails from it, send an email 
to everything-list+unsubscr...@googlegroups.com.
To view this discussion on the web visit 
https://groups.google.com/d/msgid/everything-list/CAJPayv3JZjx9KE8PwW81fE-BoYP2b%2BtFCm2U5Ja7M-gNv3Th2Q%40mail.gmail.com.


Re: Irreducible randomness in QM

2020-12-23 Thread John Clark
On Tue, Dec 22, 2020 at 5:15 PM Bruce Kellett  wrote:

*> Think of it like this: take a randomly shuffled deck of cards and hand
> one card from the deck to each of 52 people. The probability that one of
> the people will get the 3-of-Spades is one. *


If Many Worlds is correct then the probability that somebody named Bruce
Kellett who has memories and a personality identical with the person that
wrote the email this is a response to will get a 3 of spades is 1, and
the probability that somebody named Bruce Kellett who has memories and a
personality identical with the person that wrote the email this is a
response to will NOT get a 3 of spades is also 1 because in Many Worlds
everything that can happen, that doesn't violate the laws of physics, will
happen.

*> The probability that 'You' will get the 3-of-Spades in a fair shuffle is
> 1/52. *


If Many Worlds is correct then the above statement is OK if looked at from
a everyday strictly close up local perspective, but from a global viewpoint
it would be neither true nor false, it would be gibberish because the
personal pronoun is gibberish.

*> The difference is that you have identified yourself in advance. *


John Clark identifies "you" as anyone who has the memories and personality
of Bruce Kellett; there are an astronomical number of John Clark's, maybe
even an infinite number, but so far none of them have ever observed more
than one chunk of matter that has those characteristics. But there is no
reason in principle why that state of affairs couldn't change.

> *The dualist assumption is equivalent.*


That question would be better dealt with on a list about the martial arts
not this one.

John K Clark

-- 
You received this message because you are subscribed to the Google Groups 
"Everything List" group.
To unsubscribe from this group and stop receiving emails from it, send an email 
to everything-list+unsubscr...@googlegroups.com.
To view this discussion on the web visit 
https://groups.google.com/d/msgid/everything-list/CAJPayv0PnnZcKRUsqmo%3Dc6%2BRouEZYxwasF5qFWh-h2ka6fFesA%40mail.gmail.com.


Re: Irreducible randomness in QM

2020-12-23 Thread John Clark
On Tue, Dec 22, 2020 at 4:52 PM Bruce Kellett  wrote:

> *There are many 1p views*
>

Yes.

> *and for each the probability of that particular observation is one*
>

Yes, AFTER Bruce Kellett has observed that the electron went left rather
than right then the probability the electron had gone left rather than
right is exactly precisely 1. And that proves that making accurate
predictions is very very easy if one is permitted to make the predictions
AFTER the events in question have happened.

> contradicting the Born rule calculation of the probability in every case.
>

No, the Born rule Is about making a prediction BEFORE the event has
happened not AFTER.


> > *The Born rule cannot be deduced from the Schrodinger equation*
>

That is true. The Schrodinger equation just says for every particle there
is a wave that is associated with it, the equation says nothing about what
is waving, it doesn't even say if that wave has any observable
consequences; for that you need a quantum interpretation. The Born Rule is
a quantum interpretation. Max Born advanced the idea because from
experiments it was shown to have worked and for no other reason. Max Born
noticed that if Max Born took the square of the absolute value of the wave
function at a point it would provide the probability that Max Born would be
able to observe that particle at that point.

For example, Max Born noticed that if Max Born assumed that the square of
the absolute value of the wave function at a point was a probability and if
it said the electron was 60% likely to go left and 40% likely to go right
under the specified experimental conditions and the experiment was repeated
many times with the same conditions then Max Born would observe the
electron go left about 60% of the time and right about 40% of the time, and
the more times the experiment was repeated the closer it would get to that
60/40 ratio.

> *they are incompatible.*
>

If there is one thing that we know for certain about Quantum Mechanics it's
that the Born Rule WORKS, so if the Schrodinger Equation was incompatible
with the Born Rule then the Schrodinger equation would be a useless piece
of garbage. It's not.

John K Clark

-- 
You received this message because you are subscribed to the Google Groups 
"Everything List" group.
To unsubscribe from this group and stop receiving emails from it, send an email 
to everything-list+unsubscr...@googlegroups.com.
To view this discussion on the web visit 
https://groups.google.com/d/msgid/everything-list/CAJPayv0VRBiYs%2B9p0LASSD9zdZQUPngcZfsRgW_rqd4x1M21rw%40mail.gmail.com.


Re: Irreducible randomness in QM

2020-12-23 Thread Bruno Marchal

> On 22 Dec 2020, at 05:03, 'Brent Meeker' via Everything List 
>  wrote:
> 
> Science doesn't deal in proofs, only in evidence.  And the reality it deals 
> with is that which can be tested...i.e. is not "underlying”.

I sort of agree. To believe in an underlying reality, or in a fundamental 
reality is not physical science, but theology. But we can do theology with a 
scientific attitude, and find criteria to test the existence and nature of the 
underlying reality that we postulate.

Today, when we do that, the evidences are in favour of Plato, not in favour of 
Aristotle. 
We can say that we have looked carefully to Nature, and that the evidence is 
that Nature is not an ontological, “underlying” reality, but the emergent 
product from the arithmetical relations.

Unfortunately, many people confuse the many evidences for the physical reality 
with an evidence that the physical reality would the underlying reality. That 
move *is* Aristotle postulation or act of faith. It works well for many 
applications, but is not sustained, neither by facts, nor by most of our 
current theories, which, like Darwin Evolution, relies on the Mechanist 
assumption (even implicitly on the Church-Turing these part of Mechanism).

Bruno



> 
> Brent
> 
> On 12/21/2020 6:49 PM, Alan Grayson wrote:
>> Can you actually define "irreducible randomness" in order to prove it's the 
>> underlying reality of the universe? If so, what is it?   TIA, AG
>> 
>> On Monday, December 21, 2020 at 9:25:57 AM UTC-7 Lawrence Crowell wrote:
>> Bell's theorem and the Kochen-Specker theorem are indications of an 
>> irreducible randomness to measurement outcomes in QM.
>> 
>> LC
>> 
>> On Thursday, December 17, 2020 at 12:50:58 PM UTC-6 agrays...@gmail.com 
>>  wrote:
>> Can it be directly inferred from Heisenberg's Uncertainty Principle? TIA, AG
>> -- 
>> You received this message because you are subscribed to the Google Groups 
>> "Everything List" group.
>> To unsubscribe from this group and stop receiving emails from it, send an 
>> email to everything-list+unsubscr...@googlegroups.com 
>> .
>> To view this discussion on the web visit 
>> https://groups.google.com/d/msgid/everything-list/80062de9-c99b-462c-b59a-a0e622775ec1n%40googlegroups.com
>>  
>> .
> 
> 
> -- 
> You received this message because you are subscribed to the Google Groups 
> "Everything List" group.
> To unsubscribe from this group and stop receiving emails from it, send an 
> email to everything-list+unsubscr...@googlegroups.com 
> .
> To view this discussion on the web visit 
> https://groups.google.com/d/msgid/everything-list/9654c839-31c3-4344-57c2-c5ecb9a031c7%40verizon.net
>  
> .

-- 
You received this message because you are subscribed to the Google Groups 
"Everything List" group.
To unsubscribe from this group and stop receiving emails from it, send an email 
to everything-list+unsubscr...@googlegroups.com.
To view this discussion on the web visit 
https://groups.google.com/d/msgid/everything-list/19FB5056-8C9A-4460-8F38-7D793D0AB79B%40ulb.ac.be.


Re: Irreducible randomness in QM

2020-12-23 Thread Bruno Marchal

> On 21 Dec 2020, at 17:25, Lawrence Crowell  
> wrote:
> 
> Bell's theorem and the Kochen-Specker theorem are indications of an 
> irreducible randomness to measurement outcomes in QM.

That is clear for me when you assume one and only one physical universe, or one 
measurement outcome.

It is less clear if we assume 0 physical universe (like we have to if we want 
biologie and Darwin to keep their explanation power, as they use mechanism and 
this is hard to sustain with any (weak) form of materialism.

Keep in mind that Mechanism introduce already an irreducible randomness in 
self- measurement in the self-duplication/multiplication (in arithmetic, or 
anywhere).

Normally, the quantum weirdness is simply the canonical weirdness met by all 
universal numbers in arithmetic, and due to its intrinsic lack of ability to 
determine which computation run her, among the infinities of computation in any 
“universal-Turing system”.

Bell’s theorem and Kochen-Specker theorem are part of the confirmation that the 
quantum MWI is just the arithmetical multi-computations reality, structured by 
the modes of self-reference of the universal machine (an arithmetical concept).

This per se does not solve the mind-body problem, but is the beginning of its 
formulation. The solution is in the consequence of the (counter-intuitive) 
theory of machine self-references (which applies also to some “gods” (non 
Turing emulable entity) which exist also in the arithmetical reality (called 
“oracle” by Turing).

Some evidences for some type of hidden variable in the physical reality would 
give some evidence that Mechanism is false, but I have not yet seen such 
evidences.

Bruno



> 
> LC
> 
> On Thursday, December 17, 2020 at 12:50:58 PM UTC-6 agrays...@gmail.com wrote:
> Can it be directly inferred from Heisenberg's Uncertainty Principle? TIA, AG
> 
> -- 
> You received this message because you are subscribed to the Google Groups 
> "Everything List" group.
> To unsubscribe from this group and stop receiving emails from it, send an 
> email to everything-list+unsubscr...@googlegroups.com 
> .
> To view this discussion on the web visit 
> https://groups.google.com/d/msgid/everything-list/3dca72aa-febd-4b20-afee-e43a7fcee319n%40googlegroups.com
>  
> .

-- 
You received this message because you are subscribed to the Google Groups 
"Everything List" group.
To unsubscribe from this group and stop receiving emails from it, send an email 
to everything-list+unsubscr...@googlegroups.com.
To view this discussion on the web visit 
https://groups.google.com/d/msgid/everything-list/104534D7-4A45-42FF-9888-FB37A090129F%40ulb.ac.be.


Re: Irreducible randomness in QM

2020-12-23 Thread Bruce Kellett
On Wed, Dec 23, 2020 at 8:49 PM Stathis Papaioannou 
wrote:

> On Wed, 23 Dec 2020 at 13:29, Bruce Kellett  wrote:
>
>> On Wed, Dec 23, 2020 at 12:45 PM Stathis Papaioannou 
>> wrote:
>>
>>>
>>> From what I understand of your position, you would claim that the 1 in
>>> 10^100 copy will screw up the very concept of probability. If that extra
>>> copy did not exist, you would take dollars, because you will certainly need
>>> dollars; but with the extra copy you would just throw up your hands and say
>>> you don't know what to do, because it is certain you will need dollars and
>>> euros.
>>>
>>
>>
>> My complaint about your example is that you are changing the problem --
>> you are changing the probabilities in a way that is incompatible with both
>> the Schrodinger equation and the Born rule.
>>
>
> I thought you were talking about probabilities in general where the
> observer is duplicated because you have a problem with self-sampling. Do
> you think it makes a difference if there is a 1/2 event, a 1/52 event or a
> 1/10^100 event? Would the 1/10^100 event cause your metaphysical problem,
> such that ignoring it requires an assumption of an immaterial soul?
>

As I suspected, you are simply avoiding the basic question, which is "Is
MWI (Everett) consistent with the Born rule?". In order to approach this
problem, we have to look at probabilities and duplication as implied by
Everettian QM. You have lost track of this by making up extreme examples
and asking the wrong questions.

We can return to the initial problem if you are interested. But I doubt
that you are really going to engage with the case that I have made. If
every outcome occurs on every trial, each outcome individually has
unit probability -- it certainly occurs. This is incompatible with the Born
rule. (Do low probability outcomes certainly occur?)

Bruce

-- 
You received this message because you are subscribed to the Google Groups 
"Everything List" group.
To unsubscribe from this group and stop receiving emails from it, send an email 
to everything-list+unsubscr...@googlegroups.com.
To view this discussion on the web visit 
https://groups.google.com/d/msgid/everything-list/CAFxXSLTDNRHShQtcpHZ%3DiH3mNrQmJ86L537HZ%3D9y3oa7ZYxEGA%40mail.gmail.com.


Re: Irreducible randomness in QM

2020-12-23 Thread Stathis Papaioannou
On Wed, 23 Dec 2020 at 13:29, Bruce Kellett  wrote:

> On Wed, Dec 23, 2020 at 12:45 PM Stathis Papaioannou 
> wrote:
>
>> On Wed, 23 Dec 2020 at 10:58, Bruce Kellett 
>> wrote:
>>
>>> On Wed, Dec 23, 2020 at 10:45 AM Stathis Papaioannou 
>>> wrote:
>>>
 On Wed, 23 Dec 2020 at 09:15, Bruce Kellett 
 wrote:

> On Wed, Dec 23, 2020 at 9:07 AM Stathis Papaioannou <
> stath...@gmail.com> wrote:
>
>> On Wed, 23 Dec 2020 at 09:02, Bruce Kellett 
>> wrote:
>>
>>> On Wed, Dec 23, 2020 at 8:32 AM Stathis Papaioannou <
>>> stath...@gmail.com> wrote:
>>>
 On Tue, 22 Dec 2020 at 21:31, Bruce Kellett 
 wrote:

> On Tue, Dec 22, 2020 at 9:19 PM Stathis Papaioannou <
> stath...@gmail.com> wrote:
>
>>
>> All the copies could be conscious or all could be zombies; none
>> are privileged.
>>
>
> What difference does that make? One has to be privileged in some
> way if there is to be a probability different from zero.
>

 Why did you say it was dualist if it doesn't make a difference that
 it isn't dualist?

>>>
>>> It makes no difference if all copies are conscious, or if all are
>>> zombies -- you are still making a dualist assumption.
>>>
>>> The probability calculated where there are multiple copies is the
 probability that one randomly sampled copy will see a particular 
 outcome. I
 am one randomly sampled copy.

>>>
>>>
>>> And that is precisely the dualist assumption that  is intrinsic in
>>> all self-location (indexical) arguments. I think Brent has understood 
>>> this
>>> when he says "That seems to imply dualism.  All the bodies exist, but 
>>> your
>>> soul only goes with one."
>>>
>>
>> I could say that my soul is duplicated and I want to know the
>> probability that I am one randomly sampled soul. I could say that the
>> carrots are duplicated and I want to know the probability that I get a
>> particular randomly sampled carrot. I don't have a problem with it; you 
>> do,
>> and there seems to be no way around it.
>>
>
>
> Think of it like this: take a randomly shuffled deck of cards and hand
> one card from the deck to each of 52 people. The probability that one of
> the people will get the 3-of-Spades is one. The probability that 'You' 
> will
> get the 3-of-Spades in a fair shuffle is 1/52. The difference is that you
> have identified yourself in advance. The dualist assumption is equivalent.
>

 Let's say you are copied 10^100 times. One copy will end up in a place
 where they use euros and the rest will end up in a place where they use
 dollars. Do you put euros or dollars in your wallet before duplication?

>>>
>>>
>>> Let's say I wait and see! and go to the money exchange if necessary. You
>>> are posing a different problem, one in which the number of copies on a
>>> particular branch is increased. That is incompatible with MWI and Everett
>>> with non-degenerate eigenvalues.
>>>
>>> You don't avoid the dualist implications of self-selection by increasing
>>> the number of copies: the example with 52 cards says everything that is
>>> necessary.
>>>
>>
>> From what I understand of your position, you would claim that the 1 in
>> 10^100 copy will screw up the very concept of probability. If that extra
>> copy did not exist, you would take dollars, because you will certainly need
>> dollars; but with the extra copy you would just throw up your hands and say
>> you don't know what to do, because it is certain you will need dollars and
>> euros.
>>
>
>
> My complaint about your example is that you are changing the problem --
> you are changing the probabilities in a way that is incompatible with both
> the Schrodinger equation and the Born rule.
>

I thought you were talking about probabilities in general where the
observer is duplicated because you have a problem with self-sampling. Do
you think it makes a difference if there is a 1/2 event, a 1/52 event or a
1/10^100 event? Would the 1/10^100 event cause your metaphysical problem,
such that ignoring it requires an assumption of an immaterial soul?

But there could be more moderate examples of branch duplication that would
> be more in line with what is proposed by some people. For example, both
> Sean Carroll and Zurek propose a procedure whereby they expand the number
> of branches so that all branches have equal amplitudes (weights, or Born
> probabilities). This is incompatible with the Schrodinger equation, but if
> we leave that aside for the moment, it gives a branch-counting solution to
> the probability question. The idea then is that you self-select from a
> uniform random distribution over this expanded set of branches.  However,
> the expansion of the number of branches in this 

Re: Irreducible randomness in QM

2020-12-23 Thread 'scerir' via Everything List
AG asked: does randomness imply no-FTL-signaling?

Let me ask: does determinism imply FTL-signaling?

A is one of the two wings of a Bell apparatus
i is the observable to be measured in A
x is the possible value of i
B is the other wing of a Bell apparatus
j is the observable to be measured in B
y is  the possible value of j
Lambda are hidden variables
p are probabilities

If we write
p_[A,Lambda] (x|i,j) = p_[A,Lambda] (x|i)
p_[B,Lambda] (y|i,j) = p_[B,Lambda] (y|j)
the above is a sort of "locality" condition,
since the value x only depends on the observable i,
and the value y only depends on the observable j.

In a (hypothetical 
https://www.google.com/search?client=firefox-b-d=ALeKk02Cur06my_H17d_9fUXbVDqBH5ESQ:1608715874782=hypothetical=1=X=2ahUKEwiAy4rS5ePtAhXLO-wKHWX6DTYQkeECKAB6BAgEEDU
 ) deterministic theory (but reproducing QM)
the above "locality" condition is violated.

-- 
You received this message because you are subscribed to the Google Groups 
"Everything List" group.
To unsubscribe from this group and stop receiving emails from it, send an email 
to everything-list+unsubscr...@googlegroups.com.
To view this discussion on the web visit 
https://groups.google.com/d/msgid/everything-list/871899580.451822.1608716365874%40mail1.libero.it.


Re: Irreducible randomness in QM

2020-12-23 Thread Alan Grayson
If micro reality is irreducibly random, why isn't it appropriate to refer 
to it as the "underlying reality"? You seem to be splitting hairs to no 
avail. AG

On Monday, December 21, 2020 at 9:27:20 PM UTC-7 Alan Grayson wrote:

> Bullshit; Counterexample; the derivation of the LT from the principle of 
> the invariance of the SoL. AG
>
> On Monday, December 21, 2020 at 9:03:14 PM UTC-7 Brent wrote:
>
>> Science doesn't deal in proofs, only in evidence.  And the reality it 
>> deals with is that which can be tested...i.e. is not "underlying".
>>
>> Brent
>>
>>
>> On 12/21/2020 6:49 PM, Alan Grayson wrote:
>>
>> Can you actually define "irreducible randomness" in order to prove it's 
>> the underlying reality of the universe? If so, what is it? TIA, AG 
>>
>> On Monday, December 21, 2020 at 9:25:57 AM UTC-7 Lawrence Crowell wrote:
>>
>>> Bell's theorem and the Kochen-Specker theorem are indications of an 
>>> irreducible randomness to measurement outcomes in QM. 
>>>
>>> LC
>>>
>>> On Thursday, December 17, 2020 at 12:50:58 PM UTC-6 agrays...@gmail.com 
>>> wrote:
>>>
 Can it be directly inferred from Heisenberg's Uncertainty Principle? 
 TIA, AG
>>>
>>> -- 
>>
>> You received this message because you are subscribed to the Google Groups 
>> "Everything List" group.
>> To unsubscribe from this group and stop receiving emails from it, send an 
>> email to everything-li...@googlegroups.com.
>>
>> To view this discussion on the web visit 
>> https://groups.google.com/d/msgid/everything-list/80062de9-c99b-462c-b59a-a0e622775ec1n%40googlegroups.com
>>  
>> 
>> .
>>
>>
>>

-- 
You received this message because you are subscribed to the Google Groups 
"Everything List" group.
To unsubscribe from this group and stop receiving emails from it, send an email 
to everything-list+unsubscr...@googlegroups.com.
To view this discussion on the web visit 
https://groups.google.com/d/msgid/everything-list/177687db-d2f6-4ee1-ac14-23ee5f156eb9n%40googlegroups.com.


Re: Irreducible randomness in QM

2020-12-23 Thread Alan Grayson


On Sunday, December 20, 2020 at 6:36:53 AM UTC-7 Alan Grayson wrote:

> On Sunday, December 20, 2020 at 12:52:26 AM UTC-7 Bruce wrote:
>
>> On Sun, Dec 20, 2020 at 5:57 PM 'Brent Meeker' via Everything List <
>> everyth...@googlegroups.com> wrote:
>>
>>> On 12/19/2020 10:38 PM, Bruce Kellett wrote:
>>>
>>> On Sun, Dec 20, 2020 at 6:48 AM 'Brent Meeker' via Everything List <
>>> everyth...@googlegroups.com> wrote:
>>>
 As far as I know, it was Born who came up with the interpretation of 
 the equations as expressing probabilities.  But there was (and maybe still 
 is) controversy over whether this was irreducibly random or whether there 
 were hidden variables and it was just the randomness of ignorance.  For 
 most physicists this was resolved by the experimental confirmation of the 
 violation of Bell inequalities.  At that point the choice was irreducible 
 randomness or nonlocal effects

>>>
>>> That is not quite right. The choice is not between randomnesss and 
>>> non-locality. Non-local hidden variables (Bohm) do reduce the apparent 
>>> randomness to ignorance of the detailed quantum state, but at the price of 
>>> non-locality. Bell's result implies that non-locality is unavoidable, and 
>>> this has nothing to do with the presence or absence of intrinsic 
>>> randomness. 
>>>
>>>
>>> If there were not intrinsic randomness then the extra correlation of 
>>> that violates Bell's inequality could be used to signal faster than light.
>>>
>>
>>
>> True, but irrelevant to what I said. There is no theory that gives a 
>> local account of the Bell correlations. Intrinsic randomness guarantees no 
>> FTL signalling. This seems to rule out local deterministic theories.
>>
>
> *Intrinsic randomness guarantees no FTL signaling. Wow! That's a 
> breathtaking claim. How is it justified? What is the argument? TIA, AG *
>

*I'm not disputing your claim. But it's hugely profound, if true. Can you 
say something, anything about how you've reached this conclusion? TIA, AG *

>
>> Bruce
>>
>>> It is only deterministic theories like MWI and Bohm that eliminate 
>>> randomness, but MWI does not solve the locality issue either. Besides, MWI 
>>> is incompatible with the Born Rule; and the Born rule, while consistent 
>>> with Bohm, cannot be derived from Bohmian mechanics.
>>>
>>> Bruce
>>>
>>> and most physicists saw randomness as the more likely, less disruptive 
 choice.

 Brent

>>>

-- 
You received this message because you are subscribed to the Google Groups 
"Everything List" group.
To unsubscribe from this group and stop receiving emails from it, send an email 
to everything-list+unsubscr...@googlegroups.com.
To view this discussion on the web visit 
https://groups.google.com/d/msgid/everything-list/e655a650-dbf0-4232-90ac-b4ff3e72010en%40googlegroups.com.


Re: Irreducible randomness in QM

2020-12-22 Thread Bruce Kellett
On Wed, Dec 23, 2020 at 2:53 PM smitra  wrote:

>
> imposing the Born rule by fiat is not incompatible with each outcome
> being realized.


It is, actually. The Born rule gives probabilities that are incompatible
with every outcoming occurring on every trial. If every outcome always
occurs you are led to statements such as "This low probability outcome is
certain to occur", which is nonsense. If something is certain to occur, it
has probability one, but the Born rule never gives unit probability to a
single outcome from the set.


Your identity at any time is specified by all the
> information that specifies your physical state. After each observation,
> your identity changes. But this is only by a few bits of information,
> allowing us to ignore that change. However in the sort of discussion
> like this one about the MWI this change of identity due to different
> outcomes of the measurements in the different sectors is of crucial
> importance. The Born rule then specifies a measure on the space of all
> possible observers, where we also distinguish two observers who split
> off from the same observer after a measurement.
>

But the Born probabilities are inconsistent with unit probability for every
possible outcome.

Bruce

-- 
You received this message because you are subscribed to the Google Groups 
"Everything List" group.
To unsubscribe from this group and stop receiving emails from it, send an email 
to everything-list+unsubscr...@googlegroups.com.
To view this discussion on the web visit 
https://groups.google.com/d/msgid/everything-list/CAFxXSLSbHt5euur4EGLeObR2pyxwWBQWnV6nuxig3vDHvZdPDA%40mail.gmail.com.


Re: Irreducible randomness in QM

2020-12-22 Thread smitra

On 23-12-2020 03:29, Bruce Kellett wrote:

On Wed, Dec 23, 2020 at 12:45 PM Stathis Papaioannou
 wrote:


On Wed, 23 Dec 2020 at 10:58, Bruce Kellett 
wrote:

On Wed, Dec 23, 2020 at 10:45 AM Stathis Papaioannou
 wrote:

On Wed, 23 Dec 2020 at 09:15, Bruce Kellett 
wrote:

On Wed, Dec 23, 2020 at 9:07 AM Stathis Papaioannou
 wrote:

On Wed, 23 Dec 2020 at 09:02, Bruce Kellett 
wrote:

On Wed, Dec 23, 2020 at 8:32 AM Stathis Papaioannou
 wrote:

On Tue, 22 Dec 2020 at 21:31, Bruce Kellett 
wrote:

On Tue, Dec 22, 2020 at 9:19 PM Stathis Papaioannou
 wrote:

All the copies could be conscious or all could be zombies; none are
privileged.

What difference does that make? One has to be privileged in some way
if there is to be a probability different from zero.


Why did you say it was dualist if it doesn't make a difference that it
isn't dualist?

It makes no difference if all copies are conscious, or if all are
zombies -- you are still making a dualist assumption.


The probability calculated where there are multiple copies is the
probability that one randomly sampled copy will see a particular
outcome. I am one randomly sampled copy.


And that is precisely the dualist assumption that  is intrinsic in all
self-location (indexical) arguments. I think Brent has understood this
when he says "That seems to imply dualism.  All the bodies exist, but
your soul only goes with one."

I could say that my soul is duplicated and I want to know the
probability that I am one randomly sampled soul. I could say that the
carrots are duplicated and I want to know the probability that I get a
particular randomly sampled carrot. I don't have a problem with it;
you do, and there seems to be no way around it.

Think of it like this: take a randomly shuffled deck of cards and hand
one card from the deck to each of 52 people. The probability that one
of the people will get the 3-of-Spades is one. The probability that
'You' will get the 3-of-Spades in a fair shuffle is 1/52. The
difference is that you have identified yourself in advance. The
dualist assumption is equivalent.

Let's say you are copied 10^100 times. One copy will end up in a place
where they use euros and the rest will end up in a place where they
use dollars. Do you put euros or dollars in your wallet before
duplication?

Let's say I wait and see! and go to the money exchange if necessary.
You are posing a different problem, one in which the number of copies
on a particular branch is increased. That is incompatible with MWI and
Everett with non-degenerate eigenvalues.

You don't avoid the dualist implications of self-selection by
increasing the number of copies: the example with 52 cards says
everything that is necessary.

From what I understand of your position, you would claim that the 1 in
10^100 copy will screw up the very concept of probability. If that
extra copy did not exist, you would take dollars, because you will
certainly need dollars; but with the extra copy you would just throw
up your hands and say you don't know what to do, because it is certain
you will need dollars and euros.

My complaint about your example is that you are changing the problem
-- you are changing the probabilities in a way that is incompatible
with both the Schrodinger equation and the Born rule. But there could
be more moderate examples of branch duplication that would be more in
line with what is proposed by some people. For example, both Sean
Carroll and Zurek propose a procedure whereby they expand the number
of branches so that all branches have equal amplitudes (weights, or
Born probabilities). This is incompatible with the Schrodinger
equation, but if we leave that aside for the moment, it gives a
branch-counting solution to the probability question. The idea then is
that you self-select from a uniform random distribution over this
expanded set of branches.  However, the expansion of the number of
branches in this approach is, in fact, unnecessary, since random
self-selection from a distribution would give the same result if the
distribution were determined directly by the Born rule.

But this is still inconsistent with the Schrodinger equation because
there is nothing in the SE that tells you that you have a probability
distribution given by the Born weights. You can impose the Born rule
by fiat, but that is then incompatible with the fact that every
outcome in the Schrodinger equation occurs with probability equal to
one. (Which is where we started).

The self-selection idea, whether from an expanded set of branches with
equal weights, or from the original number of branches weighted by the
Born rule, still involves the idea of a random selection from a
distribution. This is not part of the Schrodinger equation, and it is
still essentially dualist since it requires the selection of one
unique individual who is not specified by the equations -- it assumes
that just one of the individuals involved is uniquely specified to be
YOU -- by virtue of an immortal soul or some 

Re: Irreducible randomness in QM

2020-12-22 Thread Bruce Kellett
On Wed, Dec 23, 2020 at 12:45 PM Stathis Papaioannou 
wrote:

> On Wed, 23 Dec 2020 at 10:58, Bruce Kellett  wrote:
>
>> On Wed, Dec 23, 2020 at 10:45 AM Stathis Papaioannou 
>> wrote:
>>
>>> On Wed, 23 Dec 2020 at 09:15, Bruce Kellett 
>>> wrote:
>>>
 On Wed, Dec 23, 2020 at 9:07 AM Stathis Papaioannou 
 wrote:

> On Wed, 23 Dec 2020 at 09:02, Bruce Kellett 
> wrote:
>
>> On Wed, Dec 23, 2020 at 8:32 AM Stathis Papaioannou <
>> stath...@gmail.com> wrote:
>>
>>> On Tue, 22 Dec 2020 at 21:31, Bruce Kellett 
>>> wrote:
>>>
 On Tue, Dec 22, 2020 at 9:19 PM Stathis Papaioannou <
 stath...@gmail.com> wrote:

>
> All the copies could be conscious or all could be zombies; none
> are privileged.
>

 What difference does that make? One has to be privileged in some
 way if there is to be a probability different from zero.

>>>
>>> Why did you say it was dualist if it doesn't make a difference that
>>> it isn't dualist?
>>>
>>
>> It makes no difference if all copies are conscious, or if all are
>> zombies -- you are still making a dualist assumption.
>>
>> The probability calculated where there are multiple copies is the
>>> probability that one randomly sampled copy will see a particular 
>>> outcome. I
>>> am one randomly sampled copy.
>>>
>>
>>
>> And that is precisely the dualist assumption that  is intrinsic in
>> all self-location (indexical) arguments. I think Brent has understood 
>> this
>> when he says "That seems to imply dualism.  All the bodies exist, but 
>> your
>> soul only goes with one."
>>
>
> I could say that my soul is duplicated and I want to know the
> probability that I am one randomly sampled soul. I could say that the
> carrots are duplicated and I want to know the probability that I get a
> particular randomly sampled carrot. I don't have a problem with it; you 
> do,
> and there seems to be no way around it.
>


 Think of it like this: take a randomly shuffled deck of cards and hand
 one card from the deck to each of 52 people. The probability that one of
 the people will get the 3-of-Spades is one. The probability that 'You' will
 get the 3-of-Spades in a fair shuffle is 1/52. The difference is that you
 have identified yourself in advance. The dualist assumption is equivalent.

>>>
>>> Let's say you are copied 10^100 times. One copy will end up in a place
>>> where they use euros and the rest will end up in a place where they use
>>> dollars. Do you put euros or dollars in your wallet before duplication?
>>>
>>
>>
>> Let's say I wait and see! and go to the money exchange if necessary. You
>> are posing a different problem, one in which the number of copies on a
>> particular branch is increased. That is incompatible with MWI and Everett
>> with non-degenerate eigenvalues.
>>
>> You don't avoid the dualist implications of self-selection by increasing
>> the number of copies: the example with 52 cards says everything that is
>> necessary.
>>
>
> From what I understand of your position, you would claim that the 1 in
> 10^100 copy will screw up the very concept of probability. If that extra
> copy did not exist, you would take dollars, because you will certainly need
> dollars; but with the extra copy you would just throw up your hands and say
> you don't know what to do, because it is certain you will need dollars and
> euros.
>


My complaint about your example is that you are changing the problem -- you
are changing the probabilities in a way that is incompatible with both the
Schrodinger equation and the Born rule. But there could be more moderate
examples of branch duplication that would be more in line with what is
proposed by some people. For example, both Sean Carroll and Zurek propose a
procedure whereby they expand the number of branches so that all branches
have equal amplitudes (weights, or Born probabilities). This is
incompatible with the Schrodinger equation, but if we leave that aside for
the moment, it gives a branch-counting solution to the probability
question. The idea then is that you self-select from a uniform random
distribution over this expanded set of branches.  However, the expansion of
the number of branches in this approach is, in fact, unnecessary, since
random self-selection from a distribution would give the same result if the
distribution were determined directly by the Born rule.

But this is still inconsistent with the Schrodinger equation because there
is nothing in the SE that tells you that you have a probability
distribution given by the Born weights. You can impose the Born rule by
fiat, but that is then incompatible with the fact that every outcome in the
Schrodinger equation occurs with probability equal to one. (Which is where
we started).

The self-selection idea, 

Re: Irreducible randomness in QM

2020-12-22 Thread Stathis Papaioannou
On Wed, 23 Dec 2020 at 10:58, Bruce Kellett  wrote:

> On Wed, Dec 23, 2020 at 10:45 AM Stathis Papaioannou 
> wrote:
>
>> On Wed, 23 Dec 2020 at 09:15, Bruce Kellett 
>> wrote:
>>
>>> On Wed, Dec 23, 2020 at 9:07 AM Stathis Papaioannou 
>>> wrote:
>>>
 On Wed, 23 Dec 2020 at 09:02, Bruce Kellett 
 wrote:

> On Wed, Dec 23, 2020 at 8:32 AM Stathis Papaioannou <
> stath...@gmail.com> wrote:
>
>> On Tue, 22 Dec 2020 at 21:31, Bruce Kellett 
>> wrote:
>>
>>> On Tue, Dec 22, 2020 at 9:19 PM Stathis Papaioannou <
>>> stath...@gmail.com> wrote:
>>>

 All the copies could be conscious or all could be zombies; none are
 privileged.

>>>
>>> What difference does that make? One has to be privileged in some way
>>> if there is to be a probability different from zero.
>>>
>>
>> Why did you say it was dualist if it doesn't make a difference that
>> it isn't dualist?
>>
>
> It makes no difference if all copies are conscious, or if all are
> zombies -- you are still making a dualist assumption.
>
> The probability calculated where there are multiple copies is the
>> probability that one randomly sampled copy will see a particular 
>> outcome. I
>> am one randomly sampled copy.
>>
>
>
> And that is precisely the dualist assumption that  is intrinsic in all
> self-location (indexical) arguments. I think Brent has understood this 
> when
> he says "That seems to imply dualism.  All the bodies exist, but your soul
> only goes with one."
>

 I could say that my soul is duplicated and I want to know the
 probability that I am one randomly sampled soul. I could say that the
 carrots are duplicated and I want to know the probability that I get a
 particular randomly sampled carrot. I don't have a problem with it; you do,
 and there seems to be no way around it.

>>>
>>>
>>> Think of it like this: take a randomly shuffled deck of cards and hand
>>> one card from the deck to each of 52 people. The probability that one of
>>> the people will get the 3-of-Spades is one. The probability that 'You' will
>>> get the 3-of-Spades in a fair shuffle is 1/52. The difference is that you
>>> have identified yourself in advance. The dualist assumption is equivalent.
>>>
>>
>> Let's say you are copied 10^100 times. One copy will end up in a place
>> where they use euros and the rest will end up in a place where they use
>> dollars. Do you put euros or dollars in your wallet before duplication?
>>
>
>
> Let's say I wait and see! and go to the money exchange if necessary. You
> are posing a different problem, one in which the number of copies on a
> particular branch is increased. That is incompatible with MWI and Everett
> with non-degenerate eigenvalues.
>
> You don't avoid the dualist implications of self-selection by increasing
> the number of copies: the example with 52 cards says everything that is
> necessary.
>

>From what I understand of your position, you would claim that the 1 in
10^100 copy will screw up the very concept of probability. If that extra
copy did not exist, you would take dollars, because you will certainly need
dollars; but with the extra copy you would just throw up your hands and say
you don't know what to do, because it is certain you will need dollars and
euros.


-- 
Stathis Papaioannou

-- 
You received this message because you are subscribed to the Google Groups 
"Everything List" group.
To unsubscribe from this group and stop receiving emails from it, send an email 
to everything-list+unsubscr...@googlegroups.com.
To view this discussion on the web visit 
https://groups.google.com/d/msgid/everything-list/CAH%3D2ypXKQiAuhicuU4Wu-3j7_D0djZdWxiafaj%2B_b9%2BmtFj4bw%40mail.gmail.com.


Re: Irreducible randomness in QM

2020-12-22 Thread Bruce Kellett
On Wed, Dec 23, 2020 at 10:45 AM Stathis Papaioannou 
wrote:

> On Wed, 23 Dec 2020 at 09:15, Bruce Kellett  wrote:
>
>> On Wed, Dec 23, 2020 at 9:07 AM Stathis Papaioannou 
>> wrote:
>>
>>> On Wed, 23 Dec 2020 at 09:02, Bruce Kellett 
>>> wrote:
>>>
 On Wed, Dec 23, 2020 at 8:32 AM Stathis Papaioannou 
 wrote:

> On Tue, 22 Dec 2020 at 21:31, Bruce Kellett 
> wrote:
>
>> On Tue, Dec 22, 2020 at 9:19 PM Stathis Papaioannou <
>> stath...@gmail.com> wrote:
>>
>>>
>>> All the copies could be conscious or all could be zombies; none are
>>> privileged.
>>>
>>
>> What difference does that make? One has to be privileged in some way
>> if there is to be a probability different from zero.
>>
>
> Why did you say it was dualist if it doesn't make a difference that it
> isn't dualist?
>

 It makes no difference if all copies are conscious, or if all are
 zombies -- you are still making a dualist assumption.

 The probability calculated where there are multiple copies is the
> probability that one randomly sampled copy will see a particular outcome. 
> I
> am one randomly sampled copy.
>


 And that is precisely the dualist assumption that  is intrinsic in all
 self-location (indexical) arguments. I think Brent has understood this when
 he says "That seems to imply dualism.  All the bodies exist, but your soul
 only goes with one."

>>>
>>> I could say that my soul is duplicated and I want to know the
>>> probability that I am one randomly sampled soul. I could say that the
>>> carrots are duplicated and I want to know the probability that I get a
>>> particular randomly sampled carrot. I don't have a problem with it; you do,
>>> and there seems to be no way around it.
>>>
>>
>>
>> Think of it like this: take a randomly shuffled deck of cards and hand
>> one card from the deck to each of 52 people. The probability that one of
>> the people will get the 3-of-Spades is one. The probability that 'You' will
>> get the 3-of-Spades in a fair shuffle is 1/52. The difference is that you
>> have identified yourself in advance. The dualist assumption is equivalent.
>>
>
> Let's say you are copied 10^100 times. One copy will end up in a place
> where they use euros and the rest will end up in a place where they use
> dollars. Do you put euros or dollars in your wallet before duplication?
>


Let's say I wait and see! and go to the money exchange if necessary. You
are posing a different problem, one in which the number of copies on a
particular branch is increased. That is incompatible with MWI and Everett
with non-degenerate eigenvalues.

You don't avoid the dualist implications of self-selection by increasing
the number of copies: the example with 52 cards says everything that is
necessary.

Bruce

-- 
You received this message because you are subscribed to the Google Groups 
"Everything List" group.
To unsubscribe from this group and stop receiving emails from it, send an email 
to everything-list+unsubscr...@googlegroups.com.
To view this discussion on the web visit 
https://groups.google.com/d/msgid/everything-list/CAFxXSLQom9aPN9qCD5hjrbwSc%3Dtpd42QdoOn_3BPT%2BCuA%3DsuDQ%40mail.gmail.com.


Re: Irreducible randomness in QM

2020-12-22 Thread Stathis Papaioannou
On Wed, 23 Dec 2020 at 09:15, Bruce Kellett  wrote:

> On Wed, Dec 23, 2020 at 9:07 AM Stathis Papaioannou 
> wrote:
>
>> On Wed, 23 Dec 2020 at 09:02, Bruce Kellett 
>> wrote:
>>
>>> On Wed, Dec 23, 2020 at 8:32 AM Stathis Papaioannou 
>>> wrote:
>>>
 On Tue, 22 Dec 2020 at 21:31, Bruce Kellett 
 wrote:

> On Tue, Dec 22, 2020 at 9:19 PM Stathis Papaioannou <
> stath...@gmail.com> wrote:
>
>>
>> All the copies could be conscious or all could be zombies; none are
>> privileged.
>>
>
> What difference does that make? One has to be privileged in some way
> if there is to be a probability different from zero.
>

 Why did you say it was dualist if it doesn't make a difference that it
 isn't dualist?

>>>
>>> It makes no difference if all copies are conscious, or if all are
>>> zombies -- you are still making a dualist assumption.
>>>
>>> The probability calculated where there are multiple copies is the
 probability that one randomly sampled copy will see a particular outcome. I
 am one randomly sampled copy.

>>>
>>>
>>> And that is precisely the dualist assumption that  is intrinsic in all
>>> self-location (indexical) arguments. I think Brent has understood this when
>>> he says "That seems to imply dualism.  All the bodies exist, but your soul
>>> only goes with one."
>>>
>>
>> I could say that my soul is duplicated and I want to know the probability
>> that I am one randomly sampled soul. I could say that the carrots are
>> duplicated and I want to know the probability that I get a particular
>> randomly sampled carrot. I don't have a problem with it; you do, and there
>> seems to be no way around it.
>>
>
>
> Think of it like this: take a randomly shuffled deck of cards and hand one
> card from the deck to each of 52 people. The probability that one of the
> people will get the 3-of-Spades is one. The probability that 'You' will get
> the 3-of-Spades in a fair shuffle is 1/52. The difference is that you have
> identified yourself in advance. The dualist assumption is equivalent.
>

Let's say you are copied 10^100 times. One copy will end up in a place
where they use euros and the rest will end up in a place where they use
dollars. Do you put euros or dollars in your wallet before duplication?


-- 
Stathis Papaioannou

-- 
You received this message because you are subscribed to the Google Groups 
"Everything List" group.
To unsubscribe from this group and stop receiving emails from it, send an email 
to everything-list+unsubscr...@googlegroups.com.
To view this discussion on the web visit 
https://groups.google.com/d/msgid/everything-list/CAH%3D2ypXArsGJhQAOQ9AMRs%3Dz3DH%2B0KVWpNmBZXUo9W9SzuK1hA%40mail.gmail.com.


Re: Irreducible randomness in QM

2020-12-22 Thread Bruce Kellett
On Wed, Dec 23, 2020 at 10:35 AM Stathis Papaioannou 
wrote:

> On Wed, 23 Dec 2020 at 09:15, Bruce Kellett  wrote:
>
>> On Wed, Dec 23, 2020 at 9:07 AM Stathis Papaioannou 
>> wrote:
>>
>>> On Wed, 23 Dec 2020 at 09:02, Bruce Kellett 
>>> wrote:
>>>
 On Wed, Dec 23, 2020 at 8:32 AM Stathis Papaioannou 
 wrote:

> On Tue, 22 Dec 2020 at 21:31, Bruce Kellett 
> wrote:
>
>> On Tue, Dec 22, 2020 at 9:19 PM Stathis Papaioannou <
>> stath...@gmail.com> wrote:
>>
>>>
>>> All the copies could be conscious or all could be zombies; none are
>>> privileged.
>>>
>>
>> What difference does that make? One has to be privileged in some way
>> if there is to be a probability different from zero.
>>
>
> Why did you say it was dualist if it doesn't make a difference that it
> isn't dualist?
>

 It makes no difference if all copies are conscious, or if all are
 zombies -- you are still making a dualist assumption.

 The probability calculated where there are multiple copies is the
> probability that one randomly sampled copy will see a particular outcome. 
> I
> am one randomly sampled copy.
>


 And that is precisely the dualist assumption that  is intrinsic in all
 self-location (indexical) arguments. I think Brent has understood this when
 he says "That seems to imply dualism.  All the bodies exist, but your soul
 only goes with one."

>>>
>>> I could say that my soul is duplicated and I want to know the
>>> probability that I am one randomly sampled soul. I could say that the
>>> carrots are duplicated and I want to know the probability that I get a
>>> particular randomly sampled carrot. I don't have a problem with it; you do,
>>> and there seems to be no way around it.
>>>
>>
>>
>> Think of it like this: take a randomly shuffled deck of cards and hand
>> one card from the deck to each of 52 people. The probability that one of
>> the people will get the 3-of-Spades is one. The probability that 'You' will
>> get the 3-of-Spades in a fair shuffle is 1/52. The difference is that you
>> have identified yourself in advance. The dualist assumption is equivalent.
>>
>
> The probability that one particular randomly sampled person will get the 3
> of spades is 1/52.
>


As I said. But who or what does the random sampling so that you are the
selected person? You cannot escape the dualist implications that easily.

Bruce

-- 
You received this message because you are subscribed to the Google Groups 
"Everything List" group.
To unsubscribe from this group and stop receiving emails from it, send an email 
to everything-list+unsubscr...@googlegroups.com.
To view this discussion on the web visit 
https://groups.google.com/d/msgid/everything-list/CAFxXSLSriUUBW59tZzT3vb%3D39DRKd%3DR%3DOq8ZhcShnJ-FN%2BY1ZQ%40mail.gmail.com.


Re: Irreducible randomness in QM

2020-12-22 Thread Stathis Papaioannou
On Wed, 23 Dec 2020 at 09:15, Bruce Kellett  wrote:

> On Wed, Dec 23, 2020 at 9:07 AM Stathis Papaioannou 
> wrote:
>
>> On Wed, 23 Dec 2020 at 09:02, Bruce Kellett 
>> wrote:
>>
>>> On Wed, Dec 23, 2020 at 8:32 AM Stathis Papaioannou 
>>> wrote:
>>>
 On Tue, 22 Dec 2020 at 21:31, Bruce Kellett 
 wrote:

> On Tue, Dec 22, 2020 at 9:19 PM Stathis Papaioannou <
> stath...@gmail.com> wrote:
>
>>
>> All the copies could be conscious or all could be zombies; none are
>> privileged.
>>
>
> What difference does that make? One has to be privileged in some way
> if there is to be a probability different from zero.
>

 Why did you say it was dualist if it doesn't make a difference that it
 isn't dualist?

>>>
>>> It makes no difference if all copies are conscious, or if all are
>>> zombies -- you are still making a dualist assumption.
>>>
>>> The probability calculated where there are multiple copies is the
 probability that one randomly sampled copy will see a particular outcome. I
 am one randomly sampled copy.

>>>
>>>
>>> And that is precisely the dualist assumption that  is intrinsic in all
>>> self-location (indexical) arguments. I think Brent has understood this when
>>> he says "That seems to imply dualism.  All the bodies exist, but your soul
>>> only goes with one."
>>>
>>
>> I could say that my soul is duplicated and I want to know the probability
>> that I am one randomly sampled soul. I could say that the carrots are
>> duplicated and I want to know the probability that I get a particular
>> randomly sampled carrot. I don't have a problem with it; you do, and there
>> seems to be no way around it.
>>
>
>
> Think of it like this: take a randomly shuffled deck of cards and hand one
> card from the deck to each of 52 people. The probability that one of the
> people will get the 3-of-Spades is one. The probability that 'You' will get
> the 3-of-Spades in a fair shuffle is 1/52. The difference is that you have
> identified yourself in advance. The dualist assumption is equivalent.
>

The probability that one particular randomly sampled person will get the 3
of spades is 1/52.


-- 
Stathis Papaioannou

-- 
You received this message because you are subscribed to the Google Groups 
"Everything List" group.
To unsubscribe from this group and stop receiving emails from it, send an email 
to everything-list+unsubscr...@googlegroups.com.
To view this discussion on the web visit 
https://groups.google.com/d/msgid/everything-list/CAH%3D2ypVOhW1H7%3D08FMNpbUV%3DXrtSEm8Xf1-qA%3DkQ7%3Dt_i9AFbA%40mail.gmail.com.


Re: Irreducible randomness in QM

2020-12-22 Thread Bruce Kellett
On Wed, Dec 23, 2020 at 9:07 AM Stathis Papaioannou 
wrote:

> On Wed, 23 Dec 2020 at 09:02, Bruce Kellett  wrote:
>
>> On Wed, Dec 23, 2020 at 8:32 AM Stathis Papaioannou 
>> wrote:
>>
>>> On Tue, 22 Dec 2020 at 21:31, Bruce Kellett 
>>> wrote:
>>>
 On Tue, Dec 22, 2020 at 9:19 PM Stathis Papaioannou 
 wrote:

>
> All the copies could be conscious or all could be zombies; none are
> privileged.
>

 What difference does that make? One has to be privileged in some way if
 there is to be a probability different from zero.

>>>
>>> Why did you say it was dualist if it doesn't make a difference that it
>>> isn't dualist?
>>>
>>
>> It makes no difference if all copies are conscious, or if all are zombies
>> -- you are still making a dualist assumption.
>>
>> The probability calculated where there are multiple copies is the
>>> probability that one randomly sampled copy will see a particular outcome. I
>>> am one randomly sampled copy.
>>>
>>
>>
>> And that is precisely the dualist assumption that  is intrinsic in all
>> self-location (indexical) arguments. I think Brent has understood this when
>> he says "That seems to imply dualism.  All the bodies exist, but your soul
>> only goes with one."
>>
>
> I could say that my soul is duplicated and I want to know the probability
> that I am one randomly sampled soul. I could say that the carrots are
> duplicated and I want to know the probability that I get a particular
> randomly sampled carrot. I don't have a problem with it; you do, and there
> seems to be no way around it.
>


Think of it like this: take a randomly shuffled deck of cards and hand one
card from the deck to each of 52 people. The probability that one of the
people will get the 3-of-Spades is one. The probability that 'You' will get
the 3-of-Spades in a fair shuffle is 1/52. The difference is that you have
identified yourself in advance. The dualist assumption is equivalent.

Bruce

-- 
You received this message because you are subscribed to the Google Groups 
"Everything List" group.
To unsubscribe from this group and stop receiving emails from it, send an email 
to everything-list+unsubscr...@googlegroups.com.
To view this discussion on the web visit 
https://groups.google.com/d/msgid/everything-list/CAFxXSLSnFR3wapGUcv%2B6weCOR%2BEjcquHsk5GaRzmiCSYgx13vw%40mail.gmail.com.


Re: Irreducible randomness in QM

2020-12-22 Thread Stathis Papaioannou
On Wed, 23 Dec 2020 at 09:02, Bruce Kellett  wrote:

> On Wed, Dec 23, 2020 at 8:32 AM Stathis Papaioannou 
> wrote:
>
>> On Tue, 22 Dec 2020 at 21:31, Bruce Kellett 
>> wrote:
>>
>>> On Tue, Dec 22, 2020 at 9:19 PM Stathis Papaioannou 
>>> wrote:
>>>

 All the copies could be conscious or all could be zombies; none are
 privileged.

>>>
>>> What difference does that make? One has to be privileged in some way if
>>> there is to be a probability different from zero.
>>>
>>
>> Why did you say it was dualist if it doesn't make a difference that it
>> isn't dualist?
>>
>
> It makes no difference if all copies are conscious, or if all are zombies
> -- you are still making a dualist assumption.
>
> The probability calculated where there are multiple copies is the
>> probability that one randomly sampled copy will see a particular outcome. I
>> am one randomly sampled copy.
>>
>
>
> And that is precisely the dualist assumption that  is intrinsic in all
> self-location (indexical) arguments. I think Brent has understood this when
> he says "That seems to imply dualism.  All the bodies exist, but your soul
> only goes with one."
>

I could say that my soul is duplicated and I want to know the probability
that I am one randomly sampled soul. I could say that the carrots are
duplicated and I want to know the probability that I get a particular
randomly sampled carrot. I don't have a problem with it; you do, and there
seems to be no way around it.


-- 
Stathis Papaioannou

-- 
You received this message because you are subscribed to the Google Groups 
"Everything List" group.
To unsubscribe from this group and stop receiving emails from it, send an email 
to everything-list+unsubscr...@googlegroups.com.
To view this discussion on the web visit 
https://groups.google.com/d/msgid/everything-list/CAH%3D2ypXr8AM8Z4LqX1QW%3D%3D7Tds4%2BziwV8qC7E2FgNYXszZUMaw%40mail.gmail.com.


Re: Irreducible randomness in QM

2020-12-22 Thread Bruce Kellett
On Wed, Dec 23, 2020 at 8:32 AM Stathis Papaioannou 
wrote:

> On Tue, 22 Dec 2020 at 21:31, Bruce Kellett  wrote:
>
>> On Tue, Dec 22, 2020 at 9:19 PM Stathis Papaioannou 
>> wrote:
>>
>>>
>>> All the copies could be conscious or all could be zombies; none are
>>> privileged.
>>>
>>
>> What difference does that make? One has to be privileged in some way if
>> there is to be a probability different from zero.
>>
>
> Why did you say it was dualist if it doesn't make a difference that it
> isn't dualist?
>

It makes no difference if all copies are conscious, or if all are zombies
-- you are still making a dualist assumption.

The probability calculated where there are multiple copies is the
> probability that one randomly sampled copy will see a particular outcome. I
> am one randomly sampled copy.
>


And that is precisely the dualist assumption that  is intrinsic in all
self-location (indexical) arguments. I think Brent has understood this when
he says "That seems to imply dualism.  All the bodies exist, but your soul
only goes with one."

Bruce

-- 
You received this message because you are subscribed to the Google Groups 
"Everything List" group.
To unsubscribe from this group and stop receiving emails from it, send an email 
to everything-list+unsubscr...@googlegroups.com.
To view this discussion on the web visit 
https://groups.google.com/d/msgid/everything-list/CAFxXSLSZ08NaQyLcpaGsjsZ55vwh0zn1q7M-Dakmr-AWQwzsig%40mail.gmail.com.


Re: Irreducible randomness in QM

2020-12-22 Thread Bruce Kellett
On Wed, Dec 23, 2020 at 2:37 AM Bruno Marchal  wrote:

> Am Mo, 21. Dez 2020, um 21:35, schrieb Bruce Kellett:
>
> On Mon, Dec 21, 2020 at 10:56 PM John Clark  wrote:
>
>
> On Sun, Dec 20, 2020 at 1:38 AM Bruce Kellett 
> wrote:
>
> *> MWI is incompatible with the Born Rule*
>
>
> How do you figure that?
>
>
>
> It's easy enough. MWI from the Schrodinger equation says that every
> outcome happens, with probability one.
>
>
> That is an exemple of confusion between the third person pictured where
> indeed MWI keep all superposition “intacte”, and that all possible quantum
> outcome are realised, and the first person account where that does not
> happen, as each brain is correlated to the terms of the superposition.
>

The 1p/3p distinction does not help you here. There are many 1p views, and
for each the probability of that particular observation is one,
contradicting the Born rule calculation of the probability in every case.
This follows from the linearity of the Schrodinger equation. The Born rule
cannot be deduced from the Schrodinger equation -- they are incompatible.

Bruce

-- 
You received this message because you are subscribed to the Google Groups 
"Everything List" group.
To unsubscribe from this group and stop receiving emails from it, send an email 
to everything-list+unsubscr...@googlegroups.com.
To view this discussion on the web visit 
https://groups.google.com/d/msgid/everything-list/CAFxXSLQkBEnR3Q32hn9xXyeT3D%2BBB26_nkv7QbCfH-WSRnfvxQ%40mail.gmail.com.


  1   2   >